You are on page 1of 465

A Teaching Atlas of

Case Studies in
Diagnostic Imaging
A Teaching Atlas of
Case Studies in
Diagnostic Imaging
Editors
Funsho Komolafe 
MBBS FMCR FWACS FICA FRCR
Adjunct Professor of Radiology
Ras al-Khaimah Medical and
Health Sciences University
United Arab Emirates
Senior Consultant Radiologist
Fujairah Hospital
United Arab Emirates
Former Professor and Head of Radiology
University of Ilorin, Nigeria
Former Professor and Chairman of Radiology
King Faisal University
Dammam, Saudi Arabia

M Haroun Dahniya 
MBBS (Durham) DMRD FRCR FWACS
Senior Consultant Radiologist
Fujairah Hospital
United Arab Emirates
Former Professor and Head of Radiology
University of Maiduguri, Nigeria
Former Director, FRCR and FFR (RCSI) Training Program
Ministry of Health, Kuwait

Foreword
SB Lagundoye

The Health Sciences Publisher


New Delhi | London | Philadelphia | Panama
  Jaypee Brothers Medical Publishers (P) Ltd.
Headquarters
Jaypee Brothers Medical Publishers (P) Ltd.
4838/24, Ansari Road, Daryaganj
New Delhi 110 002, India
Phone: +91-11-43574357
Fax: +91-11-43574314
E-mail: jaypee@jaypeebrothers.com

Overseas Offices
J.P. Medical Ltd. Jaypee-Highlights Medical Publishers Inc.
83, Victoria Street, London City of Knowledge, Bld. 237, Clayton
SW1H 0HW (UK) Panama City, Panama
Phone: +44-20 3170 8910 Phone: +1 507-301-0496
Fax: +44(0) 20 3008 6180 Fax: +1 507-301-0499
E-mail: info@jpmedpub.com E-mail: cservice@jphmedical.com

Jaypee Medical Inc. Jaypee Brothers Medical Publishers (P) Ltd.


The Bourse 17/1-B, Babar Road, Block-B, Shaymali
111, South Independence Mall East Mohammadpur, Dhaka-1207
Suite 835, Philadelphia, PA 19106, USA Bangladesh
Phone: +1 267-519-9789 Mobile: +08801912003485
E-mail: jpmed.us@gmail.com E-mail: jaypeedhaka@gmail.com
Jaypee Brothers Medical Publishers (P) Ltd.
Bhotahity, Kathmandu, Nepal
Phone: +977-9741283608
E-mail: kathmandu@jaypeebrothers.com

Website: www.jaypeebrothers.com
Website: www.jaypeedigital.com
© 2016, Jaypee Brothers Medical Publishers
The views and opinions expressed in this book are solely those of the original contributor(s)/author(s) and do not
necessarily represent those of editor(s) of the book.
All rights reserved. No part of this publication may be reproduced, stored or transmitted in any form or by any means,
electronic, mechanical, photo­copying, recording or otherwise, without the prior permission in writing of the publishers.
All brand names and product names used in this book are trade names, service marks, trademarks or registered
trademarks of their respective owners. The publisher is not associated with any product or vendor mentioned in this book.
Medical knowledge and practice change constantly. This book is designed to provide accurate, authoritative information
about the subject matter in question. However, readers are advised to check the most current information available on
procedures included and check information from the manufacturer of each product to be administered, to verify the
recommended dose, formula, method and duration of administration, adverse effects and contra­indications. It is the
responsibility of the practitioner to take all appropriate safety precautions. Neither the publisher nor the author(s)/editor(s)
assume any liability for any injury and/or damage to persons or property arising from or related to use of material in this
book.
This book is sold on the understanding that the publisher is not engaged in providing professional medical services. If
such advice or services are required, the services of a competent medical professional should be sought.
Every effort has been made where necessary to contact holders of copyright to obtain permission to reproduce copyright
material. If any have been inadvertently overlooked, the publisher will be pleased to make the necessary arrangements
at the first opportunity.
Inquiries for bulk sales may be solicited at: jaypee@jaypeebrothers.com

A Teaching Atlas of Case Studies in Diagnostic Imaging

First Edition: 2016


ISBN: 978-93-5152-950-7
Printed at
Dedicated to
our wives and our residents
past and present
Contributors

Abdulkadir Musa Tabari  MBBS FMCR FICS M Haroun Dahniya  MBBS (Durham) DMRD
Professor of Radiology FRCR FWACS
Bayero University, Kano Senior Consultant Radiologist
Consultant Radiologist Fujairah Hospital
Aminu Kano Teaching Hospital United Arab Emirates
Kano, Nigeria Former Professor and Head of Radiology
University of Maiduguri, Nigeria
Adekunle Y Abdulkadir  MBBS FMCR Former Director
Head, Department of Radiology FRCR and FFR (RCSI) Training Program
Federal Medical Center Ministry of Health, Kuwait
Gusau, Nigeria
Visiting Lecturer/Consultant Radiologist Sujatha Rajkumar  MBBS MD
Bayero University, Kano (Radiodiagnosis)
Aminu Kano Teaching Hospital Specialist Radiologist
Kano, Nigeria Fujairah Hospital
United Arab Emirates
Funsho Komolafe  MBBS FMCR FWACS
FICA FRCR
Adjunct Professor of Radiology
Ras al-Khaimah Medical and
Health Sciences University
United Arab Emirates
Senior Consultant Radiologist
Fujairah Hospital
United Arab Emirates
Former Professor and Head of Radiology
University of Ilorin, Nigeria
Former Professor and Chairman of Radiology
King Faisal University, Dammam, Saudi Arabia
Foreword

In the course of being an examiner in several radiology fellowship examinations


for over three decades, I have found that in the last decade, there has been
a decline in the performance of candidates, due to inadequate exposure to
appropriate written text.
It gives me the greatest joy to see the editors of this book, who are themselves
experienced trainers and examiners, add to the available learning resources by
providing in a unique format a text which highlights a large number of images
of unusual cases, each with significant teaching value.
I have gone meticulously through the contents of A Teaching Atlas of Case
Studies in Diagnostic Imaging, and I am convinced that it will go a long way
in stimulating interest and improving the understanding of the subject among trainee radiologists
everywhere. The reader is exposed to a wide range of images that span the whole gamut of diagnostic
imaging, using conventional radiography, ultrasonography, CT and MRI studies. Furthermore,
the book should be of immense benefit to teachers and examiners in radiology, as well as to
clinicians in other related disciplines.
I highly commend the effort that has gone into the production of this work, and I expect that
the book will be a compact volume that will grace the libraries of many medical colleges and their
radiology departments.

SB Lagundoye
MBBS (Lond) DMRD (Edin) FMCR FWACS FRCR FICS
Former Professor and Head
Department of Radiology
University of Ibadan, Nigeria
Preface

A Teaching Atlas of Case Studies in Diagnostic Imaging was motivated by the desire to share
with trainee radiologists our experience as teachers and examiners in the specialty. The images
are selected largely from our personal collections acquired over many years and across several
countries.
The cases are selected, either because they are rare or uncommon, or because they represent
unusual images of common pathologies. Each case is backed with a short clinical presentation,
a description of the main radiological features of the images, a short discussion, and a few
relevant references and suggestions for further reading. These suggestions have been selected and
are designed to guide the reader to the most relevant and current literature on the subject. The
cases collected are organized into chapters and span the major divisions of diagnostic imaging—
chest, musculoskeletal, urogenital, gastrointestinal and neuroradiological, with a chapter on
miscellaneous items, containing small parts ultrasound images and a mix-bag of images from
various organ systems.
The book is intended in particular for radiology residents preparing for the various qualifying
fellowship and board examinations. The images shown to candidates in the various examinations
keep changing. It is obvious that the more images a candidate is familiar with, the better the chances
of success.
In addition, it is our hope that this atlas will be found useful in meeting some of the diagnostic
challenges that practicing radiologists encounter in their daily practice. We also believe that
clinicians in other disciplines, especially surgery, pediatrics, medicine, obstetrics and gynecology,
will find relevant sections in the atlas that will be beneficial to their practice.
The publisher has kindly designed the layout of the cases with provision of space for the reader
to make notes as required.

Funsho Komolafe
M Haroun Dahniya
Acknowledgments

We would like to acknowledge the supportive role played in the acquisition of these images by our
technicians in various institutions where we have worked such as Nigeria, Sierra Leone, Kuwait,
Saudi Arabia, and the United Arab Emirates. We like to mention Mrs Lalitha C Veetil, sonographer
at Fujairah Hospital, United Arab Emirates, for her invaluable assistance.
We highly appreciate the meticulous and painstaking editorial and computer assistance which
we received from Dr Opeyemi Komolafe and Dr Rotimi Komolafe.
We would also like to express our deep appreciation for the copious encouragement and support
which we received from our wives, Dr (Mrs) Yemi Komolafe and Haja (Mrs) Humu Dahniya, at the
conception of this project and in the process of bringing it to reality.
Contents

1. Chest Imaging 1
2. Musculoskeletal System Imaging 71
3. Urogenital System Imaging 156
4. Gastrointestinal System Imaging 240
5. Neurological System Imaging 318
6. Miscellaneous Images 405
Index443
Abbreviations

ABC : Aneurysmal bone cyst


ACL : Anterior cruciate ligament
ADEM : Acute disseminated encephalomyelitis
AIDS : Acquired immunodeficiency syndrome
ARM : Anorectal malformation
ASD : Atrial septal defect
AVM : Arteriovenous malformation
BI-RADS : Breast imaging reporting and data system
CAM : Cystic adenomatoid malformation
CBD : Common bile duct
DVT : Deep venous thrombosis
ERCP : Endoscopic retrograde cholangiopancreatography
FLAIR : Fluid-attenuated inversion recovery
FMD : Fibromuscular dysplasia
HCC : Hepatocellular carcinoma
HH : Hiatus hernia
HLA : Human leukocyte antigen
HRCT : High-resolution computed tomography
HU : Hounsfield unit
IPF : Idiopathic pulmonary fibrosis
IUCD : Intrauterine contraceptive device
IVC : Inferior vena cava
KUB : Kidneys, ureters and bladder (Abdominal radiograph)
MCUG : Micturating cystourethrogram
MDCT : Multidetector computed tomography
MEN : Multiple endocrine neoplasia
MRCP : Magnetic resonance cholangiopancreatography
MS : Multiple sclerosis
MTS : Mesial temporal sclerosis
NEC : Nectrotizing enterocolitis
PCL : Posterior cruciate ligament
PD : Proton density-weighted MRI
PET : Positron emission tomography
PSC : Primary sclerosing cholangitis
xviii A Teaching Atlas of Case Studies in Diagnostic Imaging

PUJ : Pelvi-ureteric junction


RPA : Retropharyngeal abscess
RTA : Road traffic accident
STIR : Short T1 inversion recovery
T1w : T1-weighted image
T2w : T2-weighted image
UBC : Unicameral bone cyst
UIP : Usual interstitial pneumonia
V-Q : Ventilation-perfusion scan
WHO : World Health Organization
1 Chest Imaging

CASE 1

A 1-year-old boy presents with recurrent cough.

Figure 1  Chest radiograph


showing an enlarged right
thymus with a positive
“sail sign”. The cardiac size
and shape are normal, and
the lung fields are clear

DISCUSSION
The thymus is a lymphatic organ which plays a vital role in the exhibition of cellular and humoral
immunity. It is relatively large in young children, and it is well-demonstrated on the chest radiograph
up to the age of 3 years. As the immune system becomes well-established, the thymus undergoes
progressive involution and becomes replaced by fat.
The morphology of the thymus is very variable, and it can extend superiorly to the thyroid and
inferiorly to the diaphragm. This variable size and appearance can be a cause of misdiagnosis and
2 A Teaching Atlas of Case Studies in Diagnostic Imaging

unnecessary intervention. The thymus can shrink rapidly in response to bodily stress, but during
recovery it may return to its former size or become larger, a process called rebound hyperplasia.
Apart from the chest radiograph, the thymus is frequently imaged by ultrasonography and CT.
On the chest radiograph, the “sail sign” is the straight appearance of the inferior border of the
thymus abutting the transverse fissure. The ‘thymic wave sign’ is created by the impression of the
anterior ribs on the thyroid.

FURTHER READING
1. Han BK, Suh YL, Yoon HK. Thymic ultrasound. Intrathymic anatomy in infants. Pediatr Radiol.
2001;31(7):474-9.
2. Nasseri F, Eftekhari F. Clinical and radiological review of the normal and abnormal thymus: Pearls
and Pitfalls. Radiographics. 2010;30:413-28.
Chest Imaging 3
CASE 2

A 6-month-old baby with a history of recurrent cough and hiccups. He had been treated at a rural
clinic as a case of recurrent pneumonia.

Figure 1  AP chest X-ray


showing multiple
gas-filled viscera in the
left hemithorax, causing
mediastinal shift to the
right (Courtesy: Dr Adekunle
Abdulkadir)

Diagnosis: Congenital diaphragmatic hernia.

DISCUSSION
Congenital diaphragmatic hernia (CDH) is estimated to occur in 1:4,000 live births. It is associated
with a variable degree of pulmonary hypoplasia, as the herniated viscera interfere with normal
development of the lung in utero. In 95% of cases, the diaphragmatic defect occurs on the left and is
posterolateral (Bochdalek). There is a 40–50% association with other malformations such as central
nervous system, digestive, cardiac and urogenital anomalies.
The typical clinical presentation of CDH is respiratory distress occurring immediately after birth
or in the first few hours or days of life. Exceptionally, it can present at an older age, and its symptoms
then frequently reflect gastrointestinal obstruction or mild respiratory symptoms.
Prenatal diagnosis of CDH can be made at ultrasonography and by MRI. The latter more accurately
depicts the lung and herniated bowel which may sometimes be difficult to identify by ultrasound.
4 A Teaching Atlas of Case Studies in Diagnostic Imaging

The recent advances in postnatal intensive respiratory supportive therapy and innovative surgical
techniques in specialized tertiary centers have had a major impact on survival of babies with CDH.
Despite these advances, the mortality rate remains as high as 50–60%.

FURTHER READING
1. Bohn D. Congenital diaphragmatic hernia. Am J Respir Crit Care Med. 2002;166:911-5.
2. Chavhan GB, Babyn PS, Cohen RA, Langer JC. Multimodality imaging of the pediatric diaphragm:
Anatomy and pathologic conditions. Radiographics. 2010;30:1797-817.
3. Grisaru-Granovsky S, Rabinowitz R, Ioscovich A, Elstein D, Schimmel MS. Congenital diaphragmatic
hernia: Review of the literature in reflection of unresolved dilemmas. Acta Paediatr. 2009;98(12):
1874-81.
Chest Imaging 5
CASE 3

A 14-hour-old female newborn was brought to hospital because of a wide chest defect containing
a pulsating mass. She was born to an unbooked 23-year-old woman who had an uneventful home
delivery at term.

Figure 1  Photograph of the


neonate with extra­thoracic
heart (ectopia cordis)
emerging through a sternal
cleft. Note the attachment
of the umbilicus at the
xiphisternum

Figures 2A and B  AP and


lateral babygrams showing
complete extrathoracic
heart. Note the
depressed precordium
(Courtesy: Dr Adekunle
Abdulkadir) A B
6 A Teaching Atlas of Case Studies in Diagnostic Imaging

DISCUSSION
Ectopia cordis is an extremely rare malformation resulting from failure of fusion of the left and the
right sternal anlage at about the ninth embryonic week. It usually occurs as a part of the pentalogy
of Cantrell, also known as thoracoabdominal syndrome or pentalogy syndrome. It is a rare disorder
consisting of defects of the anterior chest and abdominal walls, anterior diaphragm, sternum,
pericardium and heart. This failure of midline fusion of the sternum results in sternal cleft with the
heart being partly or completely outside the thorax. Herniation of bowel and other intra-abdominal
structures through a diaphragmatic defect can lead to significant respiratory embarrassment.
Prenatal diagnosis of thoracic ectopia cordis can readily be made with ultrasonography and fetal
MRI, permitting prenatal counseling, planning for delivery and possible postnatal intervention.
The management of complete ectopic cordis is still very challenging. The chest deformity, with an
anteroposterior diameter less than that of the heart, poses space limitations, with a high possibility
of cardiac tamponade after sternal closure. There is also the challenge of raising a sufficient skin flap
to cover the wide defect. Death ensues within the first year of life in over 90% of the reported cases.

FURTHER READING
1. Cabrera A, Rodrigo D, Luis MT, Pastor E, Galdeano JM, Esteban S. Ectopia cordis and cardiac
anomalies. Rev Esp Cardiol. 2002;55:1209-12.
2. Cantrell JR, Haller JA, Ravitch MM. A syndrome of congenital defects involving the abdominal wall,
sternum, diaphragm, pericardium and heart. Surg Gynecol Obstet 1958;107:602-4.
3. Engum SA. Embryology, sternal clefts, ectopia cordis, and Cantrell’s pentalogy. Semin Pediatr Surg.
2008;17:154-60.
4. Moniotte S, Powell AJ, Barnewolt CE, Annese D, Geva T. Prenatal diagnosis of thoracic ectopia
cordis by real-time fetal cardiac magnetic resonance imaging and by echocardiography. Congenit
Heart Dis. 2008;3:128-31.
Chest Imaging 7
CASE 4A

A male neonate delivered at 32 weeks gestation presented with severe tachypnea shortly after birth.

Figure 1  Chest radiograph


at 1 hour shows diffuse
reticulogranular opacities
in both lungs. The cardiac
size and shape are within
normal limits

Figure 2  A follow-up
radiograph at 3 hours shows
opacity of both lung fields,
despite the insertion of an
endotracheal tube. Note the
presence of extensive air
bronchograms

Diagnosis: Surfactant deficiency syndrome.


8 A Teaching Atlas of Case Studies in Diagnostic Imaging

CASE 4B

Another baby with surfactant deficiency disease had prolonged positive pressure assisted
respiration with high oxygen concentration.

Figure 1  Chest radiograph


shows cystic spaces of
varying sizes in both
lungs with fibrotic
areas, nonuniform
pulmonary aeration and
pulmonary emphysema.
Features are typical of
bronchopulmonary
dysplasia

DISCUSSION
Surfactant deficiency syndrome (SDS), other­ wise known as hyaline membrane disease or
respiratory distress syndrome of the newborn, is typically a disease of the premature neonate
delivered before 36 weeks and weighing less than 2,500 gram. Surfactant helps lower the surface
tension in the alveoli, preventing them from collapsing at each expiration. The premature neonate
is deficient in surfactant, and hence the pulmonary changes.
There is a higher prevalence of SDS in babies delivered by cesarean section and babies of diabetic
mothers.
The hallmark diagnostic features of SDS on chest X-ray are a bell-shaped thorax with reduced lung
volume due to under aeration, a diffuse reticulonodular pattern, and air bronchogram. Apart from
the administration of surfactant, treatment involves mechanical ventilation and administration of
concentrated oxygen.
Chest Imaging 9

Common complications of the oxygen therapy include bronchopulmonary dysplasia (See Case
4B, Fig. 1), pneumothorax and pneumomediastinum.

FURTHER READING
1. Gerten KA, Coonrod DV, Bay RC, et al. Cesarean delivery and respiratory distress syndrome:
Does labor make a difference? Am J Obstet Gynecol. 2005;192(3):1061-4.
2. Paschechera R, Andrisani MC, et al. Diagnostic imaging of hyaline membrane disease. Rays.
2004;29(2):175-8.
3. Singh J, Sinha SK, et al. Long-term follow-up of very low birth weight infants from a neonatal
volume versus pressure mechanical ventilation trial. Arch Dis Child Fetal Neonatal Ed.
2009;94(5):360-2.
10 A Teaching Atlas of Case Studies in Diagnostic Imaging

CASE 5

An 18-year-old male, known sickle cell disease patient, was admitted via emergency with a history
of low-grade fever, cough and acute chest pain. He was mildly jaundiced and tachypneic. Chest
X-rays were obtained on admission and after 24 hours shown in the following figures, respectively.

Figure 1  Plain PA chest X-ray


reveals moderate generalized
cardiomegaly, prominent
main pulmonary and hilar
vessels and left lower lobe
consolidation. The ribs,
scapulae and clavicles show
increased density and coarse
trabeculation

Figure 2  24 hours later,


the chest X-ray shows bilateral
pneumonic consolidation
and left pleural effusion.
The appearances indicate
the so-called acute chest
syndrome (ACS) of sickle cell
disease
Chest Imaging 11

DISCUSSION
There is an increased risk of pneumonia and pulmonary infarction in patients with sickle cell
disease. Distinguishing between these two, both clinically and on imaging, can be difficult. Because
of this, the general term “acute chest syndrome” is often used to describe fever, a pulmonary
process and radiographic evidence of new pulmonary consolidation in a patient with sickle cell
hemoglobinopathy. ACS is a common cause of hospitalization in these patients and mortality.
Radiographic findings include confluent lobar or segmental consolidation, interstitial infiltration,
pulmo­nary edema (alveolar or interstitial), cardio­megaly and rib infarcts. HRCT may reveal areas of
ground-glass opacification and hypo-perfusion.

FURTHER READING
1. Maitre B, Habibi A, Roudol-Thorarah F, et al. Acute chest syndrome in adults with sickle cell
disease. Chest. 2000;117:1386-92.
2. Platt OS. The acute chest syndrome of sickle cell disease. N Eng J Med. 2000;342:1904-7.
12 A Teaching Atlas of Case Studies in Diagnostic Imaging

CASE 6

A 40-year-old female presented with persistent back pain over three months, anorexia and
intermittent fever. Physical examination was normal.

Figure 1  Chest radiograph


shows normal heart and lung
fields. A left paravertebral
soft tissue density is shown
(arrows). Destructive changes
involving the 6th and 7th
dorsal vertebrae were
shown in more penetrated
radiographs

Figure 2A
Chest Imaging 13

Figure 2B
Figures 2A and B  T2-
weighted and T1-contrast
enhanced images show
extensive osteolytic
destruction of vertebrae
and a large paravertebral
abscess with peripheral
contrast enhancement. The
appearances indicate spinal
tuberculosis (Pott’s disease)

DISCUSSION
Tuberculosis remains a worldwide health problem, and is still a leading cause of death, especially
in third world countries in which musculoskeletal involvement remains common. Spinal
tuberculosis may be sus­pected on a chest X-ray as in this case. The dorsolumbar region is the
most commonly affected. Prominent bone destruction in­volving several adjacent vertebral
bodies, intervertebral disc destruction, formation of paravertebral abscesses and skip lesions
are important radiological features. The paravertebral abscesses may calcify, resulting in cord
compression and myelopathic damage.

FURTHER READING
1. Al-Mulhim FA, Ibrahim EM, El-Hassan AY, Moharram HM. Magnetic resonance imaging of
tuberculous spondylitis. Spine. 1995;20(21):2287-92.
2. Harada Y, Tokuda O, Matsunaga M. Magnetic resonance imaging characteristics of tuberculous
spondylitis vs pyogenic spondylitis. Clin Imaging. 2008;32(4):303-9.
3. Roos AE, Meerter EL, Bloem JL, Bleumm RG. MRI of tuberculous spondylitis. AJR Am J Roentgenol.
1986;147:79-82.
14 A Teaching Atlas of Case Studies in Diagnostic Imaging

CASE 7

A 42-year-old male living with AIDS presented with cough, fever and weight loss. A chest X-ray done
on the day of admission is shown.

Figure 1  PA chest
radiograph reveals
multiple cavitary nodules
in both lungs

DISCUSSION
In an AIDS patient, the diagnostic possibilities are Pneumocystis jiroveci pneumonia, tuberculosis,
including Mycobacterium avium complex, Kaposi sarcoma, non-Hodgkins lymphoma and fungal
infections. In a nonimmunocompromised patient, the differential diagnosis will include Wegener’s
granulomatosis, rheumatoid arthritis (Caplan’s syndrome), sarcoidosis, hydatid disease and
necrotizing pneumonia, e.g. Klebsiella pneumoniae.

FURTHER READING
1. Allen CM, Al-Jahdali HH, Irion KL, et al. Imaging lung manifestations of HIV/AIDS. Ann Thorac
Med. 2010;5(4):201-16.
2. Lillington GA, Caskey CI. Evaluation and management of solitary and multiple pulmonary nodules.
Clin Chest Med. 1993;14:111-9.
3. McGuiness G, Graden JF, Bhalla, et al. AIDS related airway disease. AJR Am J Roentgenol.
1997;168:67-77.
Chest Imaging 15
CASE 8

A 55-year-old man presented with progressive right upper limb weakness and slurred speech.

Figure 1  Chest X-ray


showed bilateral ground-
glass bats-wing opacities,
which were confirmed
by the CT images (Figs 2A
and B)

Figure 2A
16 A Teaching Atlas of Case Studies in Diagnostic Imaging

Figure 2B
Figures 2A and B  CT
images

Figure 3  Diffusion-
weighted brain MR
images show a left parietal
hyperintense rounded
lesion and multiple small
hyperintense, including
some ring lesions in the
white matter. They show
no edema. The ventricles
and cisterns are normal
Chest Imaging 17

DISCUSSION
The bilateral bats-wing ground-glass opacities strongly suggest Pneumocystis jiroveci pneumonia.
Hematological examination con­firmed HIV infection. The lesions in the brain demonstrated by MRI
may represent metastatic disease, infections such as tuberculosis or toxoplasmosis or lymphoma.
In an AIDS patient, toxoplasmosis or lymphoma are the most likely possibilities. HIV infection is
one of the major causes of mortality and morbidity, especially in sub-Saharan Africa, where because
of inadequate treatment facilities, the problem remains significant.

FURTHER READING
1. Burrill J, Williams CJ, Bain G, et al. Tuberculosis: A Radiologic Review. Radiographics. 2007;27:
1155-273.
2. McGuinness G. Changing trends in the pulmonary manifestations of AIDS. Radiol Clin North Am.
1997;35:1029-82.
3. Rosen MJ. Pulmonary manifestations of HIV infection. Respirology. 2008;13:181-90.
18 A Teaching Atlas of Case Studies in Diagnostic Imaging

CASE 9

A 23-year-old man presented at the emergency unit with chest pain and difficulty in breathing of
one-week duration, and a one month history of fever, productive cough and weight loss.

Figures 1A and B  (A) Chest


radiograph at presentation
showing an opaque
left hemithorax,
air-bronchogram and
mediastinal shift to the
left. Streaky opacities
are noted in the right
upper and middle zones,
with compensatory
hyperinflation of the right
lung; (B) Closeup of the left
upper zone shows better
demonstration of the air-
bronchogram, indicative of
upper lobe consolidation.
Note the obliteration of the
B left cardiac outline
Chest Imaging 19

Figure 2  Left chest


ultrasound to assess any
pleural fluid shows streaky
hyperechoic shadows from
airfilled bronchi in the
consolidated lung

Figure 3  After 13 months


of anti-TB treatment, there is
a near complete resolution
of the radiological changes.
Note residual left
basal pneumatoceles
(Courtesy: Dr Adekunle
Abdulkadir)
20 A Teaching Atlas of Case Studies in Diagnostic Imaging

DISCUSSION
Pulmonary tuberculosis (PTB) is a common lung infection affecting approximately two billion
people worldwide. Approximately 2 million deaths are reported annually from TB. Pulmonary
consolidation from PTB is commonly limited to the middle or lower lobes or the anterior
segment of an upper lobe. Whole lung consolidation with extensive air bronchogram, as in this
patient, is a rare presentation in PTB. The broad spectrum of radiographic findings in PTB include
consolidation, cavitation, pneumatocele, segmental or lobar atelectasis, pleural effusion, hilar,
mediastinal lymphadenopathy, miliary nodules. A normal chest radiograph may be seen in about
15% of patients.
Computed tomography (CT) is occasionally useful for clarifying confusing findings but has not
been conclusively shown to have a significant impact on patient management.
Ultrasound has found use in the guidance of needle aspiration for establishing etiologic diagnosis
in atypical cases. A heterogeneous iso-echoic density with interspaced echogenic structures
(air-bronchogram) are reported classic ultrasonographic findings in lung consolidation or
atelectasis.

FURTHER READING
1. Andreu J, Caceres J, Pallisa E, Martinez-Rodriguez M. Radiological manifestations of pulmonary
tuberculosis. Eur J Radiol. 2004;51:139-49.
2. Tsao TC, Juang YC, Tsai YH, Lan RS, Lee CH. Whole lung tuberculosis. A disease with high mortality
which is frequently misdiagnosed. Chest. 1992;101:1309-11.
3. Woodring JH, Vandiviere HM, Fried AM, Dillon ML, Williams TD, Melvin IG. Update: the
radiographic features of pulmonary tuberculosis. AJR Am J Roentgenol. 1986;146:497-506.
4. Yang PC, Luh KT, Chang DB, Yu CJ, Kuo SH, Wu HD. Ultrasonographic evaluation of pulmonary
consolidation. Am Rev. Resp Dis. 1992;146:757-62.
Chest Imaging 21
CASE 10

A 41-year-old male was admitted with a history of persistent cough, fever and weight loss.

Figure 1  PA chest
radiograph shows gross
globular cardiomegaly,
suggestive of pericardial
effusion, and bilateral upper
lobe pulmonary infiltrates
consistent with PTB

Figure 2  Axial CT chest


showing bilateral upper
lobe infiltrates, fibrosis and
cysts, consistent with PTB
22 A Teaching Atlas of Case Studies in Diagnostic Imaging

Figure 3  Axial CT chest


with mediastinal window
confirms massive pericardial
effusion. Sputum and
pericardial fluid aspirate
confirmed TB

DISCUSSION
Tuberculosis remains a worldwide problem, mainly because of the upsurge of AIDS and widespread
international travel. The radio­logical features of both primary and postpri­mary TB are well-
established. Pericardial involvement is not uncommon, as in this case.

FURTHER READING
1. Mayosi BM, Burgess LJ, Doubell AF. Heart Disease in Africa: Tuberculous pericarditis. Circulation.
2005;112:3608-16.
2. Woodring JH, Vandiviere HM, Fried AM, et al. Update radiographic features of pul­ monary
tuberculosis. AJR Am J Roentgenol. 1986;146:497-506.
Chest Imaging 23
CASE 11

A 37-year-old man treated for pulmonary tuberculosis several years earlier, complained of recurrent
chest pain and tightness.

Figures 1A and B  Frontal


chest radiograph demon­
strates a thin egg-shell
pericardial calcification and
calcifications (arrow) in the
interventricular groove.
Note also the bilateral
pleural fluid collection
(Courtesy: Dr Adekunle
Abdulkadir) B
24 A Teaching Atlas of Case Studies in Diagnostic Imaging

DISCUSSION
Pericardial calcification is usually preceded by a prior episode of inflammation from a variety of
infections, trauma, and neoplasms. It is strongly associated with constrictive pericarditis, which
occurs when there is interference with diastolic filling of the heart. The patient may present
with fatigue, exertional dyspnea, cough, orthopnea, angina like chest pain, hypotension, reflex
tachycardia, jugular venous distension, hepatomegaly with marked ascites, and peripheral edema.
Pericardial calcification is detected in up to 50% of patients with constrictive pericarditis. Therefore,
constrictive pericarditis may exist without pericardial calcification, and pericardial calcification
can be present without constrictive pericarditis.
Pleural effusions are present in about 60% of patients, and persistent unexplained pleural
effusions can be the presenting manifestation.
CT is the best technique to detect pericardial calcification, but overpenetrated plain films,
fluoroscopy and MRI are helpful. Pericardial calcifications most commonly occur along the inferior
diaphragmatic surface of the pericardium surrounding the ventricles. Pericardial calcification with
thin, egg-shell calcification is more often associated with viral infection or uremia. In addition,
tuberculosis causes irregular slightly coarse calcifications along the atrioventricular groove.

FURTHER READING
1. Langer C, Butz T, Horstkotte D. Multimodality in imaging calcific constrictive pericarditis. Heart.
2006;92(9):1289.
2. Schwefer M, Aschenbach R, Heidemann J, Mey C, Lapp H. Constrictive pericarditis, still a diagnostic
challenge: comprehensive review of clinical management. Eur J Cardiothorac Surg. 2009;36:502-10.
3. Sengupta PP, Eleid MF, Khandheria BK. Constrictive pericarditis. Circ. 2008;72:1555-62.
Chest Imaging 25
CASE 12

A 37-year-old man presented with a 5-year history of dyspnea of increasing severity. Pre­vious
investigations included echocardiogra­phy, multidetector computerized tomography (MDCT) and
cardiac MRI. A chest X-ray was done prior to readmission.

Figure 1  PA chest radiograph


shows a normal sized heart,
marked enlargement of
the main pulmonary and
central hilar arteries, marked
peripheral vascular pruning
and oligemic and hyperlucent
lung fields

Impression: Severe pulmonary arterial hyper­tension, consistent with longstanding arterial septal
defect (ASD) and shunt reversal (Eisenmenger’s syndrome).

DISCUSSION
Echocardiography, MDCT and cardiac MRI confirmed sinus venous type ASD with bidirectional
flow and pulmonary emboli. Pulmonary arterial hypertension is defined as a systolic pressure in
the pulmonary artery exceeding 30 mm Hg measured directly by catheterization of the pulmonary
artery or indirectly by echocardiography. The diagnosis is usually evident from the clinical
history, physical findings and chest radiographic appearances. It may be primary or secondary
to pulmonary venous hypertension, thrombo-embolic disease, pulmonary interstitial diseases,
schistosomiasis, drugs, etc.

FURTHER READING
1. Grosse C, Grosse A. CT findings in diseases associated with pulmonary hypertension: A current
review. Radiographics. 2010;30:1753-77.
2. Ng CS, Wells AU, Padley SP. A CT sign of chronic pulmonary arterial hypertension: The ratio of main
pulmonary artery to aortic diameter. J Thorac Imag. 1999;14:270-8.
3. Peacock AJ. Primary pulmonary hyper­tension. Thorax. 1999;54:1107-18.
26 A Teaching Atlas of Case Studies in Diagnostic Imaging

CASE 13

A 61-year-old male presented with chest pain, dyspnea, cough and hypotension. He has a previous
history of swollen left leg. His chest X-ray was normal, but a V/Q scan shows a high probability of
pulmonary embolism. ACT pulmonary angiogram was done.

Figure 1  Axial CT showing


a saddle embolus extending
across the main right and left
pulmonary arteries

Figure 2  Axial image shows


extension of the emboli into
the lobar and segmental
branches of the pulmonary
arteries
Chest Imaging 27

DISCUSSION
Pulmonary embolism is a major cause of death. Risk factors include hypercoagulable states,
pregnancy, malignancy, surgery, indwelling catheters and DVT. Investigations include Chest X-ray,
ECG, echocardiography, PO2, D-dimer and V/Q scans. Definitive diagnosis is by CT angiography or
catheter angiography. Pulmonary infarction and pleural effusions are not uncommonly seen.

FURTHER READING
1. Backner CB, Walker CW, Pumell GL. Pulmonary embolism: Chest radiographic abnormalities.
J Thorac Imag. 1989;4:23-7.
2. Hansell DM, Padley SP. Continuous volume computed tomography in pulmonary embolism:
The answer or just another test? Thorax. 1996;51:1-2.
3. Patel S, Kazerooni EA. Helical CT for the evaluation of acute pulmonary embolism. Am J Roentgenol.
2005;185(1):135-49.
28 A Teaching Atlas of Case Studies in Diagnostic Imaging

CASE 14

A 32-year-old man had a routine preemploy­ment chest radiograph. On clinical examination, he was
found to be hypertensive.

B Figures 1A and B
Chest Imaging 29

Figure 1C
Figures 1A to C  (A) Chest
X-ray shows normal lung
fields; (B) The descending
aorta demonstrates a
“figure 3 sign”, and there is
bilateral rib notching, better
demonstrated on close-
up. Features are those of
coarctation of the aorta;
(C) CT angiogram of another
patient with coarctation of
the aorta. Note the pre- and
poststenotic dilatation

DISCUSSION
Coarctation of aorta is a congenital narrowing which occurs in the region of the insertion of
the ductus arteriosus, with 3 recognized types: preductal, ductal, and postductal. It is twice as
common in males as in females, and it is associated with Turner syndrome. Bilateral inferior rib
notching occurs due to the intercostal arteries in the neurovascular bundle which dilate as they
act as collaterals. The “figure 3 sign” represents the prestenotic and post-stenotic dilatation of the
descending aorta.
Balloon dilatation and stenting have become effective alternatives to surgical intervention in the
treatment of coarctation of the aorta.

FURTHER READING
1. Fawzy ME, Fathala A, et al. Twenty-two years of follow-up results of balloon angioplasty for discreet
native coarctation of the aorta in adolescents and adults. Am Heart J. 2008;156(5):910-7.
2. Rao PS. Coarctation of the aorta. Curr Cardiol Rep. 2005;7(6):425-34.
3. Silvilairat S, Cetta F, et al. Abdominal aortic pulsed wave Doppler patterns reliably reflect clinical
severity in patients with coarctation of the aorta. Congenit Heart Dis. 2008;3(6):422-30.
4. Weber HS, Cyran SE. Endovascular stenting for native coarctation of the aorta is an effective
alternative to surgical intervention in older children. Congenit Heart Dis. 2008;3(1):54-9.
30 A Teaching Atlas of Case Studies in Diagnostic Imaging

CASE 15

A 55-year-old male diabetic hypertensive complains of shortness of breath, fatigue and cough.
He has a previous history of coronary by-pass surgery.

Figure 1  PA chest
radiograph on admission
shows mild cardiomegaly
with left ventricular
configuration, congested
hilar vessels with distended
upper lobe veins, numerous
septal lines (Kerley A and
B lines), and thickened
horizontal fissure. An azygos
lobe and fissure as well as
sternal sutures are noted

Impression: Pulmonary interstitial edema due to left ventricular failure of hypertensive origin.

DISCUSSION
Septal lines, first described by Kerley in patients with pulmonary edema represent thickened
interlobular septa, the interstitial tissues that separate the secondary pulmonary lobules.
B lines are short, 1–2 cm lines perpendicular to the pleural surface and parallel to one another.
A lines are up to 4 cm long and radiate from the hila to the central parts of the lung. Septal lines are
seen in pulmonary edema, lymphangitis carcinomatosa, pneumoconiosis, sarcoidosis, etc.

FURTHER READING
1. Kerley P Radiology in heart disease. BMJ. 1933;2:594-7.
2. Paterson DI, O’Meara E, Chow BJ, et al. Recent advances in cardiac imaging for patients with heart
failure. Curr Opin Cardiol. 2011;26(2):132-43.
3. Trapnell DH. The differential diagnosis of linear shadows in chest radiographs. Radiol Clin North
Am. 1973;11:77-92.
Chest Imaging 31
CASE 16

A 10-year-old boy presented to the accident and emergency department after a blunt injury to
the chest in a motor car accident. He was in respiratory distress but hemodynamically stable,
with decreased air entry on the left side and vague epigastric discomfort. A plain chest radiograph
showed a large left-sided tension hydropneumothorax with a mediastinal shift to the right. No rib
injury was demonstrated. A left chest tube was inserted. A repeat chest radiograph 3 days later
showed mottled opacities in the left hemithorax, and a barium meal confirmed an intrathoracic
stomach.

Figures 1A and B B
32 A Teaching Atlas of Case Studies in Diagnostic Imaging

Figure 1C
Figures 1A to C  (A) Erect chest
radiograph about 6 hours after
the chest trauma. A giant air-fluid
level is present at the left base
with the mediastinum shifted to
the right. Note absence of the
left hemidiaphragm; (B) Supine
chest radiograph 3 days later
showed mottled opacities in the
left hemithorax that appear to
have intra-abdominal extension;
(C) Chest radiograph after barium
meal confirmed gastric herniation
into the left hemithorax. Note the
gastric rugae outlined by barium
(Courtesy: Dr Adekunle Abdulkadir)

Diagnosis: Diaphragmatic rupture with gastric herniation.

DISCUSSION
In post-traumatic diaphragmatic rupture, the left hemidiaphragm is affected three times
more frequently than the right, possibly due to a buffering effect of the liver on the right. Wide
diaphragmatic tears permit intrathoracic her­niation of abdominal viscera, which if extensive can
lead to disturbance of cardiorespiratory function due to compression.
Diaphragmatic rupture may be asympto­matic for months or years following trauma. Therefore,
it remains a diagnostic challenge despite the use of a variety of imaging options.
Chest radiographs are recommended for all patients after major trauma. Its sensitivity in depicting
diaphragmatic rupture is about 46% for left-sided ruptures and 17% for right-sided ruptures.
The fluoroscopic demonstration of absent or decreased diaphragmatic motion is suggestive of
diaphragmatic injury. The diagnosis of herniated viscera can be confirmed by barium studies.
Ultrasonography makes a limited contri­bution, but may depict large defects with her­niation
of intra-abdominal viscera. Peristaltic bowel loops may be identified as passing upward into the
thorax, although intraluminal bowel gas may obscure the diaphragm.
CT is the imaging modality of choice in the evaluation of severe blunt thoracoabdominal
trauma. The CT signs of diaphragmatic rupture include discontinuity of the diaphragm, visceral
herniation, and a waist-like constriction (collar sign) produced by diaphragmatic compression
Chest Imaging 33

of herniated organs. On CT, congenital posterolateral defect (Bochdalek hernia), may mimic
diaphragmatic rupture.
MRI with breath-hold acquisition permits good visualization of diaphragmatic abnor­malities,
however, this technique is difficult to perform in the patient involved in polytrauma.

FURTHER READING
1. Shackleton KL, Stewart ET, Taylor AJ. Traumatic diaphragmatic injuries: Spectrum of radiographic
findings. Radiographics. 1998;18(1):49-59.
2. Van Hise ML, Primack SL, Israel RS, et al. CT in blunt chest trauma: Indications and limitations.
Radiographics. 1998;18:1071-84.
3. Shanmuganathan K, Killeen K, Mirvis SE, et al. Imaging of diaphragmatic injuries. J Thorac Imag.
2000;15:104-11.
34 A Teaching Atlas of Case Studies in Diagnostic Imaging

CASE 17

A 19-year-old man was brought into the emergency room with chest injuries sustained in a high
velocity motor vehicle accident. The right chest showed abrasions, with a suspicion of underlying
rib injury. CT done showed pnuemothorax.
A thoracostomy tube was inserted, and the patient made a full recovery, with only minimal
residual pulmonary scarring and pleural thickening.

Figure 1  Axial CT with lung


window shows a large right
tension pneumothorax,
with several lacerations,
contusion and atelectasis of
the right lung

DISCUSSION
Pulmonary laceration may result from penetrating chest trauma, from shear stress, or from
penetrating rib fractures. Fortunately, lacerations well-managed with chest tube insertion, as in this
patient, recover with minimal sequelae. Possible complications include bronchopleural fistula and
lung abscess.

FURTHER READING
1. Gavelli G, Canini R, Bertaccini P. Traumatic injuries: Imaging of thoracic injuries. Eur Radiol.
2002;12(6):1273-94.
2. Nishiumi N, Imakuchi S, et al. Diagnosis and treatment of deep pulmonary laceration with
intrathoracic hemorrhage from blunt trauma. Ann Thorac Surg. 2010;896:232-8.
3. Omert L, Yeany WW, Protetch J. Efficiency of thoracic computerized tomography in blunt chest
trauma. Am Surg. 2001;67(7):660-4.
4. Plurad D, Green D, Demetriades D, Rhee P. The increasing use of chest computed tomo­graphy for
trauma: Is it being overutilized? J Trauma. 2007;62(3):631-5.
Chest Imaging 35
CASE 18

A 29-year-old man had a screening chest X-ray due to a residency visa processing requirement.
He was symptom-free and physical examination was normal.

Figure 1  The frontal chest


radiograph shows a large,
well-defined soft tissue
mass in the right lower
mediastinum, producing a
double contour to the right
cardiac border and causing
widening of the carina,
suggestive of an enlarged
left atrium

Figure 2  A barium swallow


fails to show evidence of
esophageal compression
expected in left atrial
enlargement
36 A Teaching Atlas of Case Studies in Diagnostic Imaging

Figures 3A and B  CT
images confirm a large
broncho­genic cyst in the
posterior mediastinum,
B with splaying of the carina
Chest Imaging 37

DISCUSSION
Bronchogenic cysts are the most common congenital intrathoracic cysts. Most occur in the middle
mediastinum, but some are seen in the posterior mediastinum. Most occur near the carina and may
coexist with bronchial atresia and pericardial defects. Intrapulmonary cysts may be large and show
air-fluid levels. MRI shows the cyst well in additional planes.

FURTHER READING
1. Erasmus JJ, McAdams HP, Donelly LF, Spritzer CE. MR imaging of mediastinal masses. Magn Reson
Imaging Clin North Am. 2000;8:59-89.
2. McAdams HP, Kirejczyk WM, Rosadode-Christenson ML, Matsumoto S. Broncho­genic cysts:
Imaging features with clinical and histopathologic correlation. Radiology. 2000;217:441-6.
3. Nokata H, Nakayama C, Kimoto T, et al. Computed tomography of mediastinal bronchogenic cysts.
J Comput Assist Tomogr. 1982;6:733-8.
38 A Teaching Atlas of Case Studies in Diagnostic Imaging

CASE 19

A morbidly obese 57-year-old female presented with severe shortness of breath, lethargy and
cyanosis. She had pedal edema and basal crepitations. This prompted a suspicion of cardiac failure.

Figure 1  A penetrated PA
chest radiograph shows
marked widening of the
mediastinum. The aorta is
identified, but the cardiac
borders and costophrenic
angles are obliterated. The
trachea is relatively central
and is not compressed. The
pulmonary vascularity is
within normal

Figure 2A
Chest Imaging 39

Figures 2B and C
Figures 2A to C  CT images
show normal cardiac and
vascular structures and
excessive fat deposition in
the mediastinum C
40 A Teaching Atlas of Case Studies in Diagnostic Imaging

Diagnosis: Mediastinal lipomatosis.

DISCUSSION
Mediastinal lipomatosis results in mediastinal widening when associated with generalized obesity,
as in this patient. It may, however, be seen in patients on steroid therapy and with Cushing’s disease
in which unencapsulated fat may be deposited in other sites. In mediastinal lipomatosis, there is
usually no mass effect on the trachea and other mediastinal structures. The differential diagnosis
includes neoplasms of fat tissue (lipoma, liposarcoma, lipoblastoma etc.), and Madelung’s disease
in which multiple masses of benign fatty tissue proliferate at various sites including the
mediastinum.

FURTHER READING
1. Gaerte SC, Meyer CA, Winer-Muram HT, et al. Fat-containing lesions of the chest. Radiographics.
2002;22:61-78.
2. Koerner HJ, Sam DI. Mediastinal lipomatosis secondary to steroid therapy. Am J Roentgenol.
1966;98:461-4.
Chest Imaging 41
CASE 20

A 45-year-old man presented with chronic productive cough and occasional hemoptysis.

Figure 1  Chest X-ray film


shows hyperinflated lungs
with multiple cystic areas in
the bases, best appreciated
over the cardiac shadow.
The cardiac size and shape
are normal

Figure 2A
42 A Teaching Atlas of Case Studies in Diagnostic Imaging

Figures 2B and C
Figures 2A to C  Axial,
coronal, and sagittal
reformatted CT images
showing multiple cystic
bronchiectasis, most severe
C in the lower lobes
Chest Imaging 43

DISCUSSION
Bronchiectasis is a pathological dilatation of medium sized bronchi. Its causes may be grouped as:
•• Congenital: Congenital cystic bronchiectasis, associated with cystic fibrosis
•• Postinfective: Staph. aureus, Klebsiella, Aspergillosis, TB
•• Obstructive: Foreign bodies, malignancy.
Three types of bronchiectasis are described: cylindrical, cystic (or saccular), and varicose.
In many cases, bronchiectasis may not be recognized on regular chest radiographs. High
resolution CT is the imaging modality of choice in defining the location and extent of the disease.

FURTHER READING
1. Barker AF. Bronchiectasis. N Engl J Med. 2002;346(18):1383-93.
2. Cantin L, Bankier AA, Eisenberg RL. Bronchiectasis. Am J Roentgenol. 2009;193(3):158-71.
3. Collins J, Stern EJ. Chest Radiology: The essentials. Lippincott Williams & Wilkins.
2007;ISBN:0781763142.
44 A Teaching Atlas of Case Studies in Diagnostic Imaging

CASE 21

A 53-year-old man, known smoker, presented with cough, fever, and left chest pain of three weeks
duration. After investigation, he was placed on antibiotics and had a good initial response, but
discharged himself against medical advice and was lost to follow-up.

Figure 1  Chest radiograph


shows a cavitating lesion
overlying the left cardiac
margin, with an air-fluid
level. It is surrounded
by patchy pulmonary
infiltrates. The cardiac size
and shape are normal.
No associated mediastinal
lymphadenopathy

Figure 2A
Chest Imaging 45

Figures 2B and C
Figures 2A to C  Axial CT
chest with sagittal and
coronal reformatted images.
The cavity shows thick
and irregular walls, with
surrounding infiltrates.
Features are those of a lung
abscess, but a cavitating
bronchogenic carcinoma
is a differential C

DISCUSSION
Causes of pulmonary cavitation are varied and may be classified as follows:
•• Infective: Pyogenic abscess, TB
•• Parasitic: Hydatid disease, strongyloidiasis, Aspergillosis
46 A Teaching Atlas of Case Studies in Diagnostic Imaging

•• Neoplastic: Bronchogenic carcinoma (Ca) (especially squamous cell Ca)


•• Miscellaneous: Pulmonary infarction, sarcoidosis, traumatic pulmonary laceration.
This case was diagnosed as a lung abscess, but the irregular outline of the inner cavity wall is more
typical of neoplasm, although this is not invariable. Also, thin-walled cavities may occasionally be
present in bronchogenic carcinoma from severe necrosis.
CT is particularly sensitive in defining any mediastinal lymph nodes or pulmonary nodules
elsewhere, both of which were absent in this patient.
Treatment of lung abscess is by prolonged use of antibiotics, percutaneous catheter drainage,
lobectomy or pneumonectomy.

FURTHER READING
1. Hern F, Ernst A, Becker HD. Endoscopic drainage of lung abscesses. Chest. 2005;147(4):1378-81.
2. Kunst H, Mack D. Kon OM, et al. Parasitic infections of the lung: A guide for the respiratory physician.
Thorax. 2011;66:528-36.
3. Patz Jr EF. Imaging bronchogenic carcinoma. Chest. 2001;117(4):905-55.
4. Podbilski FJ, Rodriguez HE, Wiesman IM, et al. Pulmonary parenchyma abscess: VATS approach to
diagnosis and treatment. Asian Cardiovas Thorac Ann. 2001;9:339-41.
Chest Imaging 47
CASE 22

An 8-year-old boy presented with a one week history of cough and dyspnea. He denied any history
of foreign body inhalation.
The tip of a ballpoint pen was removed at bronchoscopy, and the patient made full recovery.

Figure 1  Chest radiograph


shows an opaque
left hemithorax with
mediastinal shift to the
left and compensatory
emphysema of the
right lung

Figure 2A
48 A Teaching Atlas of Case Studies in Diagnostic Imaging

Figure 2B
Figures 2A and B  Axial
and coronal reformatted
CT chest showing a foreign
body (arrows) occluding the
left main stem bronchus,
producing total collapse
of the left lung with an air-
bronchogram
Chest Imaging 49
CASE 23

A 6-year-old boy was brought with complaints of a one day history of severe dyspnea. He denied a
history of foreign body inhalation.

Figures 1A and B  Frontal


and lateral chest radio­
graphs showing a metallic
screw lodged in the right
lower lobe bronchus. The
screw was successfully
retrieved at brochoscopy
with excellent result B
50 A Teaching Atlas of Case Studies in Diagnostic Imaging

DISCUSSION
Foreign body inhalation is far more common in children because of their tendency to put objects in
their mouth. The objects may be radiopaque or radiolucent (e.g. peanuts and plastic). Patients may
present acutely with dyspnea, wheezing and choking, but hours or weeks later, they may become
asymptomatic. Some patients present later still with complications, such as atelectasis, pneumonia,
and abscess.
In the older child who can cooperate, chest X-ray exposures taken at inspiration and expiration
may show poor air entry and exit from the affected side. As in Case 22, CT is invaluable in the
diagnosis of airway foreign bodies. CT virtual bronchoscopy has also been found useful. With
specific reference to inhaled peanuts, MRI has been used to advantage in locating them because of
their fat content.

FURTHER READING
1. Adaletli I, Kurugoglu S, Ulus S, et al. Utilization of low-dose multidetector CT and virtual
bronchoscopy in children with suspected foreign body aspiration. Pediatr Radiol. 2009;37:33-40.
2. Cohen S, Avital A, Godfrey S, et al. Suspected foreign body inhalation in children: What are the
implications of bronchoscopy? J Pediatr. 2009;155(2):276-80.
3. Imaizumi H, Kaneko M. Nara S, et al. Definitive diagnosis and location of peanuts in the airways
using magnetic resonance imaging techniques. Ann Emerg Med. 1994;23(6):1379-82.
4. Odelowo EOO, Komolafe OF. Diagnosis, management and complications of oesoph­ageal and
airway foreign bodies. Inter­national Surg. 1990;75(3):148-54.
Chest Imaging 51
CASE 24

A 59-year-old woman presented with cough, hemoptysis, chest pain and weight loss.

Figures 1A and B B
52 A Teaching Atlas of Case Studies in Diagnostic Imaging

Figure 1C
Figures 1A to C  CT chest
with coronal reformatted
and axial images showing
a large mass in the right
upper lobe with irregular,
lobulated outlines. There is a
pleural tail extending to the
lateral chest wall and to the
mediastinum. Wide-spread
nodular opacities are noted
in both lungs. Features
are highly suggestive of
bronchogenic carcinoma
with extensive metastases

DISCUSSION
Bronchogenic carcinoma is the most common cancer in men in the United States and the
6th most common in women. It accounts for 20% of all cancer deaths worldwide. Cigarrete smoking
is implicated in 90% of cases.
The most common sites of metastases include hilar/mediastinal lymph nodes, brain, bone, liver,
adrenals and the contralateral lung. The pattern of distribution of the pulmonary nodules in this
patient probably represents hematogenous spread.

FURTHER READING
1. Fauci AS, Braunwald E, Kasper DL, et al. Harrison’s Principles of Internal Medicine. McGraw-Hill
Professional. 2008; ISBN:0071466339.
2. Patz EF. Imaging Bronchogenic Carcinoma. Chest. 2000;117(4):905-55.
Chest Imaging 53
CASE 25

An 8-year-old female complained of cough, shortness of breath, and poor appetite. She was mildly
febrile and had hepatosplenomegaly and cervical lymphadenopathy.

Figure 1  Frontal chest X-ray


shows enormous bilateral
mediastinal soft tissue
masses. The trachea
remains central

Figure 2A
54 A Teaching Atlas of Case Studies in Diagnostic Imaging

Figure 2B
Figures 2A and B  The
contrast-enhanced CT
chest images show a large
anterior mediastinal mass
of homogenous density
extending to the middle
and posterior mediastinum
and encasing the major
vessels, trachea and
main bronchi. There are
no pulmonary or pleural
lesions. Other findings
included hepatomegaly,
massive splenomegaly and
peritoneal effusion

Diagnosis: Lymphoma or acute lymphoblastic leukemia. Hematological investigations con­firmed


the latter.

DISCUSSION
Lymph node enlargement is seen in some cases of leukemia, the pattern being the same as with
lymphoma. The lymph node enlargement in both may resolve remarkably rapidly with therapy.
T-cell leukemias may show massive mediastinal lymphadenopathy that responds rapidly to
chemotherapy or radiation treatment, as occurred in this case.

FURTHER READING
1. Felson B. The lymphomas and leukaemias–Part 1. Semin Roentgenol. 1980;15(3).
2. Felson B. The lymphomas and leukaemias–Part 2. Semin Roentgenol. 1980;15(3).
3. Lee KS, Kim Y, Primack SL. Imaging of Pulmonary lymphoma. AJR Am J Roentgenol. 1997;168(2):
339-45.
Chest Imaging 55
CASE 26

A 51-year-old man complained of left-sided chest pain radiating to the infrascapular region for two
months. Physical examination was normal.

Figure 1  A PA chest
X-ray revealed an ovoid
well-defined and slightly
lobulated opacity in the
left midzone near the
hilum. There is associated
osteolytic destruction of the
posterior ends of the left 7th
and 8th ribs in relation
to the mass

Figure 2  Axial CT images


identified a well-defined
posterior mediastinal mass
with multiple calcific foci,
destruction of the adjacent
rib and vertebra with
extension into the spinal
canal, consistent with a
neurogenic tumor
56 A Teaching Atlas of Case Studies in Diagnostic Imaging

Diagnosis: Biopsy confirmed a schwanomma.

DISCUSSION
Most neurogenic tumors (neurofibromas, schwanommas, ganglioneuromas, ganglioneuroblas­
tomas, etc.) manifest as well-defined masses with smooth or lobulated outlines. Most are spherical
and located in the posterior mediastinum. Distinction between benign and malignant tumors
may be difficult based on imaging features. Calcification is seen in all types. They may be homogenous
or of mixed attenuation on plain CT and may show cystic degeneration. Enhancement patterns are
variable—homogenous or heterogenous, rim or target patterns. MR signal intensity patterns are
variable, but spinal and intraspinal involvements are better demonstrated.

FURTHER READING
1. Lee JY, Lee KS, Han J, et al. Spectrum of neurogenic tumors in the thorax: CT and pathologic
findings. J Comput Assist Tomogr. 1999;23:399-406.
2. Pilavaki M, Chourmouze D, Kizirdou A, et al. Imaging of peripheral nerve sheath tumors with
pathologic correlation. Pictorial review. Eur J Radiol. 2004;52:229-39.
3. Ribet ME, Cordot GR. Neurogenic tumors of the thorax. Ann Thorac Surg. 1999;58:1091-5.
Chest Imaging 57
CASE 27

A 38-year-old man had a routine pre-employment chest X-ray.

Figure 1  Chest radiograph


showing opacity of the left
apex. The rest of the lung
fields and the cardiac size
and shape are normal

Figure 2  Axial CT chest


shows a left apical mass of
low density, similar to the
subcutaneous fat
58 A Teaching Atlas of Case Studies in Diagnostic Imaging

Figure 3  Left parasagittal


reformatted image showing
a superior mediastinal
mass extending from the
thoracic inlet, with a smooth
inferior margin. Attenuation
coefficient was -100
Hounsfield units, consistent
with fat

Diagnosis: Superior mediastinal lipoma.

DISCUSSION
Intrathoracic lipomas are rare and usually discovered incidentally during routine chest radiographs.
They are usually symptomless, but when they occur, symptoms depend on their size and location.
Giant lipomas may cause dyspnea, whereas bronchial lipomas may produce bronchial occlusion
or hemoptysis.
The differential diagnosis of superior mediastinal lipoma includes thymolipoma, harmatoma,
and liposarcoma. In the manage­ment of intrathoracic lipomas, surgical re­section is advocated,
as liposarcoma needs to be excluded.

FURTHER READING
1. Gaerte SC, Meyer CA, Winer-Muram HT, et al. Fat-containing lesions of the chest. Radiographics.
2002;22:S61-S78.
2. Karlo CA, Stolzmann P, Fravenfelder T, et al. Computed tomography imaging of subpleural lipoma
in two men: Two case reports. J Med Case Reports. 2010;4:380.
3. Kransdorf MJ, Bancroft LW, Peterson JJ, et al. Imaging of fatty tumors: Distinction of lipoma and
well-differentiated liposarcoma. Radiology. 2002;224:99-104.
4. Sarukai H, Kaji M, Yamazaki K, Suemasu K. Intrathoracic lipomas: Their clinic-pathological
behaviors are not as straightforward as expected. Ann Thorac Surg. 2008;86:261-5.
Chest Imaging 59
CASE 28

A 53-year-old man presented to the emergency room with a history of chronic pain in both shoulders.
X-rays of both shoulders were done. Incidental finding of elevated left dome of diaphragm was seen
and a chest radiograph was done. Detailed clinical history revealed no significant chest symptoms,
and there was no history of previous trauma or chest surgery.
Fluoroscopy displayed poor excursion of left dome of diaphragm with respiration.

Figure 1  Posteroanterior chest


radiograph shows marked
elevation of an intact left dome of
diaphragm with fundal gas and
colonic gas shadows high in
the left chest. There is also
marked mediastinal and tracheal
shift to the right

Figure 2A
60 A Teaching Atlas of Case Studies in Diagnostic Imaging

Figure 2B
Figures 2A and B  (A) Axial CT
chest mediastinal window and
lung window; (B) At mid chest level
show stomach and bowel loops in
the left chest (Courtesy: Dr Sujatha
Rajkumar)

Diagnosis: Congenital unilateral eventration of the diaphragm in an adult.

DISCUSSION
Unilateral elevation of hemidiaphragm may be secondary to adjacent pulmonary, pleural or
subphrenic pathology, phrenic nerve palsy or trauma.
Congenital eventration of the diaphragm is due to a thin, hypoplastic but intact diaphragm
with displacement of abdominal contents into thorax. It has a right-sided predominance of 5:1,
being anteromedial on the right and total involvement on the left. Left-sided eventration is
indistinguishable from diaphragmatic paralysis on radiographs, which may be due to surgery,
neoplastic infiltration or idiopathic phrenic nerve dysfunction due to viral neuritis.
Chronic loss of lung volume due to collapse or lobectomy may cause elevated diaphragmatic
dome. Splenomegaly, gaseous distension of stomach/splenic flexure of colon, subpulmonic pleural
effusion or large tumor adjacent to the dome are other causes of unilateral elevation of the dome
of diaphragm.

FURTHER READING
1. Brant WE, Helms CA. Fundamentals of Diagnostic Radiology, 3rd edition, Lippincott Williams &
Wilkins, Philadelphia; 2007. pp. 547-8.
2. Dahnert W. Radiology Review Manual, 7th edition, Chest Disorders, Lippincott Williams & Wilkins,
Philadelphia; 2011. p. 499.
3. Mantoo SK, Mak K. Congenital diaphrag­matic eventration in an adult: A diagnostic dilemma.
Singapore Med J. 2007;48(5):136-7.
Chest Imaging 61
CASE 29

A 23-year-old male was referred for a chest X-ray because of increasing shortness of breath and
fatigue.
Physical examination and echocardiography confirmed the diagnosis of atrial septal defect
(ASD).

Figure 1  Posteroanterior
chest radiograph shows
mild cardiomegaly, enlarged
right atrium and large
central and peripheral
pulmonary arteries.
The upper zone vessels
are prominent. There are
no changes of
pulmonary edema

DISCUSSION
The types of ASD are ostium primum, ostium secundum, sinus venosus and anomalous pulmonary
veins. ASDs are the most common congenital heart defects in adults. Ostium secondum defects
form about 80% of all ASDs and are located in the fossa ovalis. The sinus venosus defect is rare and is
often associated with partial anomalous pulmonary venous drainage. Evaluation of ASDs involves
establishment of the type and location of the defect, quantification of the shunt, assessment of right
ventricular function and pulmonary venous anatomy and detection of any intra-atrial thrombus.
Management is with transcatheter ASD mechanical closure or by open heart surgery.

FURTHER READING
1. Gatzoulis MA, Webb GA, Daudency P. Diagnosis and management of adult con­genital heart disease.
Churchill Livingstone, London; 2003.
2. Taylor AM, Reck C. Imaging of congenital heart disease. In: Grainger and Allison’s Diagnostic
Radiology—A Textbook of Medical Imaging. Adam A, Dixon AK (Eds), 5th edition, Churchill
Livingstone, London; 2008. pp. 455-66.
62 A Teaching Atlas of Case Studies in Diagnostic Imaging

CASE 30

A 58-year-old male presents with cough and progressive shortness of breath.

B Figures 1A and B
Chest Imaging 63

Figure 1C
Figures 1A to C  HRCT
images show that
the abnormalities are
predominantly subpleural
and bibasal, and comprise
a reticular pattern within
which there are areas
of honey-combing and
traction bronchiectasis. The
pattern is that of idiopathic
pulmonary fibrosis (IPF)/
usual interstitial pneumonia
(UIP). Areas of ground glass
opacity are shown

DISCUSSION
Usual interstitial pneumonia is the most common histopathological pattern in patients presenting
as cryptogenic fibrosing alveolitis/IPF. Other causes of UIP-type pattern include chronic
hypersensitivity pneumonitis, connec­tive tissue diseases and certain drugs, e.g. nitrofurantoin.
The chest radiograph shows bilateral asymmetric peripheral reticular opacities, most marked in
the lung bases, with volume loss. As the disease advances, it creeps around the periphery of the
lungs to involve the upper lobes. Complications of UIP include Pneumocystis jiroveci pneumonia,
carcinoma and pulmonary TB.

FURTHER READING
1. Johkoh T, Muller NI, Cartier Y, et al. Idiopathic interstitial pneumonia: Diagnostic accuracy of thin
section CT in 129 patients. Radiology. 1999;211:555-60.
2. Muller NI. Clinical value of high resolution CT in chronic diffuse lung disease. AJR Am J Roentgenol.
1991;157:1163-70.
3. Schaefer-Prokop C, Prokop M, Fleischmann D, et al High-resolution CT of diffuse interstitial lung
disease; key findings in common disorders. Eur Radiol. 2001;11:373-92.
64 A Teaching Atlas of Case Studies in Diagnostic Imaging

CASE 31

A 40-year-old man presents with mild shortness of breath. A chest radiograph shows bilateral
parenchymal nodular opacities.

B Figures 1A and B
Chest Imaging 65

Figures 1A to C  HRCT
images show extensive
nodular opacities which
have become confluent
and are predominantly
subpleural and along
neurovascular bundles,
with cystic areas and
interlobular septa
thickening. Nodules are
shown in the fissures giving
them a beaded appearance.
The findings are consistent
with stage III sarcoidosis.
The fissural nodules are a
useful clue to the diagnois.
This HRCT appearance is
similar to the crazy-paving
seen in pulmonary alveolar
proteinosis Figure 1C

DISCUSSION
Sarcoidosis is a multisystem granulomatous disorder of young adults. The hilar and
mediastinal lymph nodes and the lungs are affected much more than any other organ. The
skin, eyes, spleen, CNS, parotids and bones may be involved. Lymphadenopathy occurs in
70–80% of cases. Remaining parenchymal changes which may lead to fibrosis account for most of
the morbidity and mortality.

FURTHER READING
1. Brauner MW, Grenier P, Mompoint D, et al. Pulmonary sarcoidosis: Evaluation with high resolution
CT. Radiology. 1989;172:467-71.
2. Criado E, Sanchez M, Ramirez J, et al. Pulmonary sarcoidosis: Typical and Atypical manifestations
at high resolution CT with pathologic correlation. Radiographics. 2010;30:1567-86.
3. Hamper UM, Fishman EK, Khouri NF, et al. Typical and atypical CT manifestations of pulmonary
sarcoidosis. J Comput Asst Tomogr. 1986;10:928-36.
4. Kinks DR, McCormick VD, Greenspan RH. Pulmonary sarcoidosis: Roentgenologic analysis of
150 patients. AJR Am J Roentgenol. 1973;117:777-85.
66 A Teaching Atlas of Case Studies in Diagnostic Imaging

CASE 32

A 22-year-old man presented to emergency with one episode of hemoptysis and was sent to the
radiology department for chest X-ray to rule out tuberculosis.

Figure 1  Erect frontal chest


radiograph shows three large
sharply-defined opacities
with lobulated outline in right
hilar and lower zone and left
paracardiac regions. Coiled
tubular shadows are seen
within the masses and from
the mass to the hilum. No
calcifications are seen

Figure 2A
Chest Imaging 67

Figure 2B
Figures 2A and B  Plain axial
and sagittal reformatted,
contrast-enhanced CT chest
images show the well-
circumscribed, lobulated
noncalcified masses
connected by blood vessels,
the feeding and draining
vessels being clearly seen.
The features are those of
pulmonary arteriovenous
malformations (PAVM)
(Courtesy: Dr Sujatha
Rajkumar)

DISCUSSION
Pulmonary arteriovenous malformations are rare pulmonary vascular anomalies with abnor­mal
communication between the pulmonary artery and the pulmonary vein (95%) or systemic artery
and pulmonary vein (5%). PAVMs are usually congenital in origin; however, they may be acquired
in a variety of conditions, such as hepatic cirrhosis, schistosomiasis, mitral stenosis, trauma,
actinomycosis, metastatic thyroid carcinoma and TB (Rasmussen’s Aneurysm).
Although most patients are asymptomatic, PAVMs can cause dyspnea from right-to-left shunt.
Because of paradoxical emboli, various central nervous system complications have been described
including stroke and brain abscess. There is a strong association between PAVM and hereditary
hemorrhagic telangiectasia and screening of first degree relatives is recommended.
Chest radiography and contrast-enhanced computed tomography are essential initial diagnostic
tools, but pulmonary angiography is the gold standard. Although pulmonary angiography is for
confirmation of a PAVM, angiography is required only when further intervention is planned.
Therapeutic options include angiographic embolization with metal coil or balloon occlusion
and surgical excision.

FURTHER READING
1. Dahnert W. Radiology review manual, 7th edition. Chest Disorders, Lippincott Williams and
Wilkins, New Delhi; 2011. p. 531.
2. Hansell DM, Lynch DA, McAdams HP, et al. Imaging diseases of the chest, 5th edition, Chapter 16.
Congenital disorders of the lung and airways. Elsevier Ltd.; 2009. pp. 1079-86.
3. Khurshid I, Downie GH, Pulmonary arte­riovenous malformation. Postgrad Med J. 2002;78:191-7.
68 A Teaching Atlas of Case Studies in Diagnostic Imaging

CASE 33

An asymptomatic 37-year-old man had a chest radiograph for pre-employment assessment.

Figure 1  PA chest
radiograph shows multiple
cystic shadows of varying
sizes occupying the right
hemithorax. The trachea and
heart are markedly shifted
to the right, and the left
lung shows compensatory
emphysema.
Note the flattened left
hemidiaphragm.
Impression: The features are
those of cystic adenomatoid
malformation of the right
lung presenting in an adult

Figure 2  Coronal reformatted


CT image showing the
multiple deformed cysts,
some communicating with
each other
Chest Imaging 69

Figures 3A and B  Axial


and right parasagittal
reformatted images show
the anomalous right lung
posterior to the heart and
the emphysematous left
lung crossing the mid-line
to lie anterior to the heart B
70 A Teaching Atlas of Case Studies in Diagnostic Imaging

Figure 5  Mediastinal
window shows the heart
displaced against the right
chest wall. Note the right
paravertebral location of
the descending aorta

DISCUSSION
Cystic adenomatoid malformation (CAM) is believed to result from a developmental error at
or before the 7th week of intrauterine life, in which there is an adenomatous proliferation of
the terminal bronchioles and cystic distortion of the lung parenchyma. Most cases of CAM are
diagnosed prenatally by ultrasonography or by the age of 5 years, with affected children presenting
with neonatal respiratory distress or recurrent chest infection. Presentation in the adult is extremely
rare.
CAM is classified into 3 types: Type 1 with large cysts (2–10 cm), Type 2 with small cysts (0.5–2
cm), and Type 3 with microscopic cysts (macroscopically solid).
Surgical resection is the treatment of choice for CAM, to avert complications, such as hemorrhage,
recurrent infection and malignancy.

FURTHER READING
1. Avitabile AM, Greco MA, Hulnick DH, Feiner HD. Congenital cystic adenomatoid malformation of
the lung in adults. Am J Surg Pathol. 1984;8(3):193-202.
2. Davies AR, Bapat V, Treasure T. Adult presen­tation of congenital cystic adenomatoid mal­formation:
Successful surgical management. J Thorac Cardiovasc Surg. 2006;132:1493-4.
3. Han YM, Lee DK, Lee SY, et al. Adult pre­sentation of congenital cystic adenomatoid malformation
of the lung; A Case Report. J Korean Med Sci. 1994;9:86-91.
4. Lujan M, Bosque M, Mirapeix RM, et al. Late-onset congenital cystic adenomatoid mal­formation of
the lung: Embryology, Clinical symptomatology, diagnostic procedures, therapeutic approach and
clinical follow-up. Respiration. 2002;69(2):148-54.
2 Musculoskeletal
System Imaging

CASE 1

A neonate died of respiratory failure within 10 minutes of birth.

Figure 1  Babygram
shows short limb bones
with telephone handle
deformities, narrowing of
vertebral column and the
interpedicular distances
from above downward,
and the near square-
shaped iliac blade
(Courtesy: Dr Adekunle
Abdulkadir)

Diagnosis: Thanatophoric dysplasia.

DISCUSSION
Thanatophoric dysplasia (TD) is the most common of the congenital lethal skeletal dysplasias.
It occurs sporadically in 1/64,000-100,000 live births, and its etiology is unknown. Short limbs,
small conical thorax, platyspondyly and macrocephaly are the characteristic features. The affected
fetuses usually die within the first 48 hours of life.
72 A Teaching Atlas of Case Studies in Diagnostic Imaging

Two variants have been described. Type 1 variant presents with polyhydramnios, macrocephaly,
short limbs, narrow thoracic cage and curved short femur (the typical telephone receiver
appearance). Type 2 variant presents with short limbs, narrow thoracic cage, straight short femora,
hydrocephalus, and cloverleaf skull.

FURTHER READING
1. Komolafe F, Adetoro OO. Thanatophoric dwarfism in a Nigerian child. Afr J Med Med Sci.
1986;15:109-12.
2. Miller E, Blaser S, Shannon P, Widjaja E. Brain and bone abnormalities of thanatophoric dwarfism.
Am J Roentgenol. 2009;192:48-51.
3. Pretorius DH, Rumack CM, Manco-Johnson ML, Manchester D, Meier P, Bramble J, Clewell W.
Specific skeletal dysplasias in utero: Sonographic diagnosis. Radiology. 1986;159:237-42.
4. Spirt BA, Oliphant M, Gottlieb RH, Gordon LP. Prenatal sonographic evaluation of shortlimbed
dwarfism: An algorithmic approach. Radiographics. 1990;10:217-36.
Musculoskeletal System Imaging 73
CASE 2

A 38-year-old man had a chest radiograph taken for pre-employment purposes. He was
asymptomatic.

Figures 1A and B  Heart


and lung fields are normal.
The left 5th rib is bifid,
beginning at its posterior
end. A close-up image
is shown in (B) B
74 A Teaching Atlas of Case Studies in Diagnostic Imaging

DISCUSSION
Bifid ribs usually involve the sternal ends of affected ribs. They are usually asymptomatic and are
discovered incidentally. They are estimated to occur in 0.2% of the population, and have a right-
sided predilection. When symptomatic, they may be a cause of musculoskeletal chest pain and
intercostal nerve entrapment. If the first rib is affected, they may be a cause of thoracic outlet
syndrome.
There is an association of bifid ribs with nevoid basal cell carcinoma syndrome (Gorlin–Goltz
syndrome).
This case is unusual in the far posterior separation of the bifid rib.

FURTHER READING
1. Cagli K. Thoracic outlet syndrome in an adolescent with bilateral bifid ribs. Clin Anat. 2006;19(6):
558-60.
2. Daramola JO, Komolafe OF, Ajagbe HA, Lawoyin DO. Syndrome of multiple jaw cysts, skeletal
anomalies, and basal cell nevi: Report of a case. J National Med Ass. 1980;72(3):259-62.
3. Hegde S, Shetty SR. Radiological features of familial Gorlin-Goltz syndrome. Imaging Sci Dent.
2012;42(1):55-60.
4. Song WC, Kim SH, Park DK, Koh KS. Bifid rib: Anatomical considerations in three cases. Yonsei
Med J. 2009;50(2):300-03.
Musculoskeletal System Imaging 75
CASE 3

A 13-year-old boy presented with pain and limitation of movements in both hip joints, associated
with difficulty in walking.

Figures 1A and B  AP
radiograph of the pelvis and
frog lateral views of both
hips are shown.
The radiographs reveal
bilateral slipped capital
femoral epiphyses, more
severe on the right side.
The growth plates appear
irregular and widened, and
the epiphyses are displaced
posteriorly and medially B
76 A Teaching Atlas of Case Studies in Diagnostic Imaging

DISCUSSION
Slipped capital femoral epiphysis (epiphysio­lysis) is the result of repetitive trauma to the proximal
femoral growth plate. It typically occurs in adolescence (boys more frequently than girls) aged
10 to 16, and is associated with either no or minor injury. Childhood obesity is believed to be an
etiological factor. Other factors reported include hyperthyroidism, growth hormone deficiency or
hypogonadism. Frog leg lateral views of the hip better demonstrate displacement compared with
standard AP views. The epiphysis is displaced posteriorly and medially, and the growth plate is
poorly defined, irregular and widened. Klein’s line, drawn along the superior cortex of the femoral
neck, normally cuts through a small portion (6th) of the femoral epiphysis. In epiphysiolysis,
this line does not intersect the femoral epiphysis. Bilateral hip involvement is common. It is
more common in boys of Afro-American origin. MRI is useful for confirmation in doubtful cases.
Some authors advocate the use of ultrasonography.

FURTHER READING
1. Loder RT. Unstable slipped capital femoral epiphysis. J Paediatr Orthop. 2001;27:694-9.
2. Wells D, King JD, Roe TF, Kaufmann FR. Review of slipped capital femoral epiphysis associated
with endocrine disease. J Paediatr Orthop. 1993;13:610-4.
Musculoskeletal System Imaging 77
CASE 4

A 43-year-old man presents with severe backache and stiffness.

Figures 1A and B  AP
and lateral views of the
lumbosacral spine show
complete fusion of the
sacroiliac joints, marginal
symmetric syndesmophytes
bridging the disc spaces
(bamboo spine), ossification
of the anterior and posterior
longitudinal ligaments and
of the supraspinous and
interspinous ligaments A B

Diagnosis: Ankylosing spondylitis.

DISCUSSION
Ankylosing spondylitis primarily affects the axial skeleton. It is an autoimmune disease in which 96%
of patients are HLA B27 positive. Sacroiliac joint involvement is common and is usually symmetrical.
The thoracolumbar and lumbosacral joints are also commonly involved. The peripheral skeleton is
involved in 10–20% of cases.
Radiological findings include vertebral “squaring”, erosive osteitis, discitis, marginal
syndesmophyte formation, ligamentous ossification, joint ankylosis, dorsal arachnoid diverticula
and atlantoaxial subluxation. Around 1–2% of individuals develop upper lobe pulmonary
fibrosis with the development of bullae and cavities and a tendency to mycetoma formation
78 A Teaching Atlas of Case Studies in Diagnostic Imaging

with aspergillosis. Five percent of cases develop aortitis of the ascending aorta and aortic valve
insufficiency. Spinal fractures are an important complication. Differential diagnoses include
inflammatory bowel disease, Reiter’s syndrome, psoriatic arthropathy and Sapho syndrome. MRI,
multidetector CT and nuclear medicine are valuable in the diagnosis and assessment of ankylosing
spondylitis.

FURTHER READING
1. Dale K, Vinje O. Radiography of the spine and sacroiliac joints in ankylosing spondylitis and
psoriasis. Acta Radiol Diagn. 1985;26:145-59.
2. Forrester DM. Imaging of the sacroiliac joints. Radiol Clin North Am. 1990;28:1955-72.
3. Goldberg AL, Keaton NL, Rothfus WE, Daffner RH. Ankylosing spondylitis complicated by trauma.
MR findings correlated with plain radiographs and CT. Skeletal Radiol. 1993;22:333-6.
Musculoskeletal System Imaging 79
CASE 5

A 28-year-old male presented with a painless lump on the left side of his head. A CT of the skull
was done.

Figures 1A and B  CT brain


and bone windows show
a well-defined rounded
osteolytic lesion in the left
frontal bone, with scalloped
and sclerotic margins. The
cortex is expanded and
thinned, and a pathological
fracture is shown. There is
no calcification or soft tissue
extension. The appearance
is typical of an epidermoid
inclusion cyst B
80 A Teaching Atlas of Case Studies in Diagnostic Imaging

DISCUSSION
Epidermoid inclusion cysts are composed of stratified squamous epithelium, keratin and cholesterol
crystals implanted under skin, with secondary bone erosion. They are asymptomatic, and are
seen from the second to the fourth decades. Males are more commonly involved. Epidermoids
are classically located in superficially situated bones such as calvarium, (in the frontal or parietal
bones), and in the phalanx (usually terminal tuft of the middle finger). The foot is occasionally
involved. The differential diagnosis in the finger includes glomus tumor and enchondroma; and in
the skull, infection, eosinophilic granuloma and metastasis.

FURTHER READING
1. Du Boulay GH. Principles of X-ray diagnosis of the skull, Butterworth, London; 1980.
2. Narlawar RS, Nagar A, Hira P, Raut AA. Intradiploic epidermoid cyst. J Postgrad Med. 2002;48:213.
Musculoskeletal System Imaging 81
CASE 6

A 25-year-old male presented with frontal headaches and nasal congestion.

Figures 1A and B  Occipito-


mental and lateral views of the
paranasal sinuses show a large
lobulated ivory osteoma of
the frontal sinus B

DISCUSSION
See under case 7 (Page 82)
82 A Teaching Atlas of Case Studies in Diagnostic Imaging

CASE 7

Another patient, a 44-year-old hypertensive diabetic female presented with signs suggestive of
transient ischemic attack. A CT brain showed normal brain parenchyma and ventricular system.
She was however found to have numerous osteomas in the frontal bone, which had been present
for about 20 years.

Figure 1  Surface shaded


display of the skull shows
multiple cranial vault
swellings. There was no
familial history of such
lesions, and no evidence
of polyposis

DISCUSSION
Osteomas are slow growing benign surface lesions comprising well-differentiated mature bone.
They classically occur in the frontal and ethmoidal sinuses, less commonly in the skull vault and
mandible. They rarely arise in long bones, but if large, they may mimic parosteal osteosarcoma or
melorheostosis. Large paranasal osteomas may cause compression and obstructive symptoms and
erode into the anterior cranial fossa. Gardner’s syndrome is a combination of mandibular osteomas,
colonic polyposis and cutaneous lesions, with a very high possibility of developing colon cancer.

FURTHER READING
1. Alexander AA, Patel AA, Odland R. Paranasal sinus osteomas and Gardner’s syndrome. Ann Oto
Rhinol Laryngol. 2007;116(9):658-62.
2. Earwaker J. Paranasal sinus osteomas: A review of 46 cases. Skeletal Radiology 1993;22(6):417-23.
3. Lee HJ, Shin MS, Park By, et al. Multiple osteomas in the skull vault: Case Report. J Korean Soc Plast
Reconstr Surg. 2011;38(4):512-5.
4. Ruggieri M, Pavone V, Polizzi A, et al. Familial osteoma of the cranial vault. Br J Radiol.
1998;71(842):225-8.
Musculoskeletal System Imaging 83
CASE 8

An adult male patient known with bladder extrophy.

Figure 1  AP pelvis showing


typical diastasis of
the symphysis pubis in
this condition

DISCUSSION
Bladder extrophy—epispadias complex re­presents a spectrum of anomalies. The defect occurs
in the anterior abdominal wall, pubic area, anterior wall of the bladder and dorsal aspect of the
urethra. The defect causes the bladder to be open and the mucosa to be continuous with the skin.
Radiographic features include: diastasis of the pubic symphysis (width correlates with severity
of extrophy), omphalocele confluent with extrophic bladder, cryptorchidism, inguinal hernia,
acquired ureterovesical junction obstruction in untreated cases, small bladder and reflux (after
surgical repair). Other associated findings include rectal prolapse, bifid or unicornuate uterus and
spinal anomalies. Extrophy can be diagnosed on prenatal ultrasonography.

FURTHER READING
1. Bhatnagar V. Bladder extrophy: An overview of the surgical management. J Indian Assoc Pediatr
Surg. 2011;16(3):81-7.
2. Gargollo PC, Borer JG. Contemporary outcomes in bladder extrophy. Current opinion in urology.
2007;17(4):272-80.
3. Mathews R, Gearhart JP, Bhatnagar R, Sponseller P. Staged pelvic closure of extreme pubic diastasis
in the extropy-epispadias complex. J Urol. 2006;176(5):2196-8.
4. Pinette MG, Pan YQ, Pinette SG, et al. Prenatal diagnosis of fetal bladder and cloacal extrophy
by ultrasound. A report of three cases. J Reprod Med. 1996;41(2):132-4.
84 A Teaching Atlas of Case Studies in Diagnostic Imaging

CASE 9

A 37-year-old man presented with insidious progressive, tortuous swelling of the medial aspect of
the left arm proximal to the elbow.

Figures 1A and B  T1w MRI


coronal and axial images
showing tubular masses
with flow-voids in the
subcutaneous tissues
of the arm just above
B the elbow
Musculoskeletal System Imaging 85

Figure 2  MR angiogram
showing a leash of vessels
deriving its supply from
the brachial artery and
draining into multiple
superficial veins. Features
are those of arteriovenous
malformation

DISCUSSION
Peripheral vascular malformations (AVM) may have a variable presentation, in terms of age
of onset and clinical severity. Noninvasive diagnostic modalities include Doppler ultrasound,
CT and MRI angiography. They are essential in determining the flow velocity, flow direction,
location and involvement of surrounding structures, and planning therapy. Surgical treatment
comprises extirpation of the malformation, but embolotherapy and percutaneous sclerotherapy
have become preferable. AVMs have a high recurrence rate.

FURTHER READING
1. Hyodoh H, Hori M, Akiba H, et al. Peripheral vascular malformations: Imaging, Treatment
Approaches and Therapeutic Issues. Radiographics. 2005;25:159-71.
2. Upton J, Coombs CJ, Mulliken JB, et al. Vascular malformations of the upper limb: A review of
270 patients. J Hand Surg. 1999;24(5):1019-35.
86 A Teaching Atlas of Case Studies in Diagnostic Imaging

CASE 10

A 40-year-old man presented with a 2-year history of a cystic lesion in the right popliteal fossa. It was
non-tender, and there was no bruit over it. Ultrasonography showed a nonvascular cystic mass.

B Figures 1A and B
Musculoskeletal System Imaging 87

Figures 1C and D
Figures 1A to D  MRI
images showing T1w
sagittal, Sagittal PD,
Coronal T2w, and Axial
T2w, respectively, showing
a large cyst with smooth
outlines on the lateral
aspect of the popliteal
fossa. There is no joint
effusion, and no evidence
of communication with the
joint cavity. A hydatid cyst
was considered likely D
88 A Teaching Atlas of Case Studies in Diagnostic Imaging

DISCUSSION
Popliteal synovial cysts (Baker’s cysts) are probably the most common cyst in the popliteal fossa.
They are located medially between the medial head of the gastrocnemius and the semimem­
branosus muscles. Other popliteal fossa lesions that need to be differentiated from Baker’s cysts on
imaging include: sebaceous cysts, hydatid disease and synovial sarcoma.

FURTHER READING
1. Battyany I, Andrea L, Nagy KK. Subcutaneous hydatid cyst in the popliteal fossa at the site of
a previous wasp sting. Diagn Interv Radiol. 2011;17(2):163-5.
2. Bharati S, Pal M. Primary Hydatid cyst in Gastrocnemius muscle. Nig J Surg. 2012;18(1):19-21.
3. Bixby SD, Hettmer S, Taylor GA, Voss SD. Synovial sarcoma in children: Imaging features and
common benign mimics. AJR Am J Roentgenol. 2010;195(4):1026-32.
4. Toussaint SP, McCabe S. Baker’s cyst Imaging. Int J Emerg Med. 2010;3(4):469-70.
Musculoskeletal System Imaging 89
CASE 11

A 20-year-old man presented with a painless right upper arm swelling which was discovered
incidentally during a medical examination.
A well capsulated exostosis was excised.

Figures 1A and B  Right


humerus showing a sessile
bony mass in its proximal
epiphysis, with a partially
calcified cartilage cap.
The mass is producing
deformity of the overlying
soft tissues. An MRI
(not included) showed a
cartilage cap <5 mm thick.
Features are those of a
solitary osteochondroma B
90 A Teaching Atlas of Case Studies in Diagnostic Imaging

DISCUSSION
Osteochondromas are composed of cortical bone with an overlying hyaline cartilage. They constitute
20–50% of benign bone lesions. They may be solitary or multiple. They are of clinical importance
because they may produce deformities, cause impingement of adjacent neurovascular structures,
predispose to fractures and rarely (1%) undergo malignant transformation. Sarcomatous change
is suspected if the thickness of the cartilage cap, best demonstrated by MRI, exceeds 1.5 cm, and
PET/CT is invaluable in evaluating malignant transformation.

FURTHER READING
1. Murphey MD, Choi JJ, Kransdorf MJ, et al. Imaging of osteochondroma: Variants and Complications
with Radiologic–Pathologic Correlation. Radiographics. 2000;20:1407-34.
2. Purandare NC, Rangarajan V, Agarwal M, et al. Integrated PET/CT in evaluating sar­comatous
transformation in osteochondro­mas. Clin Nucl Med. 2009;34(6):350-4.
Musculoskeletal System Imaging 91
CASE 12

A 19-year-old male presented with mild swelling and discomfort on the lateral aspect of the
right leg.

Figures 1A and B  AP and


lateral radiographs of
the leg show an oval cystic
expansion of the mid-
shaft of the fibula, with
thinning of the cortex,
but no fracture A B

Figure 2A
92 A Teaching Atlas of Case Studies in Diagnostic Imaging

Figures 2B and C
Figures 2A to C  T1w, T2w
and inversion recovery
coronal MR images show
low signal intensity on
T1w and high on the other
sequences. The lesion was
considered a unicameral
C bone cyst
Musculoskeletal System Imaging 93
CASE 13

A 35-year-old woman presented with a painful right wrist following minor trauma.

Figure 1  Plain radiograph


of the wrist shows an
expansile lucent lesion in
the distal metaphysis of
the ulna, with a
pathological fracture

Figure 2A
94 A Teaching Atlas of Case Studies in Diagnostic Imaging

Figure 2B
Figures 2A and B  (A) T1w
coronal MRI image showing
a low signal in the lesion;
(B) Enhances profusely with
gadolinium

Figure 3A
Musculoskeletal System Imaging 95

Figure 3B
Figures 3A and B  (A) T2w
coronal and axial images
show high signal intensity;
(B) There are fluid/fluid
levels, better shown in the
axial image. The lesion was
considered an aneurysmal
bone cyst

DISCUSSION
A unicameral bone cyst (UBC), otherwise known as simple bone cyst, occurs in children between
ages 5 and 20 years. They are often not readily distinguishable from aneurysmal bone cyst (ABC)
on radiography, but the latter has characteristic features, such as fluid levels and septations.
ABC lesions show low signal intensity on T1w and high on T2w MRI sequences.

FURTHER READING
1. Cottalorda J, Bourelle S. Modern concepts of primary aneurysmal bone cyst. Archiv Orthopaed
Trauma. 2007;127(2):105-14.
2. Mahnken A, Nolte-Ernsting C, Wildberger J, et al. Aneurysmal bone cyst: Value of MR imaging
and conventional radiography. European Radiol. 2003;13(5):1118-24.
96 A Teaching Atlas of Case Studies in Diagnostic Imaging

CASE 14A

An 11-year-old girl presented with a blunt domestic injury to the right thigh.

Figure 1  Ultrasound shows


a 15 mL hematoma in the
vastus lateralis muscle
Musculoskeletal System Imaging 97
CASE 14B

A 20-year-old man presented with pain in the left thigh and limping following a football injury.

Figures 1A and B  T1w and


T2w axial MR images show
that the vastus intermedius
muscle is badly torn, along
with fibers of the vastus
lateralis, with intra- and
intermuscular hematoma B
98 A Teaching Atlas of Case Studies in Diagnostic Imaging

CASE 15

A 50-year-old man was seen one week after falling off a horse, with tenderness and swelling in the
medial aspect of the left thigh with difficulty in walking.

Figures 1A and B  Coronal


reformatted and axial
contrast enhanced CT
images show a large
collection in the vastus
medialis muscle, with rim
enhancement, which can be
associated with a subacute/
chronic hematoma or
abscess formation. Note also
B the subcutaneous collection
Musculoskeletal System Imaging 99
CASE 16

An 88-year-old man sustained a fall at home and presented with pain and swelling of the right knee.

Figures 1A and B B
100 A Teaching Atlas of Case Studies in Diagnostic Imaging

Figure 1C
Figures 1A to C  MR sagittal
images of the knee show
complete detachment of
the quadriceps tendon from
the patella and retraction
with extensive hemathrosis
and surrounding edema
Musculoskeletal System Imaging 101
CASE 17

A 33-year-old woman sustained a right knee injury and presented several days later with pain and
swelling in the knee.

Figures 1A and B B
102 A Teaching Atlas of Case Studies in Diagnostic Imaging

Figures 1C and D
Figures 1A to D  Sagittal
and axial MR images of the
knee showing
fluid/fluid levels from a
D subacute hemarthrosis
Musculoskeletal System Imaging 103

DISCUSSION (CASES 14–17)


The imaging of musculoskeletal trauma spans from conventional radiography to ultrasonography,
CT and MRI. MRI is the imaging of choice, because of its ability to define details, such as cartilage
injury, hemarthrosis, muscle contusion, etc.

FURTHER READING
1. Ahn JM, El-Khoury GY. Role of Magnetic Resonance Imaging in musculoskeletal trauma.
Top Magn Reson Imaging. 2003;18(3):158-68.
2. May DA, Disler DG, Jones EA, et al. Abnormal signal intensity in skeletal Muscle at MR imaging:
Patterns, Pearls, and Pitfalls. Radiographics. 2000;20:295-315.
3. Rubenstein J. The value of diagnostic imaging in musculoskeletal trauma. Current Orthopaedics.
2003;17(5):346-59.
104 A Teaching Atlas of Case Studies in Diagnostic Imaging
CASE 18

A 10-year-old boy had a sudden, severe pain in the back of his left leg while playing football.
The leg was swollen, and movements were restricted. MRI of the leg and ankle was performed.

Figure 1  A STIR
sagittal image shows
complete rupture of the
Achilles tendon, tendon
discontinuity and retraction
of its ends. The tendon gap
is filled with hemorrhage
and edema, showing high
signal intensity.
The tarsal navicular shows
intense bone edema.
T2-weighted axial images
show the ruptured tendon
to be diffusely swollen due
to hemorrhage and edema,
and showing high signal
intensity

DISCUSSION
The Achilles tendon, the largest tendon in the body, is formed by the confluence of tendons from
the gastrocnemius and soleus muscles. It is normally of low signal intensity diffusely, but some
tendons may have vertical lines of high signal intensity. Ruptures may be partial or complete,
typically measuring 2–6 cm above the calcaneal attachment, but occasionally proximally at its
musculotendinous junction. The left Achilles tendon is more commonly affected. Diagnosis may
also be made by ultrasonography.

FURTHER READING
1. Leffler S, Disler DG. MR imaging of tendon, ligament and osseous abnormalities of the ankle and
hind foot. Radiol Clin North Am. 2002;40:1147-70.
2. Schweitzer ME, Karasick D. MR imaging disorders of the Achilles tendon. AJR Am J Roentgenol.
2000;175(3):613-25.
Musculoskeletal System Imaging 105
CASE 19

A 45-year-old man sustained wrist injury after a fall on the right hand.

Figure 1  There is a waist


fracture of the scaphoid
bone (arrow). No other
injury is shown

DISCUSSION
The scaphoid is the most commonly fractured carpal bone. The mechanism of injury is usually a fall
on the outstretched hand, although fractures can also occur from direct impact. Scaphoid fractures
have a reputation for non-union, and a 15–30% rate of associated avascular necrosis of the proximal
pole, a phenomenon that results from the peculiar blood supply of the scaphoid bone.
Many occult fractures of the scaphoid occur, being imperceptible in the acute stage on plain
radiography. MRI can reveal fractures when radiographs are normal or equivocal, although false
positives have been reported. MRI is also useful in assessing possible avascular necrosis. CT has
also been found useful in equivocal cases of scaphoid fracture.

FURTHER READING
1. De Zwart AD, Beeres FJP, Ring D, et al. MRI as a reference standard for suspected scaphoid fractures.
Br J Radiol. 2012;85:1098-01.
2. Hunter JC, Escobedo EM, Wilson AJ, et al. MR Imaging of clinically suspected scaphoid fractures.
AJR Am J Roentgenol 1997;168(5):1287-93.
3. Stevenson JD, Morley D, Srivastava S, et al. Early CT for suspected occult scaphoid fractures. J Hand
Surg Eur. 2012;37(5):447-51.
106 A Teaching Atlas of Case Studies in Diagnostic Imaging
CASE 20

A 56-year-old male fell and injured his wrist which was clinically swollen and deformed.

Figures 1A and B  AP
and lateral radiographs
of the wrist revealed a
comminuted fracture of
the distal radius with
posterior displacement
(Colles fracture). There
is also a scapholunate
dislocation of the carpus,
with widening of the
scapholunate interval
(Terry Thomas or David
Letterman Sign), and the
signet ring sign of the
B scaphoid on the AP view
Musculoskeletal System Imaging 107

DISCUSSION
Colles fracture is the most common fracture around the wrist in adults, and tends to result from a
fall on the outstretched hand. Up to 8 classifications of Colles fractures have been described. It may
be associated with injuries in the carpal bones, as in this patient.

FURTHER READING
1. Campbell R. Lunate and peri-lunate dislocations. J Bone Joint Surg Br. 1964;46:55-72.
2. Rao N, Hrehorovich P, Mathew M. Acute osseous injury to the wrist. Imaging of the
Musculoskeletal system. In: Pope TL, et al. (Eds) Vol. 1, Saunders Elsevier, Philadelphia; 2008.
pp. 309-21.
108 A Teaching Atlas of Case Studies in Diagnostic Imaging
CASE 21

A 23-year-old man sustained trauma to his right knee during a football game. Clinically, he had
signs highly suggestive of a medial meniscus tear. An MRI was requested.

Figure 1  Sagittal T2w MR


image of the right knee shows
a low-intensity structure lying
anteroinferior to and parallel
to the PCL

Figure 2  Sagittal T2w fat


suppressed image shows
complete fragmentation
of the posterior horn of
the medial meniscus, with
associated joint fluid
Musculoskeletal System Imaging 109

DISCUSSION
The “double posterior cruciate ligament sign” is associated with a bucket-handle tear of the medial
meniscus, in which a displaced but attached meniscal fragment appears as a low intensity band
that lies anteroinferior to the posterior cruciate ligament. The sign has a specificity of between
98–100%.

FURTHER READING
1. Camacho MA. The Double Posterior Cruciate Ligament Sign. Radiology. 2004;233(2):503-4.
2. Ruff C, Weingardt JP, Russ PD, Kilcoyne RF. MR imaging patterns of displaced meniscus injuries of
the knee. AJR Am J Roentgenol. 1998;170:63-7.
110 A Teaching Atlas of Case Studies in Diagnostic Imaging
CASE 22

A 22-year-old man was brought to the emergency room having sustained a blunt blow to the anterior
neck at a construction site. The neck was swollen and tender. Initial plain radiographs of the neck
showed soft-tissue emphysema. A contrast-enhanced CT study was done.

Figure 1  Coronal reformatted


CT image shows extensive
soft-tissue emphysema from
laceration to the cervical
trachea, more extensive on
the right

Figure 2  Axial CT neck at


the level of the thyroid gland
shows the emphysema
anterior to the thyroid.
There was no major
vascular injury
Musculoskeletal System Imaging 111

Figure 3  Sagittal
reformatted CT image
showing the emphysema
anterior to the trachea
and tracking into the
superior mediastinum.
The esophagus also
contains a lot of air

DISCUSSION
Tracheobronchial injuries are rare and can be life-threatening. They can arise from external causes
from blunt or penetrating trauma, or can be iatrogenic from tracheal intubation. Tracheal injury
can be easily overlooked, therefore delaying prompt intervention, with serious consequences.

FURTHER READING
1. Chen JD, Shanmuganathan K, Mirvis SE, et al. Using CT to diagnose tracheal rupture. AJR Am J
Roentgenol. 2001;176:1273-80.
2. Moriwaki Y, Sugiyama M, Matsuda G, et al. 3-dimentionally reconstructed Computed Tomography
imaging for Diagnosis of the site of Tracheal injury (3D–Tracheography). World J Surg. 2005;
29(1):102-05.
3. Scaglione M, Romano S, Pinto A, et al. Acute tracheobronchial injuries: Impact of imaging on
diagnosis and management implications. Eur J Radiol. 2006;59(2):336-43.
4. Stark P. Imaging of tracheobronchial injuries. J Thorac Imaging. 1995;10:206-19.
112 A Teaching Atlas of Case Studies in Diagnostic Imaging
CASE 23

A 10-year-old boy presented with an 8-week history of dull pain and swelling over the lateral
lower aspect of the right leg.

Figure 1  AP radiograph
of the ankle shows an oval
lucency in the distal fibular
metaphysis, lying just
beneath the epiphyseal plate.
The surrounding bone is
sclerotic

Figure 2  T2w coronal MR


image shows the fibular
lesion to be hyperintense,
with overlying soft tissue
edema
Musculoskeletal System Imaging 113

Figure 3  T2w sagittal MR


image shows soft tissue
swelling anterior to the
fibula. Note the associated
periosteal reaction

Figure 4  T2w axial image


showing concentric lines
around the lesion, giving a
“double line” or “penumbra”
sign. The features are those
of Brodie’s abscess
114 A Teaching Atlas of Case Studies in Diagnostic Imaging

DISCUSSION
Brodie’s abscess is a chronic abscess of bone surrounded by dense fibrous tissue and sclerotic bone.
It is thought to develop as a result of increased host resistance and decreased bacterial virulence.
Although Staph. aureus is the most frequently isolated organism, cultures are often negative.
Brodie’s abscess occurs commonly in children, and may be difficult to diagnose because of its
insidious onset and mild symptoms. It is frequently confused with benign or malignant bone tumors.
Besides plain films, MRI and nuclear medicine imaging are useful modalities for its investigation.
The “penumbra” is a rim of vascularized granulation tissue surrounding the abscess (See Fig. 4).

FURTHER READING
1. Davies AM, Grimer R. The penumbra sign in subacute osteomyelitis. Eur Radiol. 2005;15(6):
1268-70.
2. Lopes TD, Reinus WR, Wilson AJ. Quantitative analysis of the plain radio­graphic appearance of
Brodie’s abscess. Invest Radiol. 1997;32(11):51-8.
3. Prandini N, Lazzeri E, Rossi B, et al. Nuclear Medicine imaging of bone infections. Nucl Med
Commun. 2006;27(8):633-44.
4. Rasool MN. Primary subacute haemato­genous osteomyelitis in children. J Bone Joint Surg Br.
2001;83(1):93-8.
Musculoskeletal System Imaging 115
CASE 24

A 47-year-old female presented with backache, pelvic pain, abdominal pain and loss of weight.
Physical examination revealed weight loss and bone tenderness. A plain radiograph of the
pelvis (See Fig. 1A) necessitated a skeletal survey including radiographs of the hands and wrists
(See Figs 1B and 1C).

Figures 1A and B B
116 A Teaching Atlas of Case Studies in Diagnostic Imaging

Figure 1C
Figures 1A to C  The bones
show severe osteope­nia
with coarse trabeculae and
diffuse cortical change.
The hands show classical
subperiosteal phalangeal
resorption diagnostic of
hyperparathyroidism

DISCUSSION
The most common cause of primary hyper­parathyroidism (90%) is a solitary adenoma of the
parathyroid gland. Rarer causes include hyperplasia of all four glands and carcinoma. In addition
to subperiosteal resorption and osteopenia, other classical radiological findings in primary
hyperparathyroidism include brown tumors, chodrocalcinosis, especially in the knee and waist,
soft tissue calcification, nephrocalcinosis and renal calculi. Because of the widespread availability
of routine serum chemistry panels, these classic findings in adults are today largely of historical
interest, but they are still not uncommonly encountered in developing countries.
Elevated serum calcium and parathyroid hormone levels establish the diagnosis of primary
hyperparathyroidism. Differential diagnosis includes idiopathic or familial hypercalcemia, multiple
endocrine neoplasia (MEN type I) syndrome, multiple myeloma and sarcoidosis.

FURTHER READING
1. Ashebu SD, Dahniya MH, Muhtaseb SA, Aduh P. Unusual florid skeletal manifesta­tions of primary
hyperparathyroidism. Skelet Radiol. 2002;31:720-3.
2. Hayes CW, Conway WF. Hyperpar­athyroidism. Radiol Clin North Am. 1991;29(1):85-96.
3. Sundaram M, Schils J. Hyperparathyroidism, renal osteodystrophy, osteomalacia and rickets. In:
Pope, Bloem B, et al. (Eds). Imaging of Musculoskeletal System (Vol. II). Saunders-Elsevier; 2008.
pp. 1509-10.
Musculoskeletal System Imaging 117
CASE 25

A 45-year-old female was referred for radio­logical investigation because of tiredness and bone
pains.

Figures 1A and B  Radio­


graphs of the pelvis
(A) with close up;
(B) Classical Looser’s
zones (pseudofractures,
Milkman’s fractures) in
the medial aspects of the
femoral necks, a typical site
for these fractures.
They are pathognomonic of
osteomalacia, and appear
as lucent unmineralized
linear areas perpendicular
to the cortex, and with
sclerotic margins B
118 A Teaching Atlas of Case Studies in Diagnostic Imaging

DISCUSSION
Bone matrix mineralization depends on adequate 1,25-dihydroxy vitamin D, calcium, phosphorus,
alkaline phosphatase and a normal body pH. Deficiency of any of these results in defective
bone mineralization, resulting in osteomalacia in adults, and rickets in children. Vitamin D
deficiency may be nutritional or due to malabsorption, chronic renal disease, chronic liver disease,
or to genetic abnormalities. Looser zones must be differentiated from insufficiency fractures
seen in osteoporosis and Paget’s disease. Complete fractures can occur through Looser’s zones.
In rickets, there is apparent widening of the growth plate, metaphyseal cupping, fraying and
irregularity and various deformities due to bone softening, such as rachitic rosary, genu valgum
and varum, Harrison’s sulcus, etc. Rickets and osteomalacia may be induced by certain tumors,
e.g. hemangiopericytomas.

FURTHER READING
1. Pitt MJ Rachitic and osteomalacic syn­dromes. Radiol Clin North Am. 1981;19:581-99.
2. Kline MJ, Chew FS, et al. Imaging in osteomalacia and renal osteodystrophy. Medscape. 2011;292997.
Musculoskeletal System Imaging 119
CASE 26

A 50-year-old male presented with recurrent attacks of pain, swelling and progressive deformity of
his hands and feet over a period of 6 years. He was relatively symptom-free between attacks.

Figure 1  AP radiograph
of his right hand shows
large, eccentric soft tissue
lumps (tophi). Multiple
erosions are shown in the
carpometacarpal and in
the proximal and distal
interphalangeal joints.
Cystic erosions and erosions
with overhanging edges are
revealed. The appearances
are typical of chronic
tophaceous gout

DISCUSSION
Gout results from an inborn error of purine metabolism that causes hyperuricemia and deposition
of monosodium urate crystals in joints and soft tissues, resulting in recurrent episodes of acute
arthritis. It is more common in men. Patients may have asymptomatic hyperuricemia, acute gouty
arthritis or chronic tophaceous gout. Radiological findings in the chronic form include asymmetric
soft tissue swellings (tophi) and typical punched out erosions or erosions with overhanging margins.
Spinal involvement is rare. The sacroiliac joints may be involved, but the metatarsophalangeal joint
of the big toe is most commonly affected.

FURTHER READING
1. Gottlieb NL, Gray RG. Allopurinol associated hand and foot deformities in chronic tophaceous
gout. JAMA. 1977;238:1663-4.
2. Perez-Ruiz F, Dalbeth N, Urresola A, et al. Imaging of gout: findings and utility. Arthritis Res Ther.
2009;11(3):223.
3. Resnick D, Reinke RT, Taketa RM. Early onset gouty arthritis. Radiology. 1975;114:67-73.
120 A Teaching Atlas of Case Studies in Diagnostic Imaging
CASE 27

A 10-year-old boy presented with a 4-year history of swelling of both thighs, which became
progressively firmer and was associated with limitation of movements across the knees. He had
plain radiographs of both thighs.

Figures 1A and B  AP and


lateral plain radiographs
show sheets of linear
amorphous calcification
along tendons and muscles
of both thighs, sparing the
skin (Courtesy: Dr Adekunle
B Abdulkadir)
Musculoskeletal System Imaging 121

Diagnosis: Juvenile dermatomyositis with calcinosis.

DISCUSSION
Dermatomyositis affects all ages, but the form detected in childhood or adolescence is known
as juvenile dermatomyosits (JDM). The peak incidence of JDM is 9–14 years. It is a rare disease
affecting 2–4 children per million worldwide. Development of calcinosis is one of the sequelae of
JDM in 10–50% of cases.
Plain radiographic features include soft tissue calcifications that manifest as calcareous
deposits in the intermuscular fascial planes or subcutaneous tissue. The characteristic sheet-like
calcification along the proximal striated muscles, as in this patient differentiates JDM from the
dystrophic punctate calcifications of scleroderma. The etiology is unknown, but is largely believed
to be autoimmune. It has also been reported in association with abdominal tuberculosis.

FURTHER READING
1. Awotedu AA, Komolafe F. Calcinosis universalis associated with tuberculosis. Pediatr Radiol.
1984;14:177-9.
2. Blane CE, White SJ, Braunstein EM, Bowyer SL, Sullivan DB. Patterns of calcification in childhood
dermatomyositis. Am J Roentgenol. 1984;142:397-400.
3. Escorial BM, Solis SP, Baeza VM, Alonso Rubio A, de Gregorio AZ. Juvenile amyopathic
dermatomyositis and calcinosis. An Pediatr (Barc). 2005;62:286-8.
4. Pachman LM, Hayford JR, Chung A, Daugherty CA, Pallansch MA, et al. Juvenile dermatomyositis
at diagnosis: Clinical characteristics of 79 children. J Rheumatol. 1998;25:1198-204.
122 A Teaching Atlas of Case Studies in Diagnostic Imaging
CASE 28

A 14-year-old male presented with recurrent pain and swelling of his left knee. Plain X-rays were
normal. MRI of the knee was done.

Figures 1A and B  Sagittal


images show nodular low
T1 and T2 signal intensity in
B the joint fluid
Musculoskeletal System Imaging 123

Figures 2A and B  T2 fat


suppressed sagittal and
axial images show the same
low intensity regions with
characteristic “blooming”
artefacts due to the T2 effect
of hemosiderin B
124 A Teaching Atlas of Case Studies in Diagnostic Imaging

Diagnosis: Pigmented villonodular synovitis (PVNS).

DISCUSSION
The PVNS is characterized by a benign hyper­plastic synovial proliferation, along tendon sheaths
and periarticular fascial planes or within bursae. If it occurs outside the joint, it is referred to
as giant cell tumor of tendon sheath. Its etiology is uncertain. It may be diffuse or nodular, and
most commonly involves the knee. Histologically, the hypertrophied synovium contains giant
cells and histiocytes, and both intracellular and extracellular hemosiderin, which accounts for
the characteristic MR imaging appearance. Gadolinium-enhanced images provide no additional
information. Treatment is by arthroscopic synovectomy but recurrence rates remain high in cases
of diffuse PVNS.

FURTHER READING
1. Cheng XE, You YH, Liu W, et al. MRI features of pigmented villonodular synovitis. (PVNS).
Clin Rheumatol. 2004;23(1):31-4.
2. Eckhardt BP, Hernandez RJ. Pigmented villonodular sinovitis. MR imaging in paediatric patients.
Pediatr Radiol. 2004;34(12):943-7.
3. Goldman AB, Dicarlo EF. Hyperpigmented villonodular synovitis. Radiol Clin North Am.
1988;26(6):1327-47.
Musculoskeletal System Imaging 125
CASE 29

A 23-year-old male presented with pain and swelling of his left knee. Plain X-rays showed soft tissue
swelling only. MRI was done.

Figures 1A and B  T2-


weighted, gradient and
STIR images were acquired.
A well-defined cystic
lobulated lesion is seen,
partly intrameniscal in the
lateral meniscus and partly
adjacent to the meniscus
with fluid signal intensity
on all pulse sequences.
It is typical of a meniscal/
parameniscal cyst B
126 A Teaching Atlas of Case Studies in Diagnostic Imaging

DISCUSSION
Meniscal cysts arise within (meniscal) or adjacent to (parameniscal) the meniscus in association
with a meniscal tear. They most commonly present in men 20–40 years old, and result in focal
swelling adjacent to the joint, usually on the lateral side. On MRI, they are of fluid signal intensity
on all sequences and are in direct contact with the adjacent meniscal tear in over 90% of cases.
They may extend anteriorly, posteromedially, posterolaterally into the iliotibial band and anteriorly
into Hoffa’s fat pad. They may occasionally erode bone.

FURTHER READING
1. Campbell SE, Sanders TG, Morrison WB. MR imaging of meniscal cysts: Incidence, location and
clinical significance. Am J Roentgenol. 2001;177:409-13.
2. De Maeseneer M, Shahabporer M, Vanderdood K, et al. MR imaging of meniscal cysts: Evaluation
of location and extension using a three-layer approach. Eur J Radiol. 2001;39:117-24.
Musculoskeletal System Imaging 127
CASE 30

A 32-year-old man was involved in a high velocity road traffic accident and was admitted with
severe back pain and paraparesis.

Figures 1A and B  T1w and


T2w sagittal MR images of
the thoracolumbar spine
show a wedge fracture of
L2 with dorsal displacement
into the spinal canal, leading
to severe compression of
the cauda equina. Note
hematoma collection
mainly within the posterior
and mildly, the anterior
longitudinal ligaments, and
the marrow edema in the
posterior elements B
128 A Teaching Atlas of Case Studies in Diagnostic Imaging

DISCUSSION
Thoracolumbar spinal injuries most frequently result from falls from height or from high velocity
motor vehicle accidents. Although conventional radiography is required for the initial evaluation
of such injuries, it has major limitations. CT is the best means to assess the posterior elements,
exclude instability and the degree of spinal canal compromise. Its value is greatly enhanced with
sagittal and coronal reformation.
The advantage of MRI in acute spinal injury is its ability to demonstrate the high signal intensity
associated with acute hemorrhage, demonstrate any post-traumatic herniated disc, ligamentous
and bone edema, as well as spinal cord compression.

FURTHER READING
1. Dai LY. Wang XY, Jiang S, et al. Plain radiography versus computed tomography scans in the
diagnosis and management of thoracolumbar burst fractures. Spine. 2008;33(16):E 548-52.
2. Dai LY, Ding WG, Wang XY, et al. Assessment of ligamentous injury in patients with thoracolumbar
burst fractures using MRI. J Trauma. 2009;66(6):1610-5.
3. Inamasu J, Guiot BH. Thoracolumbar junction injuries after motor vehicle collision: Are there
differences in restrained and non-restrained front seat occupants? J Neurosurg Spine. 2007;7(3):
311-4.
Musculoskeletal System Imaging 129
CASE 31

A 17-year-old boy presented with pain in the left leg following a minor fall.

Figure 1  AP radiograph
of the leg shows an
expansile distal diaphyseal/
metapyseal lesion with
overlying ground-
glass opacity. Note the
undisplaced spiral fracture.
Features are typical of
fibrous dysplasia

Figure 2A
130 A Teaching Atlas of Case Studies in Diagnostic Imaging

Figure 2B
Figures 2A and B  Sagittal
and coronal reformatted
CT images of the leg
showing internal scalloping
of the cortex

Figure 3  Axial CT showing


the ground-glass opacity
in the tibia
Musculoskeletal System Imaging 131

DISCUSSION
Fibrous dysplasia is a bone disease of unknown etiology in which there is an abnormal differentiation
of osteoblasts leading to the replacement of normal marrow and cancellous bone by immature
bone and fibrous stroma. It is usually discovered incidentally or when complicated by pathological
fracture, as in this patient. Many occur in association with McCune-Albright syndrome. Fibrous
dysplasia can affect any bone, but it is usually found in the proximal femur, the tibia, humerus, ribs
and craniofacial bones, in descending order. It may be monostotic or polyostotic, and in the latter,
it is usually unilateral.
The typical fibrous dysplasia shows a ground-glass appearance both on plain radiographs and
on CT. On MRI, it usually shows a low to intermediate signal intensity on T1w, intermediate to high
on T2w, with heterogeneous enhancement with gadolinium.

FURTHER READING
1. Bulakbasi N, Bozlar U, Karademir I, et al. CT and MRI in the evaluation of craniospinal
involvement with polyostotic fibrous dysplasia in McCune-Albright syndrome. Diagn Interv Radiol.
2008;14(4):177-8.
2. Fitzpatrick KA, Taljanovic MS, Speer DP, et al. Imaging findings of fibrous dysplasia with
histopathologic and intraoperative correlation. AJR Am J Roentgenol. 2004;182(6):1389-98.
3. Shah ZK, Peh CG, Koh WL, Shek TWH. Magnetic resonance imaging appearances of fibrous
dysplasia. Brit J Radiol. 2005;78:1104-15.
132 A Teaching Atlas of Case Studies in Diagnostic Imaging
CASE 32

A 30-year-old man presented with a 15-year history of pain and limited range of movement in
the right hip. The symptoms were preceded by blunt trauma to the hip.

Figure 1  Frontal radiograph


of the right hip shows a
densely calcified mass seen
in the anterolateral soft
tissues adjacent to anterior
inferior iliac spine

Figure 2  Lateral radiograph


of the right hip shows the
densely calcified mass in
the soft tissues adjacent
to the anterior inferior iliac
spine (Courtesy: Dr Sujatha
Rajkumar)
Musculoskeletal System Imaging 133

DISCUSSION
Myositis ossificans (now named heterotopic ossification) is a benign self-limiting ossifying soft
tissue mass typically occurring within skeletal muscle. It is classified into three types: progressive
(hereditary), post-traumatic, and paraplegic. Post-traumatic myositis accounts for 75% of cases and
is more common in young athletic males. Paraplegic myositis occurs below the level of paralysis.
Other causes include burns and tetanus.
The typical radiographic appearance of my­ositis ossificans is circumferential calcification in
the soft tissues, with a lucent center. A radiolucent cleft separates the lesion from the cortex of the
adjacent bone. Calcification usually begins to become apparent within 2–6 weeks and the lesion
reaches the classic, well-circumscribed peripherally calcified appearance by 2 months. Over the
following months, the calcification becomes denser.
CT demonstrates well-defined mineraliza­tion at periphery of the lesion after 4–6 weeks and less
distinct lucent center. The cleft between it and the subjacent bone is usually visible.
Intense tracer accumulation is seen on bone scan. Mature ossification phase shows reduced
activity and surgery may be performed with little risk of recurrence.
In early stages, it is difficult to differentiate myositis ossificans radiologically from soft tissue
sarcomas.

FURTHER READING
1. Grainger and Allison’s Diagnostic Radiology, A text book of medical imaging, 5th edition.
Chapter 45, Techniques and imaging of soft tissues. 2008; pp. 960-63.
2. Tyler P, Saifuddin A. The imaging of myositis ossificans. Sem Musculoskeletal Radiol. 2010;14(2):
201-16.
3. Wolfgang Dahnert. Bone and soft tissue disorders, Radiology Review Manual. Lippincott Williams
& Wilkins, 7th edition, 2011. pp. 131-2.
134 A Teaching Atlas of Case Studies in Diagnostic Imaging
CASE 33

A 36-year-old male presented with severe left hip pain and limitation of movements. There was no
history of trauma. Plain films show osteopenia of the left femoral head.

B Figures 1A and B
Musculoskeletal System Imaging 135

Figures 1C and D
Figures 1A to D  T1-
weighted images show
reduced signal intensity
of the left femoral head,
neck and inter-trochanteric
region, with postcontrast
enhancement. STIR and
T2-weighted fat saturated
images show high signal
intensity and joint effusion.
A repeat study after
6 months showed
complete resolution D
136 A Teaching Atlas of Case Studies in Diagnostic Imaging

Diagnosis: Transient osteoporosis of the hip joint.

DISCUSSION
Idiopathic transient osteoporosis of the hip joint is an uncommon and usually self-limiting clinical
entity of unknown cause. It primarily affects middle-aged men and pregnant women. Generally,
only one hip is affected at a time. Recurrence in the same hip can occur. When other joints are
involved, the entity becomes known as regional migrating osteoporosis. Differential diagnosis
includes early avascular necrosis of the femoral head, septic arthritis and intra-articular osteoid
osteoma.

FURTHER READING
1. Guerrra J, Steinberg M. Distinguishing transitional osteoporosis from avascular necrosis of the hip.
J Bone Joint Surg (Am). 1995;77:616-24.
2. Yamamoto T, Kubo T, Hirasawa Y. A clinicopathological study of transient osteoporosis of the hip.
Skeletal Radiol. 1999;28:621-7.
Musculoskeletal System Imaging 137
CASE 34

A 30-year-old woman had a pre-employment chest X-ray. Incidental discovery of multiple bone
lesions was made.

Figure 1  Chest X-ray


showing dense bone lesions
around the shoulders and
ribs. Normal heart and lungs

Figure 2A
138 A Teaching Atlas of Case Studies in Diagnostic Imaging

Figure 2B
Figures 2A and B  X-ray
right and left shoulders
showing multiple dense
bony foci, involving the
humeri, scapulae and
clavicles

Figure 3  X-ray pelvis


showing widespread,
punctuate sclerotic lesions
in the bony pelvis
Musculoskeletal System Imaging 139

Diagnosis: Osteopoikilosis.

DISCUSSION
Osteopoikilosis is an asymptomatic bone lesion which is usually discovered incidentally. It is
inherited as an autosomal dominant trait, and has no sex or age predilection. It has an estimated
prevalence of 1 in 50,000. The lesions are well-marginated and in long bones, are located in
the epiphyses and metaphyses. No abnormality is demonstrated on radionuclide bone scan.
Osteopoikilosis should be differentiated from osteoblastic metastases, tuberous sclero­sis and
mastocytosis. In none of these condi­tions are the lesions so discrete as they are in osteopoikilosis.

FURTHER READING
1. Benli IT, Akalin S, Boysan E. Epidemiological, clinical and radiological aspects of osteopoikilosis.
J Bone Joint Surg. 1992;­74(4):504-6.
2. Khot R, Sikarwar JS, Gupta RP. Osteopoikilosis: A Case Report. Ind J Radiol Imag. 2005;15:4453-4.
3. Primo GD. Benign spotted bones: a diagnos­tic dilemma. CMAJ. 2011;83(4):456-9.
140 A Teaching Atlas of Case Studies in Diagnostic Imaging
CASE 35

A 36-year-old man was referred for a routine chest X-ray for employment purposes. Although his
physical examination was normal, the chest X-ray findings necessitated a skeletal survey.

Figure 1  Chest radiograph


showing normal heart and
lungs. Expansion and patchy
sclerosis of the ribs are
shown. The diaphyses of both
clavicles are also expanded
and show dense sclerosis,
with normal metaphyseal
ends. The scapulae are
sclerotic

Figure 2  AP pelvis and hips


radiograph shows bilateral
uniform sclerosis of the iliac
bones centrally with linear,
nodular and patchy sclerosis
peripherally. The sacrum
and pubis are normal. The
proximal femoral shafts
show dense cortical sclerosis
reminiscent of the “flowing
candle wax” appearance of
melorheostosis
Musculoskeletal System Imaging 141

Figures 3A and B  AP and


lateral radiographs of the
skull show marked, uniform
sclerosis and thickening of
the cranial vault and base,
with involvement of the
maxillae and mandibles B
142 A Teaching Atlas of Case Studies in Diagnostic Imaging

Figure 4  Radiograph
of both hands show no
abnormality

Figure 5A
Musculoskeletal System Imaging 143

Figure 5B
Figures 5A and B  AP
and lateral radiographs of
thoracolumbar spine show
normal appearances

DISCUSSION
The radiological findings are nonspecific, but a few differential diagnostic possibilities are suggested:
•• The diaphyseal changes are seen in progressive diaphyseal dysplasia (Camurati-Engelmann
disease), but this patient is asymptomatic, with none of the features of this disease such as bone
pain and tenderness, leg weakness, neuromuscular dystrophy, etc. Also, only the skull base is
involved in this disease.
•• In Ribbing Disease (hereditary multiple diaphyseal sclerosis), only the long bones are involved
and the changes are asymmetric.
•• van Buchem disease (generalized cortical hyperostosis) shows typical facial changes with facial
nerve palsies. Headaches due to raised intracranial pressure, auditory and ocular disturbances
due to foraminal narrowing are common, in addition to the symmetrical generalized sclerotic
bony changes.
•• Polyostotic fibrous dysplasia is usually unilateral and asymmetric, changes are varied and
deformities may result.
•• Osteopetrosis and Pycnodysostosis are two dysplasias in which bone sclerosis is uniform and not
confined to the diaphyses. In pycnodysostosis the changes in the skull and mandibles include
wormian bones, prognathia and resorption of the lateral ends of the clavicles.

FURTHER READING
1. de Vernejoul MC. Sclerosing bone disorders. Best Pract Res Clin Rheumatol. 2008;22:71-83.
2. Janssens K, Vanhoenacker F, Bonduelle M, et al. Camurati-Engelmann disease: Review of the clinical,
radiological, and molecular data of 24 families and implications for diagnosis and treatment. J Med
Genet. 2006;43:1-11.
3. Wengenroth M, Vasvari G, Federspil FA, et al. Case 150: van Buchem Disease (Hyperostosis
Corticalis Generalisata. Radiology. 2009;253:272-6.
144 A Teaching Atlas of Case Studies in Diagnostic Imaging
CASE 36

A 78-year-old woman with poorly controlled diabetes and hypertension complained of swelling
and pain in the right shoulder for the previous 3 years. She had below knee amputation on the right
several years earlier because of diabetic foot gangrene.

Figure 1  AP radiograph
of the right shoulder. The
shoulder joint is destroyed,
with extensive resorption of
the humeral head and neck.
Note the bone fragments
scattered in the lesion.
The appearance is classical
of Charcot joint

DISCUSSION
Charcot joint (neuropathic arthropathy) is a chronic, degenerative arthropathy associated with
diminished sensory innervation. The common causes include: Diabetes, syringomyelia and tertiary
syphilis (Tabes dorsalis). Other causes include: Leprosy, myelomeningocele, spinal cord injury,
amyloidosis and congenital insensitivity to pain.
Diabetic neuropathic arthropathy tends to affect the foot and ankle joints more often;
syringomyelia, the shoulder and elbow; and syphilis, the knee and hip.
At the early stages, Charcot joint may simulate features of osteoarthritis.

FURTHER READING
1. Brodbelt AR, Stoodley MA. Post-traumatic syringomyelia: A Review. J Clin Neurosci. 2003;10:401-8.
2. Cullen AB, Ofluoglu O, Donthineni R. Neuropathic arthropathy of the shoulder. (Charcot Shoulder).
MedGenMed. 2005;7(1):29.
3. Jones EA, Manaster BJ, May DA, Disler DG. Neuropathic osteoarthropathy: diagnostic dilemmas
and differential diagnosis. Radiographics. 2000;20:S279-93.
4. Nozawa S, Miyamoto K, Nishimoto H, et al. Charcot joint in the elbow associated with syringomyelia.
Orthopedics. 2003;26:731-2.
Musculoskeletal System Imaging 145
CASE 37

A 39-year-old farmer presented with a 7-month history of discharging sinuses from the right foot.
He showed no response to treatment with several antibiotics. He is not diabetic.

Figure 1  Radiograph of the


right foot shows extensive
periosteal reaction with
irregular expansion of the
1st to 4th metatarsals and
some proximal phalanges
(Courtesy: Dr Adekunle
Abdulkadir)

Diagnosis: Fungal osteomyelitis (mycetoma).

DISCUSSION
Mycetoma is a chronic granulomatous infection caused by a mix of bacteria (Actinomycetoma)
and fungi (Eumycetoma). It was first documented in the Indian province of Madura, hence
the eponym Madura foot. It is prevalent in tropical and subtropical regions, but it has also been
reported in other climates. The infection is generally indolent, but if left untreated, it can lead to the
formation of sinuses, abscess and osteomyelitis. Chemotherapy is effective in early cases, but late
cases may require amputation.

FURTHER READING
1. El Bagi, MEA. New radiographic classification of bone involvement in pedal mycetoma. AJR Am J
Roentgenol. 2003;180:665-8.
2. Kumar J, Kumar A, Sethy P, Gupta S. The dot­in-circle sign of mycetoma on MRI. Diagn Interv Radiol.
2007;13:193-5.
146 A Teaching Atlas of Case Studies in Diagnostic Imaging
CASE 38

A 34-year-old male presented with a history of a rapidly growing swelling in his left gluteal region.
The swelling was accompanied by pain and was unresponsive to various types of therapy. CT of the
chest, abdomen and pelvis including the mass was done.

Figure 1  CT of lower abdomen


showing a large soft tissue mass
in the left posterior abdominal
wall, containing central
hypodensity and scattered foci
of calcification

Figure 2  CT chest shows


multiple rounded metastases
of varying sizes
Musculoskeletal System Imaging 147

Figure 3  Sagittal
reformatted image of
CT abdomen, with bone
window settings for
lumbosacral spine shows
extensive osteolytic
destruction of
the 5th lumbar and the
sacral vertebrae

Diagnosis: Rhabdomyosarcoma, confirmed by biopsy.

DISCUSSION
Soft tissue sarcomas, irrespective of their origin, frequently exhibit similar imaging findings.
Calcification is not uncommon. Ultrasonography is not very helpful, the mainstay of diagnosis
being CT and particularly MRI, which are used to distinguish between benign and malignant
lesions, but biopsy is necessary for the final definitive diagnosis. Staging of soft tissue malignancies
is mandatory. A number of staging systems are in use. The tumor is classified by histological grade,
local extent, and presence or absence of metastatic disease.

FURTHER READING
1. Berquist T, Ehman R, King B, et al. Value of MR imaging in differentiating benign from malignant
soft tissue masses: Study of 95 lesions. AJR Am J Roentgenol. 1990;155:1251-5.
2. Kransdoff MJ. Malignant soft tissue tumors in a large referral population: Distribution of diagnosis
by age, sex and location. AJR Am J Roentgenol. 1995;164:395-402.
3. Moulton J, Blebea J, Dunco D, et al. MR imaging of soft tissue masses: diagnostic efficacy and
value of distinguishing benign and malignant lesions. AJR Am J Roentgenol. 1995;164:1191-9.
148 A Teaching Atlas of Case Studies in Diagnostic Imaging
CASE 39

A 45-year-old female was referred to hospital for admission with a three months history of weight
loss, pelvic pain and inability to walk. She had a fungating right breast mass, and was dehydrated
and emaciated.

Figure 1  AP radiograph of
her pelvis reveals extensive
osteolytic destruction of
the right side of the pelvis,
involving the sacrum,
sacroiliac joint, acetabulum
and ischiopubic rami. The
left femur shows extensive
osteolytic destruction
and pathological fracture.
L5 vertebra is uniformly
sclerotic

Diagnosis: Widespread metastatic disease from breast cancer.

DISCUSSION
Metastatic disease of bone is the most common malignant process in the skeleton. The prostate,
thyroid, breast, lung, and kidney account for 80% of the primary sites in adults. Neuroblastoma
and leukemia are the most common primary malignancies to metastasize to bone in children.
Complications of metastatic bone disease include pain, fractures, spinal cord and nerve root
compression, hypercalcemia and marrow suppression. Metastases may be lytic, sclerotic or mixed.
Imaging techniques for diagnosis are radiography, CT, MRI, nuclear medicine, PET and PET/CT.
Differential diagnosis include primary bone tumor, myeloma, brown tumor, histiocytosis X and
fibrous dysplasia.

FURTHER READING
1. Edelstyn G, Gillestrie P, Grebber P. The radiological demonstration of osseous metastases.
Clin Radiol. 1967;18:156-62.
2. Soderlund V. Radiological diagnosis of skeletal metastases. Eur Radiol. 1996;6:587-95.
Musculoskeletal System Imaging 149
CASE 40

A 10-year-old boy presented with pain and progressive swelling of the right lower thigh over
a period of several months, with a doubtful history of preceding trauma. A plain X-ray was done.

Figures 1A and B  AP and


lateral radiographs of the
right lower thigh. There is
sclerosis and expansion of
the distal femur with florid
perpendicular (hair-on
end) periosteal reaction
extending into the swollen
overlying soft-tissues.
There are typical Codman
triangles at the diaphyseal
margins of the mass.
Features are classical
for osteosarcoma.
Ewing’s sarcoma was
considered a differential B
150 A Teaching Atlas of Case Studies in Diagnostic Imaging

DISCUSSION
Osteosarcoma is the most common primary malignancy of bone. The WHO classification comprises
8 subtypes, but classic or conventional osteosarcoma is the most common variety, accounting
for 75% of all cases. Osteosarcoma is most prevalent in the 2nd and 3rd decades of life. The femur
and tibia are the bones most frequently involved (85–95%), followed by the ulna and humerus.
In most cases, the plain radiographic features are diagnostic, but CT and MRI are useful, the latter
because it can better define the extent of the mass into surrounding structures.
The differential diagnosis of osteosarcoma includes Ewing’s sarcoma, chondrosarcoma,
fibrosarcoma and Langerhans cell histiocytosis.

FURTHER READING
1. Murphey MD, Jelinex JS, Temple HT, et al. Imaging of periosteal osteosarcoma: Radio­logic-
pathologic comparison. Radiology. 2004;233(1):129-38.
2. Rana PS, Wu JS, Eisenberg RL. Periosteal Reaction. Am J Roentgenol. 2009;193(4):W259-72.
3. White LM, Kandel R. Osteoid-producing tumors of bone. Semin Musculoskelet Radiol. 2000;4(1):
25-43.
4. Yarmish G, Klein MJ, Landa J, et al. Imaging characteristics of primary osteosarcoma:
Nonconventional subtypes. Radiographics. 2010;30:1653-72.
Musculoskeletal System Imaging 151
CASE 41

A 45-year-old man presented with pain, swelling and limitation of movement of the right knee of
several months duration. Plain X-rays (not shown) and MRI of the knee were done.

Figure 1  Coronal T1w MR


image of the knee shows an
eccentric, subarticular,
low signal intensity mass
in the medial aspect of the
femoral epiphysis

Figure 2  T1w fat saturated


postcontrast image
shows inhomogeneous
enhancement of the mass
with associated soft-tissue
enhancement. The persistent
low signal area within
the mass is considered a
sequestered bone fragment
152 A Teaching Atlas of Case Studies in Diagnostic Imaging

Figures 3A and B  T2w and


PD images show the mass
to be largely of high signal
intensity, with scalloping
of the bone and overlying
soft-tissue edema. Features
B are those of giant cell tumor
Musculoskeletal System Imaging 153

DISCUSSION
Giant cell tumor (GCT) is normally a benign bone tumor characterized by multinucleated
giant cells, similar to osteoclasts. Rarely, it can undergo malignant degeneration. GCT affects
typically the mature bone, with a peak prevalence between 20–40 years. The most common sites
are the proximal tibia, distal femur, distal radius and proximal humerus. It can also affect the pelvis,
spine, and bones of the feet and hands. When it affects long bones, GCT is typically epiphyseal,
eccentric in location and subarticular, with no sclerotic rim on plain radiographs. On MRI, GCT
has low to intermediate signal intensity on T1w, heterogeneously high on T2w, and intensely
high on gradient-echo images. Fluid-fluid levels may be demonstrated on MRI, but this feature is
not specific to GCT. MRI is particularly useful in defining the extension of tumor into the joint or
overlying soft-tissues.
Patients usually present with pain, swelling and limitation of movement, but a pathological
fracture may be the presenting feature in up to 10% of cases.

FURTHER READING
1. Murphey MD, Nomikos GC, Flemming DJ, et al. Imaging of giant cell tumor and giant cell
reparative granuloma of bone: radiologic pathologic correlation. Radiographics. 2001;21:
1283-309.
2. Purohit S, Pardiwala DN. Imaging of giant cell tumor of bone. Ind J Ortho. 2007;41(2):91-6.
3. Stacy GS, Peabody TD, Dixon LB. Mimics on radiography of giant cell tumor of bone. AJR Am J
Roengenol. 2003;181(6):1583-9.
154 A Teaching Atlas of Case Studies in Diagnostic Imaging
CASE 42

An 11-year-old girl presented with a left popliteal fossa swelling of 3 months duration. It was
clinically diagnosed as a Baker’s cyst, and she was referred for MRI.

Figure 1  Sagittal T1w MR


image of the left knee shows
a well-delineated, oval mass
in the subcutaneous layer of
the popliteal fossa

Figure 2  Axial T2w image


with fat suppression, shows
that the mass has a reduced
signal intensity, similar to the
adjacent subcutaneous fat.
The diagnosis of lipoma was
made and confirmed
at surgical excision and
by histology
Musculoskeletal System Imaging 155

DISCUSSION
Lipomas are the most common soft-tissue tumors. They can occur at any age and virtually at any
anatomical location. A common diag­nostic challenge is distinguishing a simple lipoma from a
well-differentiated liposarcoma.

FURTHER READING
1. Kransdorf MJ, Bancroft LW, Peterson JJ, et al. Imaging of fatty tumors: Distinction of lipoma and
well-differentiated liposarcoma. Radiology. 2002;224:99-104.
2. Navarro OM, Laffan EE, Ngan BY. Pediatric soft-tissue tumors and pseudo-tumors: MR imaging
features with pathologic correlation. Part 1. Imaging Approach, Pseudotumors, vascular lesions,
and Adipocytic Tumors. Radiographics. 2009;29:887-906.
3 Urogenital System Imaging

CASE 1

A 42-year-old male presented with recurrent urinary tract infection.

Figure 1  CT abdominal angiogram


showing horse­shoe kidneys deriving
arterial supply from multiple arteries
arising from the abdominal aorta.
Note that the lower pole calyces are closer
to the midline than the upper poles

Figure 2  Contrast CT abdomen showing


fusion of the horseshoe kidneys in their
lower poles
Urogenital System Imaging 157

Figure 3  Post-CT delayed


KUB demonstrating
the horseshoe kidneys

DISCUSSION
See under Case 3 (Page 159).
158 A Teaching Atlas of Case Studies in Diagnostic Imaging
CASE 2

A 14-year-old female with abdominal pain and recurrent urinary tract infection.

Figure 1  MIP contrast CT


images showing horseshoe
kidneys

DISCUSSION
See under Case 3 (Page 159).
Urogenital System Imaging 159
CASE 3

A 50-year-old female presenting with urinary tract infection and microscopic hematuria.

Figure 1  Axial contrast


CT abdomen showing
horseshoe kidneys, fused
at their lower poles

Figure 2  Axial contrast CT


of same patient at the level
of the renal hila showing
distended renal pelves
160 A Teaching Atlas of Case Studies in Diagnostic Imaging

DISCUSSION
Horseshoe kidneys occur in about 1 in 400 people. Fusion involves the lower poles, which may
be fibrous or parenchymal. They are usually associated with a higher risk of infection and calculi
formation. There is also an increased risk of renal cancer, especially Wilm’s tumor and transitional
cell carcinoma.

FURTHER READING
1. Glodny B, Petersen J, Hofmann KJ, et al. Kidney fusion anomalies revisited: Clinical and radiological
analysis of 209 cases of crossed fused ectopia and horseshoe kidney. BJU Int. 2009;103(2):224-35.
Urogenital System Imaging 161
CASE 4

A 45-year-old male with recurrent right iliac fossa pain and microscopic hematuria.

Figure 1  Intravenous
urogram showing a right
ilio­lumbar ectopic kidney.
Note the incomplete
calyceal rotation

DISCUSSION
Renal ectopia results from an incomplete ascent of the kidney from its embryologic position in the
pelvis to its normal location. Normal ascent is accompanied by concomitant rotation, such that
incomplete ascent is associated with incomplete rotation. Very rarely, an ectopic kidney may be
located in the thorax, usually on the left side.

FURTHER READING
1. Komolafe F. Congenital renal ectopia in Nigeria: A Report of 23 cases. Trop Doct. 1984;14(4):180-3.
2. Van der Bosch CM, van Wijk JA, Beckers GM, et al. Urological and nephrological findings of renal
ectopia. J Urol. 2010;183(4):1574-8.
162 A Teaching Atlas of Case Studies in Diagnostic Imaging
CASE 5

A 30-year-old male with bilateral loin pain.

Figure 1  Axial T1w MRI


showing enlarged kidneys
with numerous cysts

Figure 2  Axial T2w image


showing the numerous
cysts of varying sizes
Urogenital System Imaging 163

Figure 3  Coronal
T2w image with fat
suppression, showing better
demonstration of
the renal cysts

DISCUSSION
See under Case 7 (Page 166).
164 A Teaching Atlas of Case Studies in Diagnostic Imaging
CASE 6

A 33-year-old male with right renal colic and microscopic hematuria.

Figure 1  Coronal
reformatted plain CT
abdomen showing enlarged
kidneys with multiple cysts,
right upper pole calculus
and tiny calcific foci in both
kidneys. Note multiple cysts
in the liver

Figure 2  Plain axial CT


showing cysts of varying
sizes within the liver
Urogenital System Imaging 165

Figure 3  T2w coronal MRI


showing multiple renal
and hepatic cysts

DISCUSSION
See under Case 7 (Page 166).
166 A Teaching Atlas of Case Studies in Diagnostic Imaging
CASE 7

A 46-year-old male presenting with bilateral loin pain.

Figure 1  Coronal
reformatted plain CT
showing a grossly enlarged
polycystic kidney extending
into the bony pelvis.
Note scattered calcific foci

Figure 2  Coronal
reformatted contrast CT
showing contrast excretion.
Note the associated hepatic
cysts
Urogenital System Imaging 167

DISCUSSION
There are 2 types of polycystic renal disease: autosomal dominant and the less common autosomal
recessive types. The former presents in adults with abdominal pain as a result of bleeding into a
cyst, hematuria, calculi formation or urinary tract infection. The cysts are of varying sizes, and there
may be associated hepatic and pancreatic cysts. They may also be associated with intracranial
aneurysms and colonic diverticulosis. Hypertension and renal failure may ensue.
In autosomal recessive polycystic disease, there are numerous tiny cysts, and presentation is in
the neonate. They are usually associated with hepatic fibrosis. Death usually occurs in the neonatal
period or early childhood from renal and hepatic failure.

FURTHER READING
1. Torres VE, Harris PC. Autosomal dominant polycystic kidney disease: The last 3 years. Kidney Int.
2009;76(2):149-68.
2. Wilson PD. Polycystic kidney disease. N Engl J Med. 2004;350(2):151-64.
168 A Teaching Atlas of Case Studies in Diagnostic Imaging
CASE 8

A 40-year-old male with recurrent left flank pain.

Figure 1  Intravenous
urogram showing left
hydro­calycosis and a dilated
renal pelvis with an abrupt
transition to a normal
caliber ureter, consistent
with pelviureteric junction
obstruction

DISCUSSION
Pelviureteric junction (PUJ) obstruction occurs in 1/1,000 newborns, with a predilection for the
left side (67%) and male predominance. It may be bilateral in 30% of cases. It may be discovered
in the fetus during obstetric ultrasound. In adults, it is often discovered incidentally, or patients
may present with urinary tract infection, calculus or flank pain. PUJ obstruction is thought to arise
from inadequate canalization of the upper ureteric bud at about the 12th week of intrauterine life.
Some may be secondary to bands or crossing vessels. Nuclear scintigraphy demonstrates marked
accumulation of radioactive tracer on the affected side. PUJ requires no intervention in many cases,
except when there is frank structural obstruction, when pyeloplasty or stenting may be required.

FURTHER READING
1. Rouviere O, Lyonnet D, Berger P, et al. Uteropelvic junction obstruction: Use of helical CT for
preoperative assessment—comparison with intra-arterial angiography. Radiology. 1999;213(3):
668-73.
2. Lawler LP, Jarret TW, Corl FM, Fishman EK. Adult ureteropelvic junction obstruction: Insights with
three-dimensional multi­detector row CT. Radiographics. 2005;25:121-34.
Urogenital System Imaging 169
CASE 9A

A 3-month-old boy presented with dribbling of urine, straining during micturition, and bilateral
reducible inguinal hernias.

Figure 1  Abdominal
ultrasound showed gross
dilatation of the urinary
bladder and posterior
urethra. There was also
bilateral hydronephrosis
(not shown)

Figure 2  Micturating
cystourethrogram (MCUG)
demonstrates the dilated
posterior urethra, giving the
“spinning-top” appearance.
Note the abrupt transition
to a normal caliber
anterior urethra
170 A Teaching Atlas of Case Studies in Diagnostic Imaging
CASE 9B

A 3-year-old boy presenting with painful micturition and poor urinary stream.

Figure 1  MCUG showing


multiple sacculations of
the urinary bladder

Figure 2  MCUG showing


grade 4 vesicoureteric
reflux. Note an incidental
right pelvic ectopic kidney
(Courtesy: Dr Adekunle
Abdulkadir)
Urogenital System Imaging 171

DISCUSSION
Posterior urethral valve (PUV) is a congenital valvular obstruction of the male urethra, with an
incidence of between 1 in 10,000 to 1 in 25,000 live births. The diagnosis may be suspected in
prenatal ultrasonography in the presence of fetal hydronephrosis, urinary bladder distension and
oligohydramnios in a male fetus.
The presence of vesicoureteric reflux, bladder diverticula, and urinary extravasation, with
or without urine ascites, in association with PUV have been recognized as pressure “pop-off”
mechanisms to preserve renal function.

FURTHER READING
1. Kajbafzadeh A. Congenital urethral anoma­lies in boys: Part I: Posterior urethral valves. Urology J.
2005;2:59-78.
2. Krishnan A, de Souza A, Konijeti R, Baskin LS. The anatomy and embryology of posterior urethral
valves. J Urol. 2006;175:11214-20.
3. Rittenberg MH, Hulbert WC, Snyder HM, Duckett JW. Protective factors in posterior urethral
valves. J Urol. 1988;140:993-6.
172 A Teaching Atlas of Case Studies in Diagnostic Imaging
CASE 10

An 11-year-old male presenting with recurrent urinary tract infection and hypertension.

Figure 1  CT angiogram
showing normal renal
arteries but gross
irregularity of both renal
outlines from scarring.
Note delayed function from
a more severely diseased
left kidney, with foci of
cortical calcification

Figure 2  Nephrographic
phase following the
CT angiogram, further
demonstrating the severe
bilateral scarring
Urogenital System Imaging 173

Figures 3A and B  MCUG


showing bilateral grade 5
vesicoureteral reflux B
174 A Teaching Atlas of Case Studies in Diagnostic Imaging

DISCUSSION
Vesicoureteral reflux (VUR) is thought to result from an abnormal entry of the affected ureter
into the bladder trigone. Reflux permits bacteria in the lower urinary tract access to the normally
sterile upper urinary tracts, resulting in pyelonephritis, which when it becomes chronic, leads to
renal cortical scarring and hypertension. VUR is readily diagnosed by MCUG or by radionuclide
cystography.

FURTHER READING
1. Mohanan N, Colhoun E, Puri P. Renal parenchymal damage in intermediate and high grade infantile
vesicoureteral reflux. J Urol. 2008;180(4):1635-8.
2. Riccabona M, Avni FE, et al. Imaging recommendation in Paediatric Uroradiology: Minutes
of the ESPR workshop session on urinary tract infection, fetal hydronephrosis, urinary tract
ultrasonography and voiding cysto-urethrography, Barcelona, Spain. Pediatr Radiol. 2008;38(2):
138-45.
3. Vachvanichsanong P. Urinary tract infection: One lingering effect of childhood kidney disease—
review of literature. J Nephrol. 2007;20(1):21-8.
Urogenital System Imaging 175
CASE 11

A 30-year-old female, known diabetic with right flank pain.

Figure 1  Coronal reformatted


image of contrast CT abdomen.
Nephrographic phase showing
patchy hypodensities in the upper
pole of right kidney

Figure 2  Axial image of same


patient, showing the hypodensities
in the posterior segment of
the right kidney

Diagnosis: Acute pyelonephritis.

DISCUSSION
See under Case 14 (Page 179).
176 A Teaching Atlas of Case Studies in Diagnostic Imaging
CASE 12

A 45-year-old female, known diabetic, presented with left flank pain and dysuria.

Figure 1  Plain CT with


coronal reformation.
The left kidney is enlarged
and shows two subcapsular
gas bubbles in its upper pole

Figure 2  Axial contrast CT


of same patient showing
diffuse reduction in
intensity of the nephrogram
and hypodense foci in
the left kidney
Urogenital System Imaging 177

Figure 3  Sagittal reformatted


image of left kidney showing
patchy hypodensities and a
gas bubble in the upper pole

Figure 4  Coronal postcontrast


CT image of the same patient

Diagnosis: Left pyelonephritis.

DISCUSSION
See under Case 14 (Page 179).
178 A Teaching Atlas of Case Studies in Diagnostic Imaging
CASE 13

A 50-year-old diabetic male with fever, rigor and right flank pain.

Figure 1  Axial postcontrast CT


shows an enlarged right kidney
containing large collections of gas
and showing perirenal fat stranding

Figure 2  Coronal reformatted


image of same patient showing
the enlarged right kidney and
gas collection

Diagnosis: Acute pyelonephritis from gas-forming organism (Emphysematous pyelone­phritis).

DISCUSSION
See under Case 14 (Page 179).
Urogenital System Imaging 179
CASE 14

A 31-year-old female, known diabetic, with bilateral loin pain and dysuria.

Figure 1  Axial contrast


CT showing patchy
hypodensities in
both kidneys

Figure 2  Coronal
reformatted image of same
patient showing the process
to be more severe in
the left kidney
180 A Teaching Atlas of Case Studies in Diagnostic Imaging

Diagnosis: Bilateral acute pyelonephritis.

DISCUSSION
Acute pyelonephritis typically presents with dysuria, frequency, flank pain, with or without systemic
symptoms such as fever and vomiting. Costovertebral angle tenderness may be elicited, and
leukocytosis may or may not be present. Because of the diminished immune response in diabetics,
the effects of acute pyelonephritis tend to be more severe in them.

FURTHER READING
1. Pontin AR, Barnes RD, Joffe J, Kahn D. Emphysematous pyelonephritis in diabetic patients. Br J
Urol. 1995;75(1):71-4.
2. Shen Y, Brom MA. Renal imaging in pyelonephritis. Nephrology. 2004;9(1):22-5.
Urogenital System Imaging 181
CASE 15

A 28-year-old male presented with a 15-day history of fever, dysuria and urinary retention.

Figure 1  T1w postcontrast


MRI showing a grossly
enlarged, profusely
enhancing prostate
containing multiple
loculated cystic areas

Figure 2  T2w image


demonstrating the enlarged
prostate and cystic changes
182 A Teaching Atlas of Case Studies in Diagnostic Imaging

Figure 3  T1w postcontrast


image at a different level
from Figure 1, showing a
large cystic area in
the prostate, with broken
down septa

Figure 4  T1w axial


postcontrast image of
the same patient
Urogenital System Imaging 183

Diagnosis: Prostatic abscess.

DISCUSSION
Prostatic abscess is a complication of acute prostatitis. It is a rare entity and affected patients present
with fever, dysuria, frequency, urinary retention and perineal pain. It is frequently associated with
diabetics and immunocompromised individuals. E. coli is the common offending organism, but
Staphylococcus aureus has also been isolated. Surgical drainage is usually mandatory.

FURTHER READING
1. Baradkar VP, Mathur M, Kumar S. Prostatic abscess by Staphylococcus aureus in a diabetic patient.
Ind J Med Micro. 2008;26(4):395-7.
2. Jacobsen JD, Kvist E. Prostatic abscess: A review of literature and a presentation of 5 cases. Scand J
Urol Nephrol. 1993;27:281-4.
3. Meares MS Jr. Prostatic abscess. J Urol. 1996;129:1281-2.
184 A Teaching Atlas of Case Studies in Diagnostic Imaging
CASE 16

A 17-year-old girl was discovered on routine medical examination to have a blood pressure of
150/100 mm Hg. CT angiogram was performed to exclude a renovascular cause, which confirmed
fibromuscular dysplasia. She was treated with balloon angioplasty, with excellent results, her BP
returning to normal.

Figure 1  CT angiogram
showing a beaded
narrowing of the
midportion of the right
renal artery, consistent with
fibromuscular dysplasia.
Note that the right kidney
is smaller than the left, has
smooth outlines, and shows
a less dense nephrogram

DISCUSSION
Fibromuscular dysplasia (FMD) is inherited as an autosomal dominant disorder. It is the most
common cause of renovascular hypertension in young patients. The narrowing tends to occur in
the middle, and less commonly, in the distal parts of the renal artery, sparing the proximal part.
The stenosis is due to tunica media fibroplasia in the vast majority of cases, and it activates the
renin-angiotensin mechanism, leading to the development of hypertension. As in this case, 98%
of patients respond positively to balloon angioplasty. FMD has also been described in the carotid,
subclavian, axillary, iliac, femoral, popliteal and mesenteric arteries.

FURTHER READING
1. Olin JW, Pierce M. Contemporary manage­ment of fibromuscular dysplasia. Curr Opin Cardiol.
2008;23(6):527-36.
2. Slovut DP, Olin JW. Fibromuscular dysplasia. New Eng J Med. 350:1862-71.
Urogenital System Imaging 185
CASE 17A

Adult male presenting with acute left loin pain and microscopic hematuria.

Figure 1  Control film of


intravenous urogram (IVU)
shows an opacity opposite
the left transverse
process of L3, suggesting
a ureteric calculus

Figure 2  Fifteen-minute
film showing normal
function from the right
kidney with normal
pelvicalyces and ureter.
A delayed nephrogram is
present on the left
186 A Teaching Atlas of Case Studies in Diagnostic Imaging

Figure 3  Two-hour film


confirms a left ureteric
calculus, seen as a filling
defect and producing left
ureteric and pelvicalyceal
dilatation from obstruction

DISCUSSION
See under Case 17C (Page 188).
Urogenital System Imaging 187
CASE 17B

A 45-year-old male presenting with bilateral loin pain.

Figure 1  Kidney, Ureter


and Bladder (KUB) X-ray
showing large bilateral
staghorn calculi

DISCUSSION
See under Case 17C (Page 188).
188 A Teaching Atlas of Case Studies in Diagnostic Imaging
CASE 17C

A 29-year-old male presenting with chronic left flank pain.

Figures 1A and B  (Coronal)


and (Sagittal) reformatted
plain CT abdomen showing
a giant left ureteric calculus
producing gross hydroureter
B and hydronephrosis
Urogenital System Imaging 189

Figure 2  (Axial CT) shows


the calculus measuring
about 30 mm in diameter

DISCUSSION
Sixty percent of urinary calculi are radiopaque on plain X-ray examinations. Calcium phos­phate
calculi are most dense, while uric acid calculi are radiolucent. Urinary calculi have an 80% male
predilection compared with females. The role of intravenous urogram in the management of urinary
calculi is mainly to demonstrate urinary function and anatomy.
Noncontrast CT is now the gold standard for detecting almost all cases of urolithiasis. While
ultrasonography may demonstrate back pressure effects of the calculi, it is poor in detecting ureteric
calculi, except when located in the pelviureteric or vesicoureteric junctions.
Staghorn calculi are usually composed of a struvite-carbonate-apatite matrix, and are thought
to be induced by infection, especially by urea splitting organisms, and can grow rapidly within a
period of weeks or months.

FURTHER READING
1. Duty B, Okhunov Z, Smith A, Okeke Z. The debate over percutaneous nephrolithotomy positioning:
A comprehensive review. J Urol. 2011;186(1):20-5.
2. John N, Cooper B, Robertson W, et al. An update and practical guide in Renal stone management.
Nephron Clin Pract. 2010;116(3):159-71.
3. Smith RC, Varanelli M. Diagnosis of acute ureterolithiasis: CT is truth. AJR Am J Roentgenol.
2000;175(1):3-6.
190 A Teaching Atlas of Case Studies in Diagnostic Imaging
CASE 18

A 28-year-old male presenting with acute left flank pain and hematuria. A large left ureteric calculus
was seen, and patient had treatment with lithotripsy.

Figure 1  Coronal reformatted


plain CT urogram showing a
calculus, 10.5 mm diameter,
in the upper left ureter

Figure 2  10 days post-


lithotripsy. Coronal image
with contrast showing that
the calculus has disappeared,
but a perirenal urinoma is
now present. Note delayed
left nephrogram
Urogenital System Imaging 191

Figures 3A and B  Axial and


left parasagittal reformatted
images demonstrating
the urinoma B
192 A Teaching Atlas of Case Studies in Diagnostic Imaging

Diagnosis: Subcapsular urinoma complicating lithotripsy.

DISCUSSION
Plain CT is now the gold standard for detecting urinary tract calculi, with a sensitivity of 97% and a
specificity of 96%, and lithotripsy is one of the therapeutic options.
Treatment of ureteric calculi with extracorporeal shortwave lithotripsy (ESWL) has known
complications which include: incomplete stone fragmentation with persistent obstruction, renal
parenchymal injury with subcapsular hematoma and urinoma, renal tubular dysfunction, and
hypertension.

FURTHER READING
1. Alkibay T, Karaoglan U, et al. An unusual complication of extracorporeal shock wave lithotripsy:
Urinoma due to rupture of the renal pelvis. Internat Urol Nephrol. 1992;24(1):11-4.
2. Chen MYM, Scharling ES, Zagoria RJ, et al. CT diagnosis of acute flank pain from urolithiasis. Semin
Ultrasound CT MRI. 2002;21:2-19.
3. Scolavikos A, Alivizatos G, Rosette J. Extracorporeal Shock Wave Lithotripsy 25 years Later:
Complications and their prevention. Europ Urol. 2006;50:980-90.
Urogenital System Imaging 193
CASE 19

A 36-year-old male with end stage kidneys had renal transplant.

Figure 1  Coronal
reformatted contrast CT
showing severely contracted
kidneys with only a thin rim
of cortex

Figure 2  Parasagittal
reformatted image showing
a functioning donor kidney
and a large subcutaneous
collection with marginal
enhancement
194 A Teaching Atlas of Case Studies in Diagnostic Imaging

Figure 3  Axial image


showing the large
subcutaneous collection
as well as a right iliac fossa
collection. Note contrast in
the bladder, confirming a
functioning donor kidney

Diagnosis: Postrenal transplant lymphocele.

DISCUSSION
Lymphoceles are estimated to occur in about 10% of renal transplant patients, and are considered
due to transection of lymphatics during the procedure. They are often asymptomatic but may
become infected and require drainage.
Differential diagnosis includes urinoma or hematoma.

FURTHER READING
1. Derweesh IH, Ismail HR, et al. Intraoperative placing of drains decrease the incidence of lymphocele
and deep vein thrombosis after renal transplantation. Br J Urol Int. 2008;101(11):1415-9.
2. Gupta RS, Niranjan J, Srivastava A, Kumar A. Lymphocele following renal transplantation:
Comparison of open surgical and laparoscopic deroofing. Ind J Urol. 2001;18(1): 36-41.
3. Khauli RB, Stoff JS, et al. Post transplant lymphoceles: A critical look into the risk factors,
pathophysiology, and management. J Urol. 1993;150:22-6.
Urogenital System Imaging 195
CASE 20

A 20-year-old unmarried female presented with acute lower abdominal pain and dizziness.
There was no amenorrhea or fever, but she was pale and hypotensive. Abdominal ultrasound
showed a large left renal mass with a prominent hyperechoic hilum. There was associated massive
free intraperitoneal and left perirenal fluid collection consistent with hemorrhage.

Figure 1  Ultrasound of
left kidney showing a large
hyperechoic hilum from the fat
component. There is also a large
perirenal fluid collection

Figure 2  Transverse and sagittal


pelvic ultrasound images
showing large collections of
fluid (FL) around the uterus (UT)
and urinary bladder (UB)
196 A Teaching Atlas of Case Studies in Diagnostic Imaging

Figure 3  Plain axial CT


showing an enlarged
left kidney with multiple
hypodense foci indicating
the presence of fat. Note the
perirenal hematoma

Figure 4  Axial CT
postcontrast arterial phase
showing a rich vascular
enhancement of the mass.
A diagnosis of bleeding
angiomyolipoma was
made and confirmed at
laparotomy. Patient had
nephrectomy
Urogenital System Imaging 197

Figure 5  Axial CT
postcontrast delayed phase
showing a small rim of
functioning renal tissue

Figure 6  Coronal
reformatted postcontrast
CT showing the vascular
mass occupying most of the
left kidney and sparing the
upper pole
198 A Teaching Atlas of Case Studies in Diagnostic Imaging

DISCUSSION
Angiomyolipomas are the most common benign tumors of the kidney. They occur sporadically
in 80–90% of cases, and the rest are associated with tuberous sclerosis. As the name implies, they
are composed of blood vessels, immature smooth muscle cells and fat cells. The dilated blood
vessels can rupture, leading to massive hemorrhage, which may be managed by embolization and/
or nephrectomy. As in this case, the fat content is of major assistance in the diagnosis. With the
detection of fat in the tumor on CT, a confident preoperative diagnosis of angiomyolipoma can be
made.
Renal cell carcinoma remains an important differential, as fat has been reported in some
cases of renal cell carcinoma. Also, cases of malignant transformation have been reported in
angiomyolipoma.

FURTHER READING
1. Ashebu SD, Dahniya MH, Elshebiny YH, Varro J, Al-khawari H. Giant bleeding renal angiomyoli­
poma: Diagnosis and management. Australasian Radiol. 2002;46:115-8.
2. Chen SS, Lin AT, Chen KK, Chan LS. Renal angiomyolipoma: Experience of 20 years in Taiwan.
Eur Urol. 1997;32:175-8.
3. Takahashi N, Kitahara R, Hishimoto Y, Ohguro A, Hashimoto Y, Suzuki T. Malignant transformation
of renal angiomyolipoma. Int J Urol. 2003;10(5):271-3.
Urogenital System Imaging 199
CASE 21

A 52-year-old male presented with hematuria.

Figure 1  Coronal
reformatted postcontrast
CT abdomen showing a
mass expanding the upper
pole of right kidney with
scattered hypodensities,
presumably from necrosis

Figure 2  Right parasagittal


image showing that
the mass has broken
through the renal capsule
and invaded the liver
200 A Teaching Atlas of Case Studies in Diagnostic Imaging

Figure 3  There are


hypodense hepatic lesions
indicative of metastases

Figure 4  CT chest shows


multiple pulmonary
metastases
Urogenital System Imaging 201

Figures 5A and B  CT coronal


and sagittal reformatted
images showing partial lytic
destruction of T4 vertebra B

Diagnosis: Metastatic renal cell carcinoma.

DISCUSSION
See under Case 22 (Page 202).
202 A Teaching Atlas of Case Studies in Diagnostic Imaging
CASE 22

A 52-year-old man presented with pain in the left hip and difficulty in walking. Plain X-ray of the
pelvis showed a lytic left juxta-acetabular lesion. Further investigation revealed an incidental left
renal mass.

Figure 1  Plain X-ray pelvis


showing a large lytic lesion
in the left ilium adjoining
the acetabulum

Figure 2  Axial postcontrast CT


shows a large mass occupying
most of the left kidney, sparing
the upper pole
Urogenital System Imaging 203

Figure 3  Coronal
reformatted CT image
showing hypodense foci in
the mass, consistent
with necrosis. Note a simple
cortical cyst in the
right kidney

Figure 4  Axial CT showing


the large lytic area
in the left ilium
204 A Teaching Atlas of Case Studies in Diagnostic Imaging

Diagnosis: Left renal cell carcinoma with hip metastasis.

DISCUSSION
Renal cell carcinoma (RCC) is the most common malignant tumor in adults, occurring most
frequently after the age of 55 years and with a male preponderance in the ratio 1.6:1. RCC is derived
from renal epithelium.
Imaging is essential in its diagnosis and staging. Although most imaging parameters are
contributory, CT is usually preferred. With contrast CT, the tumor enhancement is less than that
of the normal residual renal tissue and may contain irregular hypodensities, representing foci of
tumor necrosis. Although the most common presentations in RCC are hematuria, flank pain and
a palpable mass, it may not manifest clinically until distant metastases have developed, as in this
case.

FURTHER READING
1. Bach AM, Zhang J. Contemporary radiologic imaging of renal cortical tumors. Urol Clin North Am.
2008;35:593-604.
2. Brufau BP, Cerqueda CS, Villalba LB, et al. Metastatic renal cell carcinomas: Radio­logic findings
and assessment of response to targeted antiangiogenic therapy by using multidetector CT.
Radiographics. 2013;33(6):1691-716.
3. Ng CS, Wood CG, Silverman PM, et al. Renal cell carcinoma: diagnosis, staging and surveillance.
Am J Roentgenol. 2008;191(4):1220-32.
Urogenital System Imaging 205
CASE 23

A 35-year-old Egyptian male presented with frank hematuria and weight loss.

Figure 1  Venous phase


of triphasic contrast CT
abdomen showing an
enhancing irregular right
vesical wall mass.
Note stranding of the
perivesical fat planes

Figure 2  Postcontrast
CT showing delayed
nephrogram on the right
and pelvicalyceal distension
due to distal ureteric
obstruction by
the vesical mass
206 A Teaching Atlas of Case Studies in Diagnostic Imaging

Figure 3  CT chest, lung


window showing an opaque
left hemithorax and a right
pleural nodule

Figure 4  CT chest,
mediastinal window
showing a massive left
pleural fluid collection
Urogenital System Imaging 207

Figure 5  Contrast MRI of


same patient showing
enhancement of
the vesical mass

Diagnosis: Bladder carcinoma with right obstructive uropathy and pulmonary and pleural
metastases.

DISCUSSION
See under Case 24 (Page 208).
208 A Teaching Atlas of Case Studies in Diagnostic Imaging
CASE 24

A 58-year-old male presented with hematuria.

Figure 1  MRI pelvis


with contrast showing
an enhancing, irregular
mass in the bladder base,
extending to its left wall.
Note infiltration of the left
perivesical fat

Figure 2  T2w sequence


showing the left vesical
wall mass. Note a dilated
left ureter (arrow) due to
vesico-ureteric junction
obstruction
Urogenital System Imaging 209

Figure 3  Sagitttal T1w with


contrast demonstrating
the vesical mass

Diagnosis: Carcinoma urinary bladder.

DISCUSSION
Most cases of bladder carcinoma present with gross hematuria, dysuria or frequency. There is a
high association with cigarette smoking and with vesical schistosomiasis.
Histological types in bladder neoplasm are transitional cell carcinoma, constituting 90%
of all cases, squamous cell carcinoma (most frequently associated with schistosomiasis),
adenocarcinoma, small cell carcinoma, and sarcoma.
In Egypt, where vesical schistosomiasis is endemic, bladder carcinoma is the most common
malignancy, and tends to occur at a relatively early age.
The gold standard for diagnosis is cystoscopy with biopsy. Apart from the diagnostic features of
an irregular vesical wall thickening and possible ureteric obstruction, as in the cases presented, the
role of radiology is in staging.

FURTHER READING
1. Boffetta P. Tobacco smoking and the risk of bladder cancer. Scand J Urol Suppl. 2008;42:45-54.
2. Heyns CF, van der Merwe A. Bladder cancer in Africa. Can J Urol. 2008;15(1):3899-908.
3. Mungadi IA, Malami SA. Urinary bladder cancer and schistosomiasis in North-Western Nigeria.
W Afr J Med. 2007;26(3):226-9.
210 A Teaching Atlas of Case Studies in Diagnostic Imaging
CASE 25

A 24-year-old male with a left undescended testis.

Figure 1  T1w axial MRI


image showing the testis in
the left inguinal canal

Figure 2  T2w image


demonstrating the left testis
Urogenital System Imaging 211

Figure 3  T1w postcontrast


coronal image showing
the left testis with
intense enhancement

DISCUSSION
See under Case 26 (Page 212).
212 A Teaching Atlas of Case Studies in Diagnostic Imaging
CASE 26

A 17-year-old male with right undescended testis.

Figure 1  T1w axial MRI


showing the testis in
the right groin

Figure 2  T2w axial image


showing the testis
Urogenital System Imaging 213

Figure 3  T1w
postgadolinium coronal
image demonstrates
intense enhancement of
the right testis

DISCUSSION
Undescended testis (Cryptorchidism) occurs in about 3% of full term and 30% of premature boys.
In 80% of cases, the testis will descend by the end of the first year of life. Because of a high risk
for developing testicular cancer, especially in the intra-abdominal testes, it is necessary to locate
and correct the anomaly, either by orchidopexy or orchidectomy. Ultrasono­graphy and MRI are
important tools in the location of the undescended testis.

FURTHER READING
1. Docimo SG. Testicular descent and ascent in the first year of life. Urology. 1996;48:458-60.
2. Kantarci M, Doganay S, Yalcin A, et al. Diagnostic performance of diffusion­weighted MRI in the
detection of nonpalpa­ble undescended testes: Comparison with conventional MRI and surgical
findings. AJR Am J Roentgenol. 2010;195(4):268-73.
214 A Teaching Atlas of Case Studies in Diagnostic Imaging
CASE 27

A 4-year-old girl presented with abdominal pain. Abdominal ultrasound showed a left renal mass
with features suggesting neoplasm. CT was done to evaluate the extent of the tumor.

Figure 1  Axial image of


contrast CT showing a large
hypodense left intrarenal
mass. Note how the normal
renal tissue is stretched
around the mass—the
“claw sign”, confirming
the intrarenal nature of
the mass

Diagnosis: Wilm’s tumor.

DISCUSSION
Wilm’s tumor (nephroblastoma) is the most common abdominal malignancy in children, with
a peak incidence between 2 and 3 years. It is commonly unilateral, but can be bilateral in 5–10%
of cases.
It most commonly presents as an abdominal mass, but may present with hematuria, pain, or
hypertension.
Radiological assessment is usually initially by ultrasonography, but CT is essential for staging,
and MRI has the advantage of better tissue characterization.
The prognosis is good, with a survival rate of between 80–90%.

FURTHER READING
1. Darge K, Anupindi SA, Jamillo D. MR Imaging of the abdomen in infants, children and adolescents.
Radiology. 2011;261:12-29.
2. Dome JS, Cotton CA, Perlman EJ, et al. Treatment of anaplastic histology Wilm’s tumor: Results
from the fifth National Wilm’s Tumor Study. J Clin Oncol. 2006;24(15):2352-8.
3. Lowe LH, Ismani BH, Heller RM, et al. Pediatric renal masses: Wilm’s tumor and beyond.
Radiographics. 2000;20:1585-603.
Urogenital System Imaging 215
CASE 28

A 62-year-old female presented with frank hematuria.

Figure 1  Contrast-
enhanced CT showing
an enlarged, poor
functioning right
kidney with hypodense
foci. Hypodensities
are also noted in the
liver, suggesting renal
malignancy with hepatic
metastases

Figure 2  CT chest showing


bilateral pulmonary
metastases
216 A Teaching Atlas of Case Studies in Diagnostic Imaging

Figures 3A and B  (A) Axial


and (B) Coronal CT images
of the bladder containing
tubular filling defects,
consistent with a positive
B “spaghetti sign”
Urogenital System Imaging 217

DISCUSSION
Tubular filling defects within the urinary bladder in a patient with hematuria represent blood clot
casts of the ureter extruded into the bladder, and have been termed the “spaghetti sign”. When
the sign is positive, it indicates that the source of hematuria is outside the bladder or perivesical
areas. The sign was first described on excretory urogram, but has also been observed on CT and
ultrasonography.

FURTHER READING
1. Dyer RB, Chen MY, Zagorva RJ. Classic signs in uroradiology. Radiographics. 2004;24:247-80.
2. Eisenberg RL. The “spaghetti sign” Atlas of signs in Radiology, Chapter 2 JB Lippincott Co.
Philadelphia; 1985. p. 169.
3. Komolafe F. The “spaghetti sign”: An uncommon radiologic sign of upper urinary tract hemorrhage.
AJR Am J Roentgenol. 1981;137:1062.
218 A Teaching Atlas of Case Studies in Diagnostic Imaging
CASE 29

A 55-year-old male presenting with acute right flank pain radiating to the groin.

Figure 1  Intravenous
urogram shows right
hydronephrosis and
hydroureter, due to a
midureteric calcular
obstruction. There is an
extrarenal smearing of
contrast over the right
upper pole

Figure 2  Postcontrast
CT confirms extrarenal
collection of contrast,
indicating calyceal rupture
Urogenital System Imaging 219

DISCUSSION
Most cases of spontaneous calyceal rupture result from increased pressure in the renal pelvis
from ureteric obstruction. Obstruction is due to calculus in 75% of cases, but may be extrinsic
compression, especially from abdominal tumors in the line of the ureter. Urinomas may also be
post-traumatic or iatrogenic.
Most urinomas remain within Gerota’s fascia, but can extend medially into the para-aortic space
or superiorly into the mediastinum. Small urinomas usually resolve spontaneously, but when
large, they may require percutaneous drainage, stent placement in the ureter and addressing the
underlying cause of obstruction. This case was managed conservatively.

FURTHER READING
1. Ashebu SD, Dahniya MH, Aduh P, et al. Rupture of the renal pelvis of a ureteropelvic junction
hydronephrosis after blunt abdominal trauma. Australas Radiol. 2004;48:256-8.
2. Gershman B, Kulkarni N, Schani DV, Eisner BH. Causes of renal forniceal rupture. BJU Internat.
2011;108:1909-11.
220 A Teaching Atlas of Case Studies in Diagnostic Imaging
CASE 30

A 54-year-old man presenting with difficulty in micturition of several years duration. He was thought
to have prostate hypertrophy or urethral stricture and was referred for retrograde urethrocystogram.

Figure 1  A preliminary
pelvic X-ray showing
two large vesical calculi
(Courtesy: Dr Adekunle
Abdulkadir)

DISCUSSION
Ninety-five percent of vesical calculi occur in men, and most are a result of bladder neck obstruction.
They also result from urinary infection, complication of (iatrogenic) foreign bodies, and stasis
in neurogenic bladders. Vesical calculi usually consist of calcium oxalate, but may also contain
large amounts of uric acid, especially in association with gout. Chronic vesical calculi have been
associated with squamous cell carcinoma of the bladder.

FURTHER READING
1. Ho KLV, Segura JW. Lower urinary tract calculi. In: Wein AJ (Ed). Campbell-Walsh Urology, Chap 84.
9th edition, Philadelphia, Saunders Elsevier; 2007.
2. Schwartz BF, Stroller MZ. The vesical calculus. Urol Clin North Am. 2000;27(2):333-46.
Urogenital System Imaging 221
CASE 31

A 16-year-old girl presented with a right flank mass, headache and palpitations. She was found to
be hypertensive. Abdominal ultrasound revealed a mass of heterogenous echo pattern in the right
hypochondrium, which was difficult to distinguish from the liver. Abdominal CT was done.

Figure 1  Axial plain CT


abdomen showing a soft
tissue mass with central
hypodense foci.
Note the clear line of
cleavage between the mass
and the liver

Figure 2A
222 A Teaching Atlas of Case Studies in Diagnostic Imaging

Figure 2B
Figures 2A and B  Contrast-
enhanced coronal
reformatted CT images
showing a large, vascular
suprarenal mass supplied
by arteries derived from the
aorta and right renal artery.
The right kidney is displaced
inferiorly, and the right
renal artery is stretched.
Note central hypodense
foci, probably from central
necrosis

Figure 3  Sagittal
reformatted image in
the venous phase. Note the
enhancement with contrast
and the clear separation of
the mass from the liver
Urogenital System Imaging 223

Diagnosis: Pheochromocytoma.

DISCUSSION
Pheochromocytoma is a neuroendocrine tumor of the adrenal medulla. It secretes excessive
amounts of epinephrine and norepinephrine. About 10% of patients are children and, as in this
case, present with hypertension and its effects. Surgical resection of the mass is curative.
Paragangliomas are extra-adrenal pheo­chromocytomas which originate in the ganglia of the
sympathetic nervous system.

FURTHER READING
1. Bessell-Browne R, O’Malley ME. CT of pheochromocytoma and paraganglioma: Risk of adverse
events with IV administra­tion of nonionic contrast material. AJR Am J Roentgenol. 2007;188(4):
970-4.
2. Park BK, Kim C, Kwon GY, Kim JH. Reevaluation of pheochromocytomas on delayed contrast-
enhanced CT: Washout enhancement and other imaging features. Eur Radiol. 2007;17(11):2804-9.
3. Waguespack SG, Rich T, Grubbs E, et al. A current review of the etiology, diagnosis and treatment
of pediatric pheochromocy­toma and paraganglioma. J Clin Endocrinol Metab. 2010;95(5):2023-37.
224 A Teaching Atlas of Case Studies in Diagnostic Imaging
CASE 32

A 52-year-old man presenting with a painless right flank mass, slowly enlarging over a period of
one year. Ultrasound scan showed a complex but largely hyperechoic mass interposed between
the liver and the right kidney, displacing the latter inferiorly.

Figures 1A and B  (A) Contrast-


enhanced CT coronal
reformatted, and (B) Axial
images showing a large right
adrenal mass producing
extrinsic compression of the
liver and displacing the right
kidney inferiorly. Note that the
B mass is largely of fat density
Urogenital System Imaging 225

Figure 2  Axial T1-weighted


MRI showing most of the
mass to be of high signal
intensity, consistent with a
high lipid content

Figure 3  Fat saturated


image showing that
the previously hyperintense
areas of the mass
are now hypointense.
The nonsuppressed
foci represent the
myeloproliferative areas
226 A Teaching Atlas of Case Studies in Diagnostic Imaging

Diagnosis: Adrenal myelolipoma.

DISCUSSION
Adrenal myelolipomas are rare benign tumors comprising mature adipose tissues as well as
hemopoietic tissues. They are usually symptom free and are discovered incidentally when patients
are being examined for other reasons. The diagnosis of adrenal myelolipoma is readily made because
of the high fat content, hyper-echoic on ultrasonography and low attenuation on CT. On MRI, the
fat content is hyperintense on T1w, intermediate on T2w and shows low signals on fat saturation
technique. They may occasionally contain punctate calcifications. Making a correct diagnosis
may be difficult if the fat content is low, with the mass consisting mainly of hematopoietic tissue.
Biopsy may be required in such situations to exclude adrenal neoplasm.

FURTHER READING

1. Boland GWL, Blake MA, Hahn PF, Mayo-Smith WW. Incidental adrenal lesions: Principles,
techniques, and algorithms for imaging characterization. Radiology. 2008;249(3):756-75.
2. Cyran KM, Kenney PJ, Memel DS, Yacoub I. Adrenal myelolipoma. AJR Am J Roentgenol.
1996;166(2):395-400.
3. Yip L, Tublin ME, Facone JA, Nordman CR, Stang MT, et al. The Adrenal mass: Correlation of
histopathology with imaging. Ann Surg Oncol. 2010;17(3):846-52.
Urogenital System Imaging 227

CASE 33

A 12-year-old premenarchal girl presented with acute, crampy lower abdominal pain and dysuria,
with several episodes of vomiting. She had no similar symptoms previously. Abdominal ultrasound
showed a large cystic mass arising from the pelvis, posterior to the urinary bladder (see Fig. 1).
No other abnormality was seen. A diagnosis of hematocol­pometra was made. This was confirmed
at hymenotomy, which provided immediate relief to her symptoms.

Figure 1  Transabdominal
ultrasound showing a
438 mL cystic mass
posterior to the bladder,
with internal echogenic
debris (Composite sagittal
and transverse images).
Note how the bladder is
stretched around
the cystic mass

DISCUSSION
Hematocolpometra results from the inability of the menses to exit from the vagina due to the presence
of an imperforate hymen. The gross distension of the vagina may interfere with micturition and
even cause constipation. The symptoms of hematocolpometra resolve readily after hymenotomy,
but spontaneous rupture has been reported. Hematometra alone without hematocolpos can occur
from cervical canal obstruction due to fibroids or adhesions.

FURTHER READING
1. Drakonaki EE, Tritou I, Pitsoulis G, et al. Hematocolpometra due to an imperforate hymen
presenting with back pain: Sono­graphic diagnosis. JUM. 2010, 29(2):321-2.
2. Kurdoglu Z, Kurdoglu M, Kucukaydin Z. Spontaneous rupture of the imperforate hymen in an
adolescent girl with he­matocolpometra. ISRN Obstet Gynecol. 2011;2011:520304.
3. Stone SM, Alexander JL. Imperforate hymen with hematocolpometra. N Engl J Med. 2004;351(7):e6.
228 A Teaching Atlas of Case Studies in Diagnostic Imaging
CASE 34

A 72-year-old woman presenting with left lower abdominal discomfort.

Figure 1  Coronal T1w MRI


showing a large, rounded
left pelvic mass with high
signal intensity

Figure 2  On fat saturation


sequence, most of the mass
has become hypointense,
indicating a high lipid
content. An oval mass
with mixed signal intensity
is revealed in its inferior
aspect
Urogenital System Imaging 229

Figure 3  Sagittal T1w fat


saturated postcontrast
study shows peripheral
enhancement of
the internal mass

DISCUSSION
See under Case 35 (Page 230).
230 A Teaching Atlas of Case Studies in Diagnostic Imaging
CASE 35

A 35-year-old woman presented with a three week history of lower abdominal pain. She has had
two normal deliveries, and her menses were regular. Clinical examination revealed a nontender
suprapubic mass. Ultrasonography was done (not shown), followed by CT.

Figure 1  Coronal reformatted


CT shows a lobulated,
hypodense mass with a
craniocaudal diameter of
12.8 cm. Note a tooth-like
opacity in its roof

Figure 2  CT of the pelvis


showing the largely fat-
containing mass with AP
diameter of 10.9 cm. Note a fleck
of calcification on its right wall
Urogenital System Imaging 231

Diagnosis: Ovarian teratomas.

DISCUSSION
Teratomas are encapsulated germ cell tumors with tissue components derived from all three germ
layers, but they may be monodermal. They range from well-differentiated (mature) cystic lesions
to those which are solid and malignant (immature). They can contain recognizable structures such
as hair, bone, teeth, sebaceous material and neural tissue. Ovarian teratomas may cause clinical
symptoms if they undergo torsion, rupture, if large enough to produce mass effects, or if they
undergo malignant transformation, (which occurs in 1–2% of cases). The terms “teratoma” and
“dermoid” are currently used interchangeably.
Fat-saturated sequences of MRI are invaluable in the diagnosis of ovarian teratomas, as their
lipid contents become hypointense from their hyperintense appearance on regular T1w sequence.

FURTHER READING
1. Outwater EK, Siegelman ES, Hunt JL. Ovarian teratomas: Tumor type and imaging characteristics.
Radiographics. 2001;21:475-90.
2. Stany MP, Hamilton CA. Benign disorders of the ovary. Obstet Gynecol Clin North Am.
2008;35(2):271-84.
232 A Teaching Atlas of Case Studies in Diagnostic Imaging
CASE 36

A 32-year-old woman with history of recurrent abortion. In this pregnancy, she presented with
lower abdominal pain in the first trimester. Ultrasound confirmed a live active fetus.

Figure 1  Coronal T2w MRI


showing a septum arising
from the superior uterine wall.
Note the fetus in transverse
lie straddling both chambers
of the uterus

Figure 2  Axial T2w


demonstrating the septum
and fetus
Urogenital System Imaging 233

Figure 3  Left parasagittal


T2w image showing
the crossed fetal lower limbs

DISCUSSION
Septate uterus is a congenital anomaly resulting from failure of resorption of the partition between
the two fused müllerian ducts. The septum may extend into the vagina, resulting in a double vagina.
Septate uterus should be distinguished from a bicornuate uterus. In the former, the müllerian
ducts fuse completely to form a single uterine body and only the lumen is partitioned, whereas in
the latter, there is incomplete fusion of the müllerian ducts, resulting in two uterine bodies. Septate
uterus is associated with recurrent abortion and fetal malpresentation.

FURTHER READING
1. Heinonen PK. Complete septate uterus with longitudinal vaginal septum. Fertil Steril.
2006;85(3):700-05.
2. Propst AM, Hill JA. Anatomic factors associated with recurrent pregnancy loss. Semn Reprod Med.
2000;18(4):341-50.
234 A Teaching Atlas of Case Studies in Diagnostic Imaging
CASE 37

A 50-year-old man presenting with acute groin pain, scrotal swelling, tenderness, and abrasions. He
was not diabetic. Plain X-ray of the pelvis (see Fig. 1) showed gas in the soft tissues. A diagnosis of
Fournier’s gangrene was made.

Figure 1  Plain X-ray of


the groin. Note scrotal
swelling and soft tissue
emphysema

DISCUSSION
See under Case 38 (Page 235).
Urogenital System Imaging 235
CASE 38

A 52-year-old known diabetic with lower abdo­minal pain, dysuria and scrotal swelling of one
day duration. Ultrasonography (not included) showed hyperechoic lesions with “dirty” posterior
acoustic shadowing in the scrotal soft tissues, highly suggestive of emphysema. Fournier’s gangrene
was suggested. CT was advised to determine the level of extension.

Figures 1A and B  CT pelvis


with (A) Axial image, and
(B) Parasagittal reformatted
image, showing extensive gas
in the scrotal soft-tissues.
The testes were not involved.
Note distension of
the urinary bladder B
236 A Teaching Atlas of Case Studies in Diagnostic Imaging

DISCUSSION
Fournier’s gangrene is a necrotizing fasciitis, which is a fulminant infection of the soft tissues of
the male genitalia, perineum and abdominal wall. Both aerobic and anaerobic bacteria may be
involved. Almost 50% of affected patients are diabetics, but there is an increased disposition to
Fournier’s gangrene in patients with medical immunosuppression, e.g. patients on steroid therapy
or cancer chemotherapy. The management usually consists of debridement and strong intravenous
antibiotics. The principal role of CT is to determine the extent of infection by assessing the spread
of the emphysema.

FURTHER READING
1. Levenson RB, Singh AK, Novelline RA. Fournier Gangrene: Role of imaging. Radiographics.
2008;28:519-28.
2. Tahmaz L, Erdemir F, Kibar Y, et al. Fournier’s gangrene: Report of 33 cases and a review of
the literature. Int J Urol. 2006;13(7):960-7.
3. Vaz I. Fournier gangrene. Trop Doct. 2006;36(4):203-4.
Urogenital System Imaging 237
CASE 39

A 39-year-old male with infertility.

Figure 1  Prominent dilated


veins in the pampiniform
plexus on gray scale
ultrasound scan

Figure 2  Pulsed Doppler


scan showing reversal of
flow demonstrated as flow
below the baseline and
above the baseline during
and after Valsalva maneuver,
respectively
238 A Teaching Atlas of Case Studies in Diagnostic Imaging

A B
Figures 3A and B  Color Doppler scan showing reversal of flow as blue and red colors (A) During, and (B) On
release of Valsalva maneuver, respectively. Features are those of scrotal varicocele (Courtesy: Dr Musa Tabari)

DISCUSSION
A scrotal varicocele is an abnormal degree of venous dilatation in the pampiniform plexus, affecting
approximately 15% of men. It can present with scrotal pain and swelling, or in association with male
subfertility.
Idiopathic varicoceles are more common on the left than on the right, presumably because the
left spermatic vein enters perpendicular to the left renal vein and the right enters obliquely into the
inferior vena cava, which appears to reduce retrograde flow on the right side. Less frequent causes
of varicoceles include compression of the renal vein sometimes by tumor, an aberrant renal vein or
an obstructed renal vein.
Ultrasound is now the most frequently used method in the diagnosis of scrotal varicocele and a
high-frequency transducer of at least 7 MHz should be used. The features on gray scale ultrasound
include a prominence of at least two to three veins of the pampiniform plexus, of which one should
have a diameter greater than 2–3 mm in a supine position. Visualization is improved by the Valsalva
maneuver, with color and pulsed Doppler. Doppler sonography can be used to grade venous reflux
as static (grade I), intermittent (grade II), or continuous (grade III).
Venography is considered to be the gold standard in the diagnosis of varicocele, but MRI and
scintigraphy have been used.

FURTHER READING
1. Hulya O, Umit Yener T, Seniha N, Suleyman B, Ali A. Hemodynamic evaluation of varicocele:
The role of scrotal scintigraphy and Doppler ultrasonography in the predic­tion of postoperative
seminal improvement. Ann Nucl Med. 2005;19(7):529-34.
2. Beddy P, Geoghegan, Browne RF, Torreggiani WC. Testicular varicocoeles. Clin Radiol. 2005;60:
1248-55.
Urogenital System Imaging 239
CASE 40

A 35-year-old man who fell from a height and sustained blunt injury to the scrotum 1 week earlier.

Figure 1  Scrotal ultrasound


showing a left peritesti­cular
collection with a lace-like
pattern, consistent with a
subacute hematocele.
Note a normal testicular
echo pattern. Color Doppler
(not shown) demonstrated
a normal testicular vascular
perfusion

DISCUSSION
A hematocele is a collection of blood between the visceral and parietal layers of the tunica vaginalis.
Scrotal injuries can result from blunt, penetrating or degloving (scrotal skin avulsion) trauma.
Electrical burns can also cause injury, however, blunt injuries are the most common, especially
resulting from road traffic accidents and sports. Color flow and duplex Doppler imaging are
essential in the assessment of testicular viability and perfusion.

FURTHER READING
1. Buckley JC, McAninch JW. Use of ultrasonography for the diagnosis of testicular injuries in blunt
scrotal trauma. J Urol. 2006;175(1):175-8.
2. Chandra RV, Dowling RJ, Ulubasoglu M, et al. Rational approach to diagnosis and management of
blunt scrotal trauma. Urology. 2007;70(2):230-4.
3. Deurdulian C, Mittelstaedt CA, Chong NK, Fielding JR. US of acute scrotal trauma: Optimal
technique, imaging findings, and management. Radiographics. 2007;27:357-69.
4 Gastrointestinal
System Imaging

CASE 1

A 2-day-old male baby presented with progressive abdominal distension and nonpassage of
meconium. A plain lateral shoot through abdominal radiograph was done.

Figure 1  There is gaseous


distension of the sigmoid
colon and rectum,
extending almost to the
anal verge. The marked
distension of the rectum
without filling the urinary
tract suggests the absence
of rectourinary fistula.
Note the absence of
the caudal segment of
the sacrum

DISCUSSION
See under Case 2 (Page 241).
Gastrointestinal System Imaging 241
CASE 2

A male baby presented at 3 days of age with progressive abdominal distension, fecaluria
and nonpassage of meconium. A colostomy was performed on him, followed by a pressured
colostography at the age of six months, prior to definitive corrective surgery.

Figure 1  A distal
colostogram showing
contrast-filled sigmoid
and rectum. Note the
termination of rectal
contrast column above
the mid-pelvic cavity and
the filling of the urinary
bladder with contrast via
a recto-urethral fistula
(Courtesy: Dr Adekunle
Abdulkadir)

DISCUSSION
Anorectal malformation (ARM) is a common congenital anomaly with an incidence of 1 in 5,000
live births. Fecaluria and obstructive gastrointestinal symptoms are the earliest diagnostic pointers,
while imperforate anus is the hallmark clinical sign. When associated with genitourinary fistula,
it permits an outlet for fecal matter, and ARM may therefore go unnoticed in the child for years.
The roles of radiological imaging in ARM are to determine the level of obstruction, identify the
presence and location of any associated fistula, and diagnose any associated anomalies. ARM is
classified as high or low depending on whether or not the rectal pouch lies above or below the level
of the puborectalis sling. This level is the M-line and it correlates on the radiograph to the line joining
the ischial spines. Available imaging methods to classify ARM include prone cross-table lateral
242 A Teaching Atlas of Case Studies in Diagnostic Imaging

abdominal radiograph and transperineal US to determine the colon pouch-perineal distance. Fistulae
may be demonstrated by distal colostography, or by retrograde or micturating cystourethrography.
However, MRI has proven to be the single stop modality to answer all the crucial questions such as
level and type of malformation, type of fistula, developmental state of the sphincter muscle complex,
and the presence of associated anomalies.

FURTHER READING
1. Abdulkadir AY, Abdur-Rahman LO, Adesiyun OM. Nonfluoroscopic pressure colostography in
the evaluation of genitourinary fistula of anorectal malformations: Experience in a resource poor
environment. Pediatr Radiol. 2009;39:132-6.
2. Endo M, Hayashi A, Ishihara M, Maie M, et al. Analysis of 1,992 patients with anorectal malformations
over the past two decades in Japan. Steering Committee of Japanese Study Group of Anorectal
Anomalies. J Pediatr Surg. 1999;34:435-41.
3. McHugh K. The role of radiology in children with anorectal anomalies with particular emphasis on
MRI. Eur J Radiol. 1998;26:194-9.
4. Pena A, Hong A. Advances in the manage­ment of anorectal malformations. Am J Surg. 2000;180:
370-6.
Gastrointestinal System Imaging 243
CASE 3

A 1-week-old male presented with lumbar mass and features of partial intestinal obstruction.

Figures 1A and B  Plain


abdominal radiograph and
intravenous urogram.
Note the right kidney among
the hernial contents B

DISCUSSION
See under Case 4 (Page 244).
244 A Teaching Atlas of Case Studies in Diagnostic Imaging
CASE 4

A male baby presented few days after birth with a congenital right lumbar swelling.

Figures 1A and B 
Photograph showing
the right lumbar mass
and a plain radiograph
showing gas-filled herniated
bowel. Note costovertebral
anomalies, with absent
right 7–12th ribs
(Courtesy: Dr Adekunle
Abdulkadir, Dr Luqman
Abdur-Rahman, and Dr Anas
Ismail)
B
Gastrointestinal System Imaging 245

DISCUSSION
Hernias such as inguinal, femoral and umbilical varieties constitute up to 98.5% of external
abdominal hernias. Lumbar and gluteal hernias account for the remaining 1.5%. Lumbar hernia
has a male-to-female prevalence of 3:1 and left-to-right ratio of 2:1. It is acquired in about 80%
of cases. Congenital lumbar hernia is often reported in association with costovertebral anomalies
thought to be due to somatic defects occurring between the 3rd and 5th weeks of embryogenesis.
Other reported associated anomalies include anorectal malformations, sacral dysgenesis, and
genitourinary anomalies. Although the diagnosis can be made clinically, imaging is required to
define associated anomalies, guide management and evaluate treatment and assess complications.

FURTHER READING
1. Aguirre DA, Santosa AC, Casola G, Sirlin CB Abdominal wall hernias: Imaging features,
complications, and diagnostic pitfalls at multi–detector row CT. Radiographics. 2005; 25:1501-20.
2. Akçora B, Temiz A, Babayi it C. A different type of congenital lumbar hernia associated with
the lumbocostovertebral syndrome. J Pediatr Surg. 2008;43:21-3.
3. Cornier AS, Ramirez N, Carlo S, Reiss A. Controversies surrounding Jarcho-Levin syndrome.
Curr Opin Pediatr. 2003;15:614-20.
4. Miller PA, Mezwa DG, Feczko PJ, Jafri ZH, Madrazo BL. Imaging of abdominal hernias. Radiographics.
1995;15,333-47.
246 A Teaching Atlas of Case Studies in Diagnostic Imaging
CASE 5

A 7-day-old premature male infant admitted in the intensive care unit was noted to have progressive
abdominal distension and bloody stool. Abdominal ultrasonography showed free peritoneal fluid.
A KUB was done 6 hours later.

Figure 1  Supine KUB shows a


large pneumoperito­neum with
dilated loops of small and large
bowel

Figure 2  Lateral decubitus


radiograph shows a long
air-fluid level, confirming the
massive pneumoperi­toneum.
Features are those of necrotizing
enterocolitis (NEC)
Gastrointestinal System Imaging 247

DISCUSSION
Necrotizing enterocolitis (NEC) is a condition of idiopathic etiology affecting premature neonates,
usually presenting with feeding intolerance, abdominal distension and bloody stool. It may
progress to bowel perforation and peritonitis. The role of imaging is to establish the diagnosis.
Abdominal ultrasonography may demonstrate intramural gas as well as free peritoneal fluid.
Abdominal radiograph may also show pneumatosis intestinalis, bowel distension or frank
pneumoperitoneum.

FURTHER READING
1. Epelman M, Daneman A, Navarro OM, et al. Necrotizing enterocolitis: Review of state-of-the-art
imaging findings with pathologic correlation. Radiographics. 2007;27(2):285-305.
2. Kosloske AM, Musemeche CA, Ball WS, et al. Necrotizing enterocolitis: Value of radiographic
findings to predict outcome. AJR Am J Roentgenol. 1988;151(4):771-4.
248 A Teaching Atlas of Case Studies in Diagnostic Imaging
CASE 6

A 2-year-old girl presented with abdominal pain. Abdominal ultrasound was first perfor­med,
followed by CT, MRI and MRCP.

Figure 1  Abdominal
ultrasound with Doppler
showing multiple, cystic
nonvascular structures in
the liver

Figure 2  Axial contrast


CT image showing the
nonenhancing cysts
Gastrointestinal System Imaging 249

Figure 3  Sagittal
reformatted contrast-
enhanced CT image
shows several of the
cystic structures to be
communicating

Figure 4  T2w axial


MRI confirms the
communicating cysts
250 A Teaching Atlas of Case Studies in Diagnostic Imaging

Figure 5  MRCP. The


gallbladder and cystic
duct are demonstrated.
At least one of the cysts is
extrahepatic

Diagnosis: Choledochal cyst, probably Todani Type IV.

DISCUSSION
A choledochal cyst is a congenital anomaly with cystic dilatation of the biliary ductal system, with
involvement of the intra- or extrahepatic system or both. Five major types have been described by
Todani et al. Ultrasonography is the usual initial preferred examination, but it may be impossible
to differentiate a choledochal cyst from a hepatic cyst or a pancreatic pseudocyst. As in this case,
CT and MRI will be required for a more definitive diagnosis.

FURTHER READING
1. Irie H, Honda H, Jimi H, et al. Value of MR cholangiopancreatography in evaluating choledochal
cysts. AJR Am J Roentgenol. 1998;171(5):1381-5.
2. Kim SH, Lim JH, Yoon HK, et al. Choledochal cyst: Comparison of MR and conventional
cholangiography. Clin Radiol. 2000;55(5):378-83.
3. Todani T, Watanabe T, Narusue M, et al. Congenital bile duct cysts: Classification, operative
procedures, and review of thirty seven cases including cancer arising from choledochal cyst. Am J
Surg. 1977;134(2):263-9.
Gastrointestinal System Imaging 251
CASE 7

A 59-year-old woman had abdominal ultra­sound examination for vague abdominal pain. Except for
a splenic lesion, no abnormality was demonstrated.

Figures 1A and B  Splenic


ultrasound with Doppler
shows a rounded, well-
defined echogenic mass,
demonstrating some
internal vascular flow.
The features are
characteristic of a
hemangioma B
252 A Teaching Atlas of Case Studies in Diagnostic Imaging

DISCUSSION
Splenic hemangiomas are rare but remain the most common benign neoplasm of the spleen.
They are usually discovered incidentally during ultrasonography performed for other reasons.
Most tend to be small (< 2 cm), and can be single or multiple. In most cases, the blood flow to
splenic hemangiomas is slow, and so they may appear on Doppler to be hypovascular.
Ultrasonography is usually adequate for their diagnosis, sometimes with contrast sonography, but
the equivocal cases should have MRI or CT for further evaluation.
Spontaneous rupture of splenic hemangiomas has been reported in up to 25% of cases, requiring
splenectomy. Malignant degeneration may rarely occur. The differential diagnosis of splenic
hemangiomas include: Littoral cell angioma, hemangiosarcoma and lymphomatous deposits.

FURTHER READING
1. Abbott RW, Levy AD, Aquilera NS, et al. From the archives of the AFIP: Primary vascular neoplasms
of the spleen: Radiologic-patho­logic correlation. Radiographics. 2004;24(4): 1137-63.
2. Taibibi A, Bartolotta TV, Matranga D, et al. Splenic hemangiomas: Contrast-enhanced sonographic
findings. JUM. 2012;31:543-53.
3. Willcox TM, Speer RW, Schlinkert RT, Sarr MG. Hemangioma of the spleen: Presentation, diagnosis,
and management. J Gastrointest Surg. 2000;4(6):611-3.
Gastrointestinal System Imaging 253
CASE 8

A 13-year-old girl presented with chronic constipation and fecal soiling. She was well until 2 years
of age when she started having occasional brief episodes of constipation. She had periodic relief
with glycerine suppositories and diet modification. A barium enema was requested.

Figure 1  A 24-hour delayed


study after barium enema
showed persistent gross
distension of the rectum,
with the barium mixed with
fecal residue. There was no
zone of transition beyond
the distended rectum.
A diagnosis of functional
constipation was made,
with a less likely possibility
of an ultra-short segment
Hirschsprung’s disease.
She had manual evacuation
under general anesthesia

DISCUSSION
Chronic constipation may be defined as difficulty in defecating for 2 or more weeks, sufficient to
cause distress to the patient. Constipation in infants and older children may be due to many causes,
principal among which are Hirschsrung’s disease and functional constipation. The main challenge
in imaging is to distinguish between them. Clinically, Hirschsprung’s tends to affect the infant child,
with a prevalence of 1:5,000 to 10,000 infants, and a 4:1 male predilection. Functional constipation
is far more common, with a prevalence of 1:10, and with no sex predilection. In Hirschsprung’s
disease, the rectum is usually of normal caliber, whereas it is distended with fecal matter in
functional constipation.

FURTHER READING
1. Biggs WS, Dery WH. Evaluation and Treatment of constipation in Infants and children. Am Fam
Physician. 2006;73(3):469-77.
2. De Lorijin F, Reitsma JB, Voskuijil WP, et al. Diagnosis of Hirschsprung’s disease: A prospective,
comparative accuracy study of common tests. J Pediatr. 2005;146(6):782-92.
254 A Teaching Atlas of Case Studies in Diagnostic Imaging
CASE 9

A 55-year-old man presented with dysphagia of 3 months duration. A barium swallow was
requested.

Figure 1  Barium swallow


oblique image showing a
diverticulum arising from the
posterior wall of the proximal
esophagus and displacing the
esophageal lumen anteriorly.
The appearance is typical for
Zenker’s diverticulum

Figure 2  AP view of
the same patient
Gastrointestinal System Imaging 255

DISCUSSION
Zenker’s diverticulum results from the herniation of mucosa and submucosa through a focal
weakness in the hypopharynx at the cleavage plane posteriorly between the fibers of the inferior
pharyngeal constrictor and the cricopharyngeus. Patients usually present with dysphagia,
regurgitation of swallowed items, and halitosis.

FURTHER READING
1. Ferreira LE, Simmons DT, Baron TH. Zenker’s diverticulum: Pathophysiology, clinical presentation,
and flexible endoscopic management. Dis Esophagus. 2008;21(1):1-8.
2. Keck T, Rozsasi A, Grun PM. Surgical treatment of hypopharyngeal diverticulum (Zenker’s
diverticulum). Eur Arch Otorhino­laryngol. 2010;267(4):587-92.
256 A Teaching Atlas of Case Studies in Diagnostic Imaging
CASE 10

A 32-year-old woman presented with dysphagia and recurrent epigastric pain. She was referred for
barium meal.

Figures 1A and B  Barium


meal shows a part of the
gastric fundus extending
into the thoracic cavity
above the esophageal
hiatus, along with the
gastroesophageal
junction. Features indicate
a sliding hiatus hernia.
Gastroesophageal reflux
was also observed at
B fluoroscopy
Gastrointestinal System Imaging 257

DISCUSSION
Hiatus hernia (HH) occurs when part of the stomach protrudes into the thoracic cavity through the
esophageal hiatus of the diaphragm. It comprises 2 types:
1. Sliding HH (99% of cases), in which the gastroesophageal junction lies in the thorax.
2. Paraesophageal HH (1% of cases), in which a part of the gastric fundus herniates into the thorax,
but the gastroesophageal junction remains intra-abdominal.
The patient may present with symptoms of reflux esophagitis or recurrent chest infection due to
aspiration. Paraesophageal HH may present acutely from volvulus or strangulation.
Apart from barium meal, HH can also be diagnosed by ultrasonography.

FURTHER READING
1. Barone M, Di Lernia P, Carbonara M, et al. Sliding gastric hiatal hernia diagnosed by transabdominal
ultrasonography: An easy, reliable and noninvasive procedure. Scand J Gastroenterol.
2006;41(7):851-5.
2. Kahrilas PJ, Kim HC, Pandolfino JE. Approach to the diagnosis and grading of hiatal hernia. Best
Pract Res Clin Gastroenterol. 2008;22(4):601-16.
258 A Teaching Atlas of Case Studies in Diagnostic Imaging
CASE 11

A 28-year-old man presented with acute abdominal pain and vomiting. He was referred for CT.

Figures 1A and B  Plain


CT axial and sagittal
reformatted images show a
central abdominal sausage-
shaped loop of jejunum,
containing mesenteric fat
and vessels, consistent
with a jejunojejunal
B intussusception
Gastrointestinal System Imaging 259

Figure 2  Coronal image


demonstrating the intussu­
septum and intussucipient.
Intussusception with a
leading jejunal polyp was
confirmed at laparotomy

DISCUSSION
See under Case 13 (Page 262).
260 A Teaching Atlas of Case Studies in Diagnostic Imaging
CASE 12

A 50-year-old man presented with recurrent right-sided abdominal pain, on and off for several
years, accompanied occasionally with vomiting. He had a past history of cholecystectomy.
Abdominal ultrasound when his pain was in remission revealed a hypertrophied right unilateral
kidney, but nothing else of note. A barium meal follow-through was suggested.

Figures 1A and B 
(A) Follow-through film of a
barium meal, with a close-up
(B) shows a segment of
ileum with a filling defect
surrounded by swollen,
B edematous mucosa
Gastrointestinal System Imaging 261

Figures 2A and B  Axial


and sagittal reformatted CT
images with oral contrast
show a central hypodensity
surrounded by contrast,
and consistent with an ileal
intussusception. This was
confirmed at laparotomy to
be due to an ileal polyp B

DISCUSSION
See under Case 13 (Page 262).
262 A Teaching Atlas of Case Studies in Diagnostic Imaging
CASE 13

A 31-year-old woman presented with acute abdominal pain. She had not missed her periods. Clinical
examination revealed a poorly defined epigastric mass. Abdominal ultrasound was requested.

Figure 1  Abdominal
ultrasound showed a rounded
subhepatic mass with
concentric alternating hyper
and hypoechoic pattern,
giving the classical “coiled
spring” or target appearance
classical of intussusception,
probably in the transverse
colon. There was no ascites

Figure 2A
Gastrointestinal System Imaging 263

Figures 2B and C
Figures 2A to C  Barium
enema confirmed the
diagnosis, with images
showing the progression
of hydrostatic reduction
down to the cecum. Note
the smooth, rounded filling
defect at the cecal pole,
indicating the leading mass C
264 A Teaching Atlas of Case Studies in Diagnostic Imaging

Figures 3A and B  Coronal


and axial CT images
show the mass with the
attenuation coefficient
of fat, confirming a cecal
B lipoma
Gastrointestinal System Imaging 265

DISCUSSION
Adult intussusception is rare, comprising 5% of all cases of intussusception. Contrary to the
situation in children, in 90% of adult intussusception, there is an underlying pathology, especially
a leading mass. Lead points may be lipomas (most common cause of colocolic intussusception),
polyps, Meckel’s diverticulum, appendix, adenocarcinoma, and metastatic melanoma, among
others. Because of the high probability of underlying leading point, reduction (e.g. hydrostatic)
before resection is controversial, but as in this patient, it may be required to relieve pain, if there is
no evidence of bowel infarction.

FURTHER READING
1. Choi SH, Han JK, Kim SH, et al. Intussusception in Adults: From Stomach to Rectum. AJR Am J
Roentgenol. 2004;183(3):691-8.
2. Kim YH, Blake MA, Harisinghani MG, et al. Adult intestinal intussusception: CT appearances and
identification of a causative lead point. Radiographics. 2006;26:733-44.
3. Yamarthi S, Smith RC. Adult intussusception: Case reports and review of literature. Postgrad Med J.
2008;81:174-7.
266 A Teaching Atlas of Case Studies in Diagnostic Imaging
CASE 14

A 26-year-old man presented with recurrent right hypochondrial pain. A plain abdominal
radiograph was done.

Figures 1A and B  There are


large multifaceted opacities
(A) at the liver edge, better
shown at close-up (B), consistent
with gallbladder calculi, and
B confirmed by ultrasound

DISCUSSION
See under Case 15 (Page 267).
Gastrointestinal System Imaging 267
CASE 15

A 71-year-old woman presented with jaundice, pain in the right hypochondrium, and elevated
liver enzymes.

Figures 1A and B  Magnetic


resonance cholangiopancrea­
tography (MRCP) shows a
distended gallbladder with
marked wall thickening,
suggesting cholecystitis.
The CBD is dilated and
contains several filling defects
in its distal lumen, indicating
calculi. The pancreatic
duct is normal B
268 A Teaching Atlas of Case Studies in Diagnostic Imaging

DISCUSSION
Gall-bladder calculi are common and are usually asymptomatic. They can, however, produce pain
if impacted in the cystic duct, or become complicated by cholecystitis. Ultra­sonography remains
the imaging of choice for the initial assessment of the biliary system. However, MRCP has gained
acceptance as the method of choice for demonstrating CBD calculi.

FURTHER READING
1. Varghese JC, Liddell RP, Farrell MD, et al. Diagnostic accuracy of magnetic resonance
cholangiopancreatography and ultrasound compared with direct cholangiography in the detection
of choledocholithiasis. Clin Radiol. 2000;55:25-35.
2. Watanabe Y, Nagayama M, Okumura A, et al. MR imaging of acute biliary disorders. Radiographics.
2007;27:477-95.
Gastrointestinal System Imaging 269
CASE 16

A 44-year-old man presented with vague generalized abdominal discomfort. Abdominal ultrasound
showed an echogenic lesion in the right lobe of the liver. CT was done for further evaluation.

Figure 1  Plain CT shows


the hypodense mass

Figure 2A
270 A Teaching Atlas of Case Studies in Diagnostic Imaging

Figures 2B and C
Figures 2A to C  Triphasic
contrast CT showing arterial,
venous and equilibrium
phases, with nodular
peripheral and progressive
centripetal contrast filling of
the mass. Note that the mass
filled in completely (C). This
fill-in pattern is characteristic
C of hepatic hemangioma

DISCUSSION
See under Case 17 (Page 271).
Gastrointestinal System Imaging 271
CASE 17

A 47-year-old woman presented with several years history of recurrent right hypochondrium
pain. Ultrasonography showed several large echogenic hepatic masses, with the biggest showing a
hypoechoic center, suggesting cystic degeneration. MRI was performed.

Figures 1A and B B
272 A Teaching Atlas of Case Studies in Diagnostic Imaging

Figure 1C
Figures 1A to C  Axial and
coronal T2w MR images
show multiple, large
masses with lobulated
outlines, consistent with
cavernous hemangiomas.
There is evidence of cystic
degeneration in the sub-
diaphragmatic mass

DISCUSSION
Hemangiomas are the most common benign tumors of the liver. They are comprised of atypical,
irregularly arranged blood vessels. Common modalities employed in the imaging of hepatic
hemangiomas include ultrasonography, without or with contrast enhancement, CT and MRI.
Hemangiomas are usually asymptomatic and discovered incidentally. When they are large, they can
cause pain, especially if they undergo cystic degeneration. Occasionally, cavernous hemangiomas
may rupture and bleed into the peritoneum, and become life-threatening. Most hemangiomas
require no treatment, but surgical excision may be required if they become symptomatic.

FURTHER READING
1. Dickie B, Dasgupta R, Nair R, et al. Spectrum of hepatic hemangiomas: Management and outcome.
J Pediatr Surg. 2009;44(1):125-33.
2. Dietrich CF, Mertens JC, Braden B, et al. Contrast-enhanced Ultrasound of histo­logically proven
liver hemangiomas. Hepatology. 2007;45(5):1139-45.
3. Gandhi SN, Brown MA, Wong JG, et al. MR contrast agents for liver imaging: What, When, How.
Radiographics. 2006;26:1621-36.
4. Ribeiro MAF, Papaiordanou F, Amodio PM, et al. Hemoperitoneum from a spontaneous rupture of
hepatic hemangiomas: A review of the literature. World J Hepatol. 2010;2(12):428-33.
Gastrointestinal System Imaging 273
CASE 18

A 38-year-old Egyptian man previously diagnosed with hepatic schistosomiasis came for a routine
follow-up abdominal ultrasound. His liver showed typical features of periportal fibrosis. He also
had splenomegaly but no ascites.

Figure 1  There is marked


increase in periportal
echogenicity (Compare with
normal liver in Figure 2)

Figure 2  Normal liver


parenchyma
274 A Teaching Atlas of Case Studies in Diagnostic Imaging

DISCUSSION
Presinusoidal periportal fibrosis is a well-recognized feature of hepatic schistosoma Mansoni
infestation, and ultrasonography has been used in monitoring response to therapy in communities
where the parasite is endemic.

FURTHER READING
1. De Jesus AR, Miranda DG, Miranda RG, et al. Schistosoma mansoni infection determined
by ultrasound in an endemic area of Brazil. Am J Trop Med Hyg. 2000;63:1-4.
2. Hassan S, Hawass ND, Zaidi AJ. Ultra­sonographic diagnosis of schistosomal periportal fibrosis.
JUM. 1984;3(10):449-52.
Gastrointestinal System Imaging 275
CASE 19

A 5-year-old boy presents with acute upper abdominal pain and epigastric tenderness. Abdominal
ultrasound showed an enlarged pancreas and probe tenderness. He was thought to have acute
pancreatitis, which was confirmed by elevated pancreatic enzymes. He responded well to
conservative management, but had a repeat episode of severe abdominal pain one month later.
A CT abdomen was done to exclude complications.

Figure 1  Contrast-
enhanced CT abdomen
showing an enlarged
pancreas with several
hypodense foci in its body
and tail and peripancreatic
fat stranding, indicating
recurrence of acute
pancreatitis

DISCUSSION
See under Case 20 (Page 276).
276 A Teaching Atlas of Case Studies in Diagnostic Imaging
CASE 20

A 34-year-old man, known alcoholic and hypertensive, presented with epigastric pain and elevated
liver enzymes.

Figures 1A and B  T2w axial


and coronal MRI abdomen
show an enlarged pancreas
with peripancreatic edema
and fat stranding, features
B indicating acute pancreatitis
Gastrointestinal System Imaging 277

Figure 2  MRCP shows a


normal gallbladder and
cystic duct. CBD is normal in
size but shows two calculi
in its distal end. Note the
widening of the duodenal
loop from the swollen head
of pancreas

DISCUSSION
Biliary calculi and alcohol abuse are the most common predisposing factors to pancreatitis in
adults. It may also follow blunt abdominal trauma or iatrogenic trauma from surgery or procedures
such as ERCP. Pancreatitis is uncommon in children, and may arise from blunt trauma, e.g. falls on
a bicycle handle bar or following systemic infection, or from mumps or rubella.
Acute pancreatitis may be mild, associated with minimal organ dysfunction and uneventful
recovery. But when severe, it may be associated with pancreatic necrosis, hemorrhage, venous
thrombosis, and pseudocyst formation.

FURTHER READING
1. Benifla M, Weizman Z. Acute pancreatitis in childhood: Analysis of literature data. J Clin
Gastroenterol. 2003;37(2):169-72.
2. O’Connor OJ, McWilliams S, Maher MM, Imaging of acute pancreatitis. AJR Am J Roentgenol.
2011;197:W 221-5.
3. Xiao B, Zhang XM, Tang W, et al. Magnetic resonance imaging for local complications of acute
pancreatitis: A pictorial review. World J Gastroenterol. 2010;16(22):2735-42.
278 A Teaching Atlas of Case Studies in Diagnostic Imaging
CASE 21A

A 35-year-old woman presented with recurrent epigastric pain and progressive abdominal swelling.

Figures 1A and B  Contrast-


enhanced CT showing a
thin-walled retroperitoneal cyst
closely related to the pancreatic
B body and tail

DISCUSSION
See under Case 21B (Page 279).
Gastrointestinal System Imaging 279
CASE 21B

A 54-year-old man presented with a history of repeated episodes of upper abdominal pain and a
swelling noticed in the previous month.

Figures 1A and B Plain


and contrast-enhanced CT
abdomen showing large
pancreatic pseudocysts
(Courtesy: Dr Adekunle
Abdulkadir) B
280 A Teaching Atlas of Case Studies in Diagnostic Imaging

DISCUSSION
Pancreatic pseudocyst is a collection of pancreatic fluid encapsulated by fibrous or granulation
tissue which is not lined by epithelium. It represents about two-thirds of pancreatic cystic lesions.
Pseudocyst formation is a well-known complication of pancreatitis and is found in about 2–4% of
acute pancreatitis and 10–15% in chronic pancreatitis. It may also follow pancreatic trauma and
cancer. Most pseudocysts are single or unilocular.
Common clinical manifestations include abdominal pain, early satiety, nausea and vomiting,
weight loss, and obstructive jaundice. Even large pancreatic pseudocysts can be asymptomatic, but
they may produce complications such as bile duct or duodenal compression, portal hypertension,
pseudoaneurysm, hemorrhage, infection, rupture or pancreatic ascites.
Abdominal CT is the method of choice for imaging pancreatic pseudocysts, with a sensitivity
of 82–100% and specificity of 98%. Not all pancreatic pseudocysts require intervention. Selected
patients who are asymptomatic can be subject to conservative management. Spontaneous
resolution has been shown to occur in 40–50% of patients with no serious complications occurring
during the observation period. Intervention is warranted if the pseudocyst shows progressive
increase in size or becomes symptomatic or complicated.

FURTHER READING
1. Gumaste VV, Aron J. Pseudocyst manage­ ment: Endoscopic drainage and other emerging
techniques. J Clin Gastroenterol. 2010;44(5):326-31.
2. Habashi S, Draganov PV. Pancreatic pseudo­cyst. World J Gastroenterol. 2009;15:38-47.
3. Sahani DV, Kadavigere R, Saokar A, et al. Cystic pancreatic lesions: A simple imaging-based classification
system for guiding management. Radiographics. 2005;25(6):1471-84.
4. Spivak H, Galloway JR, Amerson JR, Fink AS, Branum GD, Redvanly RD, et al. Manage­ment of
pancreatic pseudocysts. J Am Coll Surg. 1998;186:507-11.
Gastrointestinal System Imaging 281
CASE 22

A 73-year-old man previously diagnosed with chronic liver cirrhosis was sent for a follow-up
evaluation by ultrasonography. He was discovered to have multiple hypoechoic lesions in the liver,
splenomegaly and ascites. A diagnosis of hepatocellular carcinoma (HCC) was made, and contrast-
enhanced abdominal CT was advised.

Figure 1  Axial contrast-


enhanced CT shows a cirrhotic
liver with parenchymal
hypodensities, highly
suggestive of HCC

Figure 2  Axial image at a


lower level shows a large tumor
thrombus in the inferior vena
cava (IVC). Note ascitic fluid
surrounding the right lobe of
the liver, gastric varices
and splenomegaly
282 A Teaching Atlas of Case Studies in Diagnostic Imaging

Figure 3  Sagittal
reformatted image showing
the IVC thrombus extending
beyond the diaphragm

DISCUSSION
Most cases of HCC are secondary to chronic hepatitis (B or C) infection, cirrhosis or from toxins
(Aflatoxin). Extension of tumor cells into the portal vein occurs frequently and occasionally into the
hepatic vein. Tumor thrombus into the IVC, as in this patient, is a rare occurrence.

FURTHER READING
1. Agelopoulou P, Kapatais A, Varounis C, et al. Hepatocellular carcinoma with invasion into the right
atrium. Report of two cases and review of literature. Hepatogastroenterology. 2007;54(79):2106-8.
2. Kanematsu M, Imaeda T, Minowa H, et al. Hepatocellular carcinoma with tumor thrombus in the
inferior vena cava and right atrium. Abdom Imaging. 1994;19(4):313-6.
3. Sung AD, Cheng S, Moslehi J, et al. Hepatocellular carcinoma with intracavitary cardiac involvement:
A Case report and review of the literature. Am J Cardiol. 2008; 102(5):643-5.
Gastrointestinal System Imaging 283
CASE 23

A 50-year-old man presented with jaundice and abdominal discomfort and bloating. Initial
abdominal ultrasound showed a gallbladder calculus and an irregularly contracted common bile
duct (CBD) and intrahepatic biliary dilatation. An MRCP was advised.

Figure 1  MRCP shows


irregular narrowing of the
CBD involving most of
its length, as suggested
by involvement of the
pancreatic duct, which is
distended. Note dilatation
of the hepatic ducts and
intrahepatic biliary radicles.
Features are those of
infiltrative type extrahepatic
cholangiocarcinoma

DISCUSSION
About 88% of cholangiocarcinomas are extrahepatic, and these are classified as infiltrative,
polypoid, exophytic or combined. A Klatskin tumor is one located in the confluence of the right and
left hepatic ducts, which location poses a challenge for resectability and subsequent management.
Several risk factors have been identified in the development of cholangiocarcinoma. These
include: primary sclerosing cholangitis, choledochal cyst, hepatolithiasis, and clonorchis sinensis
infestation. In many cases, however, no predisposing factor can be identified.

FURTHER READING
1. Park MS, Kim TK, Kim KW, et al. Differentiation of extrahepatic bile duct cholangiocarcinoma from
benign stricture: Findings at MRCP versus ERCP. Radiology. 2004;233:234-40.
2. Sainani NI, Catalano OA, Holalkere NS, et al. Cholangiocarcinoma: Current and novel imaging
techniques. Radiographics. 2008;28:1263-87.
3. Vanderveen KA, Hussain HK. Magnetic resonance imaging of cholangiocarcinoma. Cancer
Imaging. 2004;4(2):104-15.
284 A Teaching Atlas of Case Studies in Diagnostic Imaging
CASE 24

A 45-year-old male presented with jaundice, right upper abdominal pain, hepatospleno­megaly and
abnormal liver function tests.

Figure 1  MRCP shows


diffuse multifocal short
strictures in both the
intrahepatic and extrahepatic
bile ducts. The strictures
alternate with normal and
mildly dilated intervening
segments resulting in a
beaded appearance.
The features are consistent
with primary sclerosing
cholangitis (PSC)

DISCUSSION
PSC is a chronic progressive cholestatic liver disease of unknown etiology. Seventy-percent of
patients are males, younger than 45 years, and 70% have ulcerative colitis. Other associated diseases
include sicca complex, retroperitoneal fibrosis and Riedel’s struma. There is no known therapy
short of liver transplantation. ERCP and/or MRCP may show, in addition to the multiple strictures,
fibrous obliteration of peripheral bile ducts (pruned tree), nodularity, intraluminal webs and small
diverticula. Findings seen on MRI include wedge-shaped T2 hyperintense areas, large nodular
lesions, isointense on T1, and hypointense on T2 in the central liver region, and areas of atrophy.
Diagnosis of cholangiocarcinoma coexisting with PSC may be very difficult to make, and PSC is
considered a risk factor for cholangiocarcinoma.

FURTHER READING
1. Gotthardt D, Chahoud F, Sauer P. Primary sclerosing cholangitis: Diagnostic and therapeutic
problems. Dig Dis. 2011;29(1):41-5.
2. Lee YM, Kaplan MM. Primary sclerosing cholangitis. N Engl J Med. 1994;332:924-33.
3. Ponsioen GJ, Tytgat GNJ. Primary sclerosing cholangitis: a clinical review. Am J Gastro­enterol.
1998;93:515-23.
Gastrointestinal System Imaging 285
CASE 25

A 41-year-old woman was involved in a road traffic accident and sustained blunt thoracoabdominal
injury. Initial abdominal ultrasonography indicated free intraperitoneal fluid. Contrast-enhanced
chest and abdominal CT were done.

Figures 1A and B  Axial


and coronal reformatted
CT show a shattered spleen
embedded in a large
hematoma, considered a
Grade V splenic injury.
The liver is intact. Note
gaseous distension of
the stomach (A) B
286 A Teaching Atlas of Case Studies in Diagnostic Imaging

Figure 2  Surface-shaded
display of the ribs shows
undisplaced fractures of
the left 9–11th ribs

DISCUSSION
See under Case 26 (Page 287).
Gastrointestinal System Imaging 287
CASE 26

A 32-year-old man was brought to A and E following a road traffic accident in which he sustained
chest and abdominal injuries.

Figures 1A and B  Axial


and coronal reformatted
contrast-enhanced
CT images show liver
laceration, perihepatic
hemoperitoneum and
extensive soft tissue
emphysema B
288 A Teaching Atlas of Case Studies in Diagnostic Imaging

DISCUSSION
Abdominal injuries are very common in clinical practice. The spleen, being a highly vascular organ
(estimated to have 350 liters of blood passing through it per day), splenic injuries can be potentially
life-threatening.
Contrast-enhanced CT has been shown to have 98% accuracy in diagnosing acute splenic and
hepatic injuries, and to help grade the extent of injury, decide on the mode of management, including
endovascular therapy in selected cases, and to determine prognosis. Although ultrasonography is
very sensitive in detecting free peritoneal fluid, in the setting of the acutely injured patient, with
reactive gas-distension of the stomach and bowel, it is not nearly as efficient as CT in assessing solid
organ injury.

FURTHER READING
1. Kendall JL, Faragher J, Hewitt GJ, et al. Emergency department ultrasound is not a sensitive detector
of solid organ ijury. West J Emerg Med. 2009;10(1):1-5.
2. Marmery H, Shanmuganathan K, Alexander MT, Mirvis SE. Optimization of selection for
non-operative management of blunt splenic injury: Comparison of MDCT grad­ing systems. AJR
Am J Roentgenol. 2007;189(6):1421-7.
3. Sharma OP, Oswanski MF, Singer D, et al. Assessment of nonoperative management of blunt
spleen and liver trauma. Am Surg. 2005;71(5):379-86.
4. Willmann KJ, Roos JE, Platz A, et al. Multi­detector CT: Detection of active hemorrhage in patients
with blunt abdominal trauma. AJR Am J Roentgenol. 2002;179(2):437-44.
5. Yao DC, Jeffrey RB, Mirvis SE, et al. Using contrast-enhanced helical CT to visualize arterial
extravasation after blunt abdominal trauma. Incidence and organ distribution. AJR Am J Roentgenol.
2002;178(1):17-20.
Gastrointestinal System Imaging 289
CASE 27

A 66-year-old woman-diabetic hypertensive, presented with acute, generalized abdominal pain


and vomiting. A KUB was nonspecific, and abdominal ultrasound showed a small amount of
peritoneal-free fluid. CT was advised, which was done plain because of a history of allergy.

Figure 1  Axial CT
showing a gas-fluid level
in the superior mesenteric
vein and tubular, gas-
filled vessels in the
mesentery. The gas is best
demonstrated on lung-
windowing, such as this

Figure 2  Right sagittal


reformatted image showing
gas layering in the superior
mesenteric vein and gas in
the smaller vessels within
the mesentery. Note the
severe atheromatous
changes in the aorta
290 A Teaching Atlas of Case Studies in Diagnostic Imaging

Figure 3  Axial CT of the


liver showing extensive gas
in the portal vein radicles.
Diagnosis: Acute bowel
infarction. The patient had
laparotomy and successful
resection of the gangrenous
small bowel segment

DISCUSSION
Acute mesenteric ischemia can be caused by various conditions, such as arterial or venous occlusion,
strangulating bowel obstruction, or hypoperfusion associated with nonocclusive vascular disease.
The clinical features can range from transient ischemia to catastrophic necrosis.
The common findings include intramural pneumatosis, mesenteric or portal venous gas,
engorged mesenteric veins and edema, lack of bowel wall contrast enhancement, wall thickening,
with alternating high and low attenuation due to submucosal edema, giving the “target sign”.
Regardless of the cause of bowel ischemia, the imaging features are similar.

FURTHER READING
1. Chou CK. CT manifestations of bowel ischemia. AJR Am J Roentgenol. 2002;178(1):87-91.
2. Furukawa A, Kanasaki S, Kono N, et al. CT diagnosis of acute mesenteric ischemia from various
causes. AJR Am J Roentgenol. 2009;192(2):408-416.
3. Ofer A, Abadi S, Nitecki S, et al. Multidetector CT angiography in the evaluation of acute mesenteric
ischemia. Eur Radiol. 2009;19(1):24-30.
4. Rha SE, Ha HK, Lee SH, et al. CT and MR Imaging findings of bowel ischemia from various primary
causes. Radiographics. 2000;20:29-42.
Gastrointestinal System Imaging 291
CASE 28

A 45-year-old female was involved in a road traffic accident and sustained abdominal and pelvic
injuries. Six months later, she developed intermittent abdominal pain and vomiting. Conventional
barium meal follow through was done.

Figure 1  The barium


follow-through study
shows multiple small bowel
strictures associated with
dilated small bowel loops
and mucosal thickening.
A left femoral neck fracture
had been pinned, and a left
herniorraphy performed.
An IUCD is noted.

Diagnosis: In view of the history of injury, the strictures are likely to be a result of mesenteric
ischemia. Differential diagnoses include tuber­culosis or Crohn’s disease, both of which were ruled
out in this patient.

DISCUSSION
Mesenteric ischemia may be acute or chronic, and may be due to thrombosis or embolism, bowel
obstruction, abdominal inflammatory conditions, trauma or neoplasm. The patho­logical effects on
the bowel are variable. Necrosis, ulceration and/or hemorrhage may occur. Necrosis of the deep
submucosal and muscular layers may lead to the development of fibrotic strictures, as in this case.
MDCT angiography is the modality of choice for the evaluation of bowel ischemia.

FURTHER READING
1. Rha SE, Ha HK, Lee S, et al. CT and MR imaging findings of bowel ischaemia from various primary
causes. Radiographics. 2000;20:29-42.
2. Weisner W, Khurana B, Ji H, Ros PR. CT of acute bowel ischaemia. Radiology. 2003;226: 635-50.
292 A Teaching Atlas of Case Studies in Diagnostic Imaging
CASE 29

A 23-year-old lady presented with recurrent episodes of abdominal cramps, vomiting, nausea and
diarrhea for 18 months. A barium meal follow through study was done to rule out inflammatory
bowel disease.

Figure 1  Supine frontal


radiograph of upper gastro­
intestinal series shows
duodenojejunal junction and
entire small bowel loops on
right side of abdomen without
crossing the spine to left

Figure 2  Normal small bowel


transit time was observed and
further radiographs show entire
colon on the left side of spine.
However, the ileocecal junction
and cecum are seen normally
positioned on the right side of
the pelvis
Gastrointestinal System Imaging 293

Figure 3  Axial contrast-enhanced


CT scan of abdo­men shows
inversion of the normal position of
the superior mesenteric artery and
vein, the superior mesenteric vein
being located on the left of
the artery which confirms
nonrotation of the intestines.
Note also associated absence of
the uncinate process of pancreas
(Courtesy: Dr Sujatha Rajkumar)

DISCUSSION
In normal upper gastrointestinal series, the duodenojejunal junction lies to the left of the spine at
the level of the duodenal bulb.
Nonrotation of the intestine occurs in about 1 in 500 live births. The midgut loop returns to the
peritoneal cavity without rotation, resulting in weak peritoneal fixation. Nonrotation is generally
asymptomatic and often an incidental finding in older children and adults. In adults, it may present
as acute bowel obstruction or as vague intermittent abdominal pain. Early diagnosis can prevent
the complications of midgut volvulus with “whirl sign” around superior mesenteric artery and small
bowel necrosis. The upper gastrointestinal series remains the gold standard of reference for the
diagnosis of malrotation with 93–100% sensitivity. Plain radiography is neither sensitive nor specific
for malrotation. Contrast enema examination usually shows malposition of the right colon, but the
cecum may assume a normal location in up to 20% of patients. Cross-sectional imaging (particularly
CT), performed for various unrelated reasons, may show the intestinal malpositioning seen on
barium studies and also deviation from the normal relationship between the superior mesenteric
artery (SMA) and superior mesenteric vein (SMV), which is a useful indicator of malrotation. In
most patients with quiescent malrotation, the SMA and SMV will assume a vertical relationship or
show left- right inversion. Similar findings can be seen on ultrasonography. Underdevelopment or
absence of the uncinate process of the pancreas has been observed in bowel nonrotation.

FURTHER READING
1. Mohan P, Ramamoorthy M, Venkatraman J. Nonrotation of the intestine. J Canadian Medical Assoc.
2008;179(1):49-50.
2. Pickhardt PJ, Bhallas. Intestinal malrotation in adolescents and adults: Spectrum of clinical and
imaging features. AJR Am J Roentgenol. 2002;179(6):1429-35.
3. Dahnert W. Radiology Review Manual, 7th edition. Gastrointestinal Disorders. Lippincott Williams
and Wilkins, Phila­delphia; 2011. pp. 869-70.
294 A Teaching Atlas of Case Studies in Diagnostic Imaging
CASE 30

A 19-year-old girl, with thalassemia developed a fever 8 days postsplenectomy.

Figures 1A and B  Axial


and coronal reformatted
contrast-enhanced CT
images show a large
collection in the splenic
bed, with loculi of gas in
it and rim enhancement,
consistent with an
abscess. Note the dense
hepatomegaly from the
B hemoglobinopathy
Gastrointestinal System Imaging 295

DISCUSSION
Intra-abdominal abscess may complicate many gastrointestinal conditions or be a complication
of surgery. Contrast-enhanced CT remains the imaging modality of choice in the assessment of
possible abdominal abscesses. It helps to determine the anatomic details, size of the cavity and
its relationship to surrounding structures. The typical abscess is well-circumscribed and may
contain micro-bubbles, which when they coalesce, form larger pockets with air-fluid levels. There is
usually an enhancing rim, and the surrounding mesentery may show fat stranding. The attenuation
coefficient of abscesses vary from 0 to +30 Hounsfield units. Large abscesses require per-cutaneous
or surgical drainage, but small ones (<3 cm diameter) may sometimes resolve with antibiotics.
CT or ultrasonography is invaluable in guiding percutaneous drainage.

FURTHER READING
1. Gervais DA, Brown SD, Connolly SA, et al. Percutaneous imaging-guided abdominal and pelvic
abscess drainage in children. Radiographics. 2004;24:737-54.
2. Kim YJ, Han JK, Lee JM, et al. Percutaneous drainage of postoperative abdominal abs­cess with limited
accessibility: Preexisting surgical drains as alternative access route. Radiology. 2006;239(2):591-8.
3. van Sonnenberg E, Wittich GR, Goodcare BW, et al. Percutaneous abscess drainage: Update.
World J Surg. 2001;25:362-72.
296 A Teaching Atlas of Case Studies in Diagnostic Imaging
CASE 31

A 4-year-old boy presented with fever, difficulty in breathing and swallowing. He was said to have
ingested a foreign body one week earlier.

Figure 1  Lateral neck


radiograph shows a large
prevertebral soft tissue
widening extending
from the base of the skull
to C7 and displacing the
trachea anteriorly. Note the
tiny nodular metallic
opacity representing the
ingested foreign body
(Courtesy: Dr Adekunle
Abdulkadir)

Diagnosis: Retropharyngeal abscess.

DISCUSSION
Retropharyngeal abscess (RPA) is an uncommon, potentially fatal condition found more frequently
in children than in adults, especially in children younger than 5 years. RPA generally arises as a
consequence of infections of the nasopharynx, paranasal sinuses, or middle ear; from trauma, such
as accidental swallowing of a foreign body or traumatic orotracheal intubation. In adults, regional
trauma, foreign body ingestion, complication of procedures, and immunocompromised states
account for the majority of cases. Cervical spine TB is also a recognized cause.
The severity of symptoms is directly related to the volume of the abscess, varying from irritability
to severe dysphagia and respiratory distress. Abscesses in the retropharyngeal space can extend
Gastrointestinal System Imaging 297

superiorly to the soft palate and downwards into the thorax. Although plain radiography remains
invaluable, CT with contrast is the current imaging modality of choice for providing more accurate
anatomic localization and in distinguishing RPA from retropharyngeal cellulitis.

FURTHER READING
1. Brechtelsbauer PB, Garetz SL, Gebarski SS, Bradford CR. Retropharyngeal abscess: Pitfalls of plain
films and computed tomo­graphy. Am J Otolaryngol. 1997;18:258-62.
2. Craig FW, Schunk JE. Retropharyngeal abscess in children: Clinical presentation, utility of imaging,
and current management. Pediatrics. 2003;111(6 Pt 1):1394-8.
3. Glasier CM, Stark JE, Jacobs RF, et al. CT and ultrasound imaging of retropharyngeal abscesses in
children. AJNR Am J Neuro­radiol.1992;13:1191-5.
4. Goldenberg D, Golz A, Joachims HZ. Retro­pharyngeal abscess: A clinical review. J Laryngol Otol.
1997;111:546-50.
298 A Teaching Atlas of Case Studies in Diagnostic Imaging
CASE 32

A 33-year-old female presented with a 24-hour history of abdominal pain and vomiting. She was not
pregnant and had no history of previous surgery. A KUB radiograph was nonspecific. Abdominal
ultrasound showed dilated loops of small bowel and a considerable amount of free peritoneal fluid.
CT abdomen was done.

Figure 1  Contrast-enhanced
axial CT abdomen shows a
mass-like cluster of bowel
loops with mesenteric vessels
extending into it. There are fluid-
distended loops of small bowel
embedded in peritoneal fluid

Figure 2A
Gastrointestinal System Imaging 299

Figure 2B
Figures 2A and B  Coronal
and parasagittal reformatted
images demonstrate the
incarcerated loops of bowel
appearing like a mass

Diagnosis: Internal hernia with possible volvu­lus and strangulation. Laparoscopy con­firmed a
paraduodenal hernia containing small loops of bowel that had undergone torsion and become
gangrenous. 1.7 m of bowel were resected, and the patient made an uneventful recovery.

DISCUSSION
Internal hernias are rare, comprising less than 1% of the causes of intestinal obstruction. They,
however, carry a high mortality rate (>50%). They may be congenital or acquired (following surgery,
trauma or abdominal inflammation). The lesion involves the herniation of a viscus (usually small
bowel) through a normal or abnormal aperture within the peritoneal cavity. The main types are:
Paraduodenal (most common), foramen of Winslow, intersigmoid, and pericecal.
CT is the modality of choice in the diagnosis of internal hernias. The main signs include evidence
of small bowel obstruction, clustering of small bowel loops, and displaced, crowded and engorged
mesenteric vessels. Paraduodenal hernias demonstrate a sac-like mass located between the
stomach and pancreatic tail.

FURTHER READING
1. Blachar A, Federle MP, Dodson SF. Internal Hernia: Clinical and imaging findings in 17 patients
with emphasis on CT criteria. Radiology. 2001;218:68-74.
2. Martin LC, Merkle EM, Thomson WM. Review of internal hernias: Radiographic and clinical
findings. AJR Am J Roentgenol. 2006;186(3):703-17.
3. Takeyama N, Gokan T, Ohgiya Y, et al. CT of internal hernias. Radiographics. 2005;25: 997-1015.
300 A Teaching Atlas of Case Studies in Diagnostic Imaging
CASES 33A AND 33B

A 31-year-old man and an 8-year-old girl, respectively presented with right iliac fossa pain, vomiting,
fever and leukocytosis. Abdominal ultrasound showed classical features of acute appendicitis in
both patients.

Figure 1  Longitudinal
view of a tender, distended
appendix in the first patient

Figure 2  Transverse view of


the swollen appendix in
the second patient,
surrounded by
hypervascularity on Doppler

DISCUSSION
See under Case 38 (Page 305).
Gastrointestinal System Imaging 301
CASE 34

A 40-year-old man presented with fever, pain in the right hypochondrium and leukocytosis.
On clinical examination, there was tenderness on deep palpation in the right flank. The clinical
suspicion was of right renal disease. The kidneys and gallbladder were normal on ultrasound,
and there was a small amount of right iliac fossa (RIF) peritoneal fluid, but tenderness was mild,
and the appendix was not identified. Contrast-enhanced CT was done.

Figure 1  Coronal
reformatted contrast-
enhanced CT shows a
long, inflamed subhepatic
appendix containing several
appendicoliths. The findings
were confirmed at
laparotomy

DISCUSSION
See under Case 38 (Page 305).
302 A Teaching Atlas of Case Studies in Diagnostic Imaging
CASE 35

A 13-year-old boy presented with fever and right upper quadrant abdominal pain. Abdominal
ultrasound was equivocal. Contrast-enhanced CT was done.

Figure 1  There is a large


enhancing subhepatic
appendix, consistent with
appendicitis, and confirmed
at surgery

DISCUSSION
See under Case 38 (Page 305).
Gastrointestinal System Imaging 303
CASE 36

A 29-year-old man presented with fever and abdominal pain for 4 days prior to presentation.
On ultrasound, there was a complex echo pattern in the RIF, but the appendix was not definitely
identified. Contrast-enhanced CT was done.

Figure 1  There is a
hyperdense nodule
consistent with an
appendicolith. It is bordered
in its posterolateral aspect
by a hypodense collection
with rim enhancement,
indicating an abscess.
Note the peritoneal fat
stranding medially

DISCUSSION
See under Case 38 (Page 305).
304 A Teaching Atlas of Case Studies in Diagnostic Imaging
CASE 37

A 28-year-old man was seen in another hospital one week earlier complaining of abdominal pain
and mild pyrexia. Ultrasound examination performed there revealed no abnormality. He was
managed conservatively, but with worsening symptoms, he presented at our center. Plain and
contrast-enhanced CT studies were done.

Figure 1  Plain axial CT shows an oval


hyperdense shadow in the right side of
the pelvis, suggesting an appendicolith
in a pelvic appendix. Hypodensities
anteromedial to it suggest an
associated collection

Figure 2  Sagittal reformatted contrast-


enhanced CT image shows large
multiloculated collections posterior
to the urinary bladder and showing
rim enhancement, consistent with
abscesses

DISCUSSION
See under Case 38 (Page 305).
Gastrointestinal System Imaging 305
CASE 38

A 25-year-old man presented with a 4-day history of abdominal pain, vomiting and constipation.
He had generalized rebound tenderness and sluggish bowel sounds.

Figure 1  Composite pelvic


ultrasound (transverse
and sagittal) showing a
sonolucent lesion with
internal echoes and
septa, lying behind the
urinary bladder, highly
suggestive of an abscess.
At laparotomy, patient had
a perforated, gangrenous
appendix with peritonitis.
200 mL of purulent fluid
was drained

Figure 2A
306 A Teaching Atlas of Case Studies in Diagnostic Imaging

Figure 2B
Figures 2A and B  Plain
axial and sagittal
reformatted CT images
showing the large collection
(M) lying behind the urinary
bladder (UB)

DISCUSSION
Ultrasonography and multidetector CT have significantly improved the diagnostic accuracy of acute
appendicitis. The common protocol is to examine suspected cases first by ultrasound, reserving CT
for cases with equivocal findings. The diagnosis of appendicitis may pose a challenge in patients
with atypical symptoms, in infants and small children, in the elderly, in young women, especially
during pregnancy, and in obese patients. Challenges also occur when the appendix is in an unusual
location, e.g. subhepatic, retrocecal or within the bony pelvis. In the specific case of the pregnant
patient, MRI has been found useful in the diagnosis of appendicitis when ultrasound is equivocal. It
is vital that an early diagnosis is made so that definite therapy can be instituted, as perforation can
lead to peritonitis which can be life-threatening.

FURTHER READING
1. Anderson BA, Salem L, Flum DR. A systematic review of whether oral contrast is necessary for the
computed tomography diagnosis of appendicitis in adults. Am J Surg. 2005;190(3):474-8.
2. Kim HC, Yang DM, Jin W, Park SJ. Added diagnostic value of multiplanar reformation of Multidetector
CT data in patients with suspected appendicitis. Radiographics. 2008;28:393-405.
3. Pedrosa I, Levine D, Eyvazzadeh AD, et al. MR imaging evaluation of acute appendicitis in
pregnancy. Radiology. 2006;238(3):891-9.
4. Pickhardt PJ, Lawrence EM, Pooler BD. Bruce RJ. Diagnostic performance of multidetector
computed tomography for suspected acute appendicitis. Ann Intern Med. 2011;154(12):789-96.
5. Whitley S, Sookur P, McLean A, Power N. The appendix on CT. Clin Radiol. 2009;64(2):190-9.
Gastrointestinal System Imaging 307
CASE 39A

A 35-year-old man complains of generalized abdominal pain and bloody diarrhea mixed with
mucus for 8 days. He had a similar attack a year previously. Physical examination revealed left-sided
abdominal tenderness and stools contained RBCs and pus cells.

Figures 1 and 2  Pelvic images


of CT abdomen with oral and IV
contrast show cecal and ascending
colon wall thickening (7–10 mm),
with pericolic fat stranding
and vasa recti engorgement.
Inflammatory changes with wall
thickening are shown in the
rectum, sigmoid and entire colon.
No fistula or abscess is shown.
The liver, pancreas, spleen and
kidneys are normal B

Diagnosis: Ulcerative colitis.


308 A Teaching Atlas of Case Studies in Diagnostic Imaging

CASE 39B

Figures 1A and B  Double


contrast barium enema
images of another patient
B showing pancolitis
Gastrointestinal System Imaging 309

DISCUSSION
Ulcerative colitis is a superficial inflammatory disease characterized by a symmetric, uniform and
contiguous process that usually involves the rectum extending proximally until the entire colon
is affected, producing a pancolitis. Imaging modalities used in its evaluation are barium studies,
ultrasonography, CT, MRI and PET/CT. Virtual colonoscopy or CT colonography is a novel and
rapidly evolving technique.

FURTHER READING
1. Camascosa P, Castiglioni R, Capunay C, et al. CT colonoscopy in inflammatory bowel disease.
Abdom Imaging. 2007;32:596-601.
2. Gore RM. CT of inflammatory bowel disease. Radiol Clin North Am. 1989;27:717-23.
310 A Teaching Atlas of Case Studies in Diagnostic Imaging
CASE 40

A middle-aged man had gastroscopy for the assessment of suspected peptic ulcer disease, during
which the stomach was insufflated with air. Shortly after the procedure, the patient became
progressively dyspneic. An abdominal radiograph was done.

Figure 1  There is a
large collection of air
in the peritoneal cavity
producing severe elevation
of the diaphragm, and
compromising respiration.
The liver, stomach and
bowel loops are compressed
to the midline. Features
are those of a tension
pneumoperitoneum,
thought to be due to the
insufflated air leaking
through a perforated
duodenal ulcer

DISCUSSION
Tension pneumoperitoneum is the accumula­tion of a massive amount of gas in the peritoneal
cavity. Although it is a rare occurrence, it is a surgical emergency, as it can produce respiratory
embarrassment and death. Needle decompression provides immediate relief while the definitive
management of the underlying problem can follow. Pneumoperitoneum has been identified to
arise from various causes of gastrointestinal tract perforation.

FURTHER READING
1. Burdett-Smith P, Jaffey L. Tension pneumoperitoneum. J Acc Emer Med. 1996; 13(3):220-1.
2. Chan SY, Kirsh CM, Jensen WA, Sherck J. Tension pneumoperitoneum. West J Med. 1996;165
(1-2):61-4.
3. Sajith A, O’Donohue B, Roth RM, Khan RA. CT scan findings in oesophagogastric perforation after
out of hospital cardio­pulmonary resuscitation. Emerg Med J. 2008;25(2):115-6.
4. Winer-Muram HT, Rumbak MJ, Bain RS. Tension pneumoperitoneum as a compli­ cation of
barotraumas. Crit Care Med. 1993; 21(6):941-3.
Gastrointestinal System Imaging 311
CASE 41

A 7-year-old boy was admitted with a history of colicky abdominal pain and vomiting. Two years
earlier, he was involved in an RTA, requiring exploratory laparotomy. He had plain radiographs of
the abdomen.

Figures 1A and B  Erect and


supine abdominal radiographs
show gaseous distension of small
bowel, with multiple fluid levels in
a step-ladder pattern, indicating
mechanical small bowel obstruction.
A repeat laparotomy revealed
obstruction from adhesive jejunal
bands, which were released
with good result B

DISCUSSION
See under Case 42 (Page 312).
312 A Teaching Atlas of Case Studies in Diagnostic Imaging
CASE 42

An 83-year-old man presented with vomiting and epigastric pain. He had laparotomy 30 years
earlier for an undisclosed reason. Clinical examination revealed borborygmi and other features of
intestinal obstruction.
The patient responded well to conservative treatment.

Figure 1  A supine abdominal


radiograph 45 minutes after
administering water-soluble oral
contrast shows gross dilatation
of the stomach, duodenal loop
and proximal jejunum, with
contrast escaping into a normal
caliber jejunum. Features indicate
incomplete upper jejunal
obstruction, probably from
adhesive bands

Figure 2A
Gastrointestinal System Imaging 313

Figure 2B
Figures 2A and B  Coronal
reformatted and axial CT images
show the dilated stomach and
proximal small bowel. Note a long
fluid level in the stomach in (B)

DISCUSSION
Adhesive bands are a recognized late complication of open and minimally invasive laparoscopic
surgery. Little is known about the fundamental etiology of postoperative fibrous adhesions, but it is
thought that it accounts for 60–70% of small bowel obstructions in the Western world.

FURTHER READING
1. Duron JJ, Hay JM, Msika S, et al. Prevalence and mechanisms of small intestinal obstruction following
laparoscopic abdomi­nal surgery. A Retrospective multicenter study. Arch Surg. 2000;135(2):208-12.
2. Fevang BT, Fevang J, Lie SA, et al. A long-term prognosis after operation for adhesive small bowel
obstruction. Ann Surg. 2004;240(2):193-201.
314 A Teaching Atlas of Case Studies in Diagnostic Imaging
CASE 43

A 78-year-old man presented with rectal bleeding and loss of weight. He had abdominal ultrasound
and CT with oral and IV contrast.

Figure 1  Pelvic image of axial


CT shows irregular thickening
of the sigmoid colon with
pericolic fat stranding.
The appearances are
consistent with sigmoid
carcinoma

Figure 2  Large, hypodense


lesions are shown in the liver,
consistent with metastases
Gastrointestinal System Imaging 315

DISCUSSION
Colon cancer is the third most common malignancy among both men and women in the Western
world. The role of imaging is not only to diagnose the presence of malignancy but to stage it,
so as to determine the mode of therapy. CT has become the modality of choice for achieving these
objectives.

FURTHER READING
1. Burton S, Brown G, Bees N, et al. Accuracy of CT prediction of poor prognostic features in colonic
cancer. Br J Radiol. 2008;81:10-9.
2. Dighe S, Swift I, Brown G. CT staging of colon cancer. Clin Radiol. 2008;63(12):1372-9.
3. Horton KM, Abrams RA, Fishman EK. Spiral CT of Colon cancer: Imaging features and role in
management. Radiographics. 2000;20:419-30.
316 A Teaching Atlas of Case Studies in Diagnostic Imaging
CASE 44

A 35-year-old male homosexual presented with lower abdominal pain, constipation and rectal
bleeding. A plain abdominal radiograph was taken.

Figures 1A and B  A most


unusual foreign body is
identified in the rectum.
It was a deodorant spray
can which he had inserted
into his rectum for erotic
purposes. There is no
evidence of perforation.
The foreign body was safely
removed under general
B anesthesia
Gastrointestinal System Imaging 317

DISCUSSION
Foreign bodies may enter the alimentary or genitourinary tracts through various ports. The majority
of rectal foreign bodies are inserted for erotic purposes by depraved or homosexual individuals.
Attempts to retrieve the objects personally or by friends may fail, and they may be compelled to seek
medical care. Foreign bodies in the rectum or urogenital tract may also be due to sexual assault,
cleansing enema, vaginal douche, criminal abortion, etc. They usually present with abdominal
pain, rectal bleeding and fever. Drug smugglers (“body packers”) are common culprits. Removal
may be accomplished manually, but in some cases laparotomy and colostomy are required.

FURTHER READING
1. Busch AB, Starling JR. Rectal foreign bodies: Case reports and comprehensive review of the world
literature. Surgery. 1986;100:512-9.
2. Cohen JS, Sackian JM. Management of colorectal foreign bodies. J Roy Coll Surg. 1996;41:312-5.
3. Goldberg JE, Steele SR. Rectal foreign bodies. Surg Clin North Am. 2010;90(1):173-84.
5 Neurological
System Imaging

CASE 1

A 1-year-old female presented with dysmorphic facies, hypotonia and developmental delay.

Figure 1  Sagittal T1w MRI brain showing


a large posterior fossa hypointense lesion
with cerebellar hypoplasia

Figure 2  Axial T1w image showing the


anterior extent of the lesion and a positive
“molar tooth sign”
Neurological System Imaging 319

Figure 3  Axial T2w image showing


a positive “molar tooth sign”

Diagnosis: Joubert syndrome.

DISCUSSION
Joubert syndrome is a genetic disorder characterized by absence or underdevelopment of the
cerebellar vermis and malformation of the brain stem. Because of lack of normal decussation of
the superior cerebellar fiber tracts, the mid-brain gives the characteristic “molar tooth sign”.
Patients usually present with ataxia, hypotonia, nystagmus, dysmorphic facies and developmental
delay.

FURTHER READING
1. McGraw P. The molar tooth sign. Radiology. 2003;229:671-2.
2. Saraiva JM, Baraitser M. Joubert syndrome: A review. Am J Med Genet. 1992:43(4):726-31.
3. van Beek EJR, MajoieCBL. Case 25: Joubert syndrome. Radiology. 2000;216:379-82.
320 A Teaching Atlas of Case Studies in Diagnostic Imaging
CASE 2

A 7-month-old female with delayed develop­mental milestones.

Figure 1  Axial CT showing


periventricular and basal ganglia
calcifications. There is ventricular
dilatation and widened frontal
subarachnoid spaces

Figure 2  Axial CT brain (higher


level than in Figure 1) showing
more calcific foci

DISCUSSION
See under Case 3 (Page 321).
Neurological System Imaging 321
CASE 3

A 12-day-old female presenting with hydro­cephalus.

Figure 1  Axial CT
showing periventricular
calcifications with massive
ventriculomegaly

Figure 2  Axial CT (higher


level than Figure 1) showing
more periventricular
calcifications and dilated
lateral ventricles
322 A Teaching Atlas of Case Studies in Diagnostic Imaging

DISCUSSION
Periventricular and basal ganglia calcifications can result from many pathological processes. These
include: hypoxia, hypoglycemia, TORCH infections (Toxoplasmosis, Rubella, Cytomegalovirus, and
Herpes), and inborn errors of metabolism. Other causes of intra-cranial calcifications in children
include tuberous sclerosis, Sturge-Weber syndrome, arteriovenous malformations and intracranial
tumors.

FURTHER READING
1. Ansari MQ, Chincanchan CA, Armstrong DL. Brain calcification in hypoxic-ischemic lesions:
An autopsy review. Pediatr Neurol. 1990;6(2):94-101.
2. Ho VB, Fitz CR, Chuang SH, Geyer CA. Bilateral basal ganglia lesions: Pediatric differential
considerations. Radiographics. 1993;13:269-92.
3. Makariou E, Patsalides Ad. Intracranial calcifications. Applied Radiology. 2009;38(11).
4. Palmer PES, Reeder MM. The imaging of tropical diseases, with epidemiological, pathologic,
& clinical correlation. Springer-Verlag, New York. 2001;1:653-62; 2001;2:743-7.
Neurological System Imaging 323
CASE 4

A 16-year-old girl presented with intractable seizures which were refractory to medical therapy.
Physical examination was essentially normal. MRI of the brain with and without intravenous
gadolinium was performed.

Figures 1A and B  T1w axial


and T2w coronal images
show a large left parietal
mass which was isointense
with normal gray matter on
all pulse sequences. It shows
multiple areas of CSF signal
due to cortical infolding.
There is no mass effect, and
there was no enhancement
with contrast B
324 A Teaching Atlas of Case Studies in Diagnostic Imaging

Diagnosis: Gray matter heterotopia.

DISCUSSION
Gray matter heterotopia is a condition arising from arrest of normal migration of nerve cells to
the cortex. On imaging, they appear as nodular or broad areas of tissue characteristically
isointense with normal cerebral gray matter on all pulse sequences. They may be sub­ependymal,
focal subcortical or band type.

FURTHER READING
1. Abdulrazek AA, Kandell AY, Elsorogy LG, et al. Disorders of cortical formation: MR imaging
features. AJNR Am J Neuroradiol. 2009;30(1):4-11.
2. Barkovich AJ, Chuang SH, Norman D. MR of neuronal migrational anomalies. AJNR Am J
Neuroradiol. 1987;8:1009-17.
3. Barkovich AJ, Kjos BO. Grey matter heteroto­ pias: MR characteristics and correlation with
developmental and neurologic manifestations. Radiology. 1992;182:493-9.
4. Barckovich AJ, Kuzniecky RI. Gray matter heterotopias. Neurology. 2000;55(11):1603-8.
Neurological System Imaging 325
CASE 5

A 4-month-old male child presented with progressive hydrocephalus.

Figure 1  Transcranial
Doppler sonography
showed a large, well-defined
vascular midline structure,
consistent with a vein of
Galen aneurysm, with
associated hydrocephalus

Figure 2A
326 A Teaching Atlas of Case Studies in Diagnostic Imaging

Figures 2B and C
Figures 2A to C  MRI studies
of the brain show the large
flow void of the vein of
Galen malformation and
the gross hydrocephalus
with periventricular and
C cerebral atrophy

DISCUSSION
See under Case 6 (Page 327).
Neurological System Imaging 327
CASE 6

A 2-year-old girl with progressive enlargement of the head and delayed developmental milestone.
Imaging confirmed the presence of a vein of Galen aneurysmal malformation (VGAM).

Figures 1A and B  Sagittal


and coronal CT angiogram
(CTA) of the brain, showing
globular dilatation of the
vein of Galen and multiple
feeding anomalous vessels A B

Figure 2  3D volume
rendered CTA of the brain,
showing the feeding
pericallosal arteries to the
vein of Galen
328 A Teaching Atlas of Case Studies in Diagnostic Imaging

Figure 3  3D volume
rendered CTA of the brain,
showing the draining
infrafalcine vein from the
vein of Galen

Figure 4  Transcranial
Doppler sonogram, showing
the dilated VGAM, and
spectral waveform of one
of the feeding arteries
anteriorly, characterized by
continuous forward and low
resistance flow throughout
the cardiac cycle
Neurological System Imaging 329

Figure 5  Transcranial
Doppler sonogram, showing
the spectral wave of the
VGAM, demonstrating
to-and-fro pattern of flow
(Courtesy: Dr Musa Tabari)

DISCUSSION
Vein of Galen aneurysmal malformations (VGAM) are midline structures, extending from the
interventricular foramen to the choroidal fissure, and laterally to the atria.
The embryonic precursor to the normal vein of Galen is the median prosencephalic vein,
which has arteriovenous fistulous connections within the primitive vascular bed. The fistulous
connections normally involute between fifth and seventh weeks of intrauterine development,
and by 3 months of development, the posterior part of the median prosencephalic vein joins
the internal cerebral veins and the basal veins to form the vein of Galen. Persistence of the
median prosencephalic vein and its primitive arteriovenous connections leads to a true vein of
Galen malformation.
The vein of Galen malformation is a form of AVM in which the venous component is the enlarged
vein of Galen and the arteries drain either directly into the vein of Galen or into veins that empty
into the vein of Galen, such as the internal cerebral vein or basal vein of Rosenthal. Affected
children are diagnosed as neonates or infants with heart failure, hydrocephalus or both. The arterial
feeders are from the choroid arteries. The anterior cerebral and transmesencephalic arteries are
330 A Teaching Atlas of Case Studies in Diagnostic Imaging

often involved. Associated venous anomalies are common involving stenosis or progressive
occlusion of the straight, transverse and sigmoid sinuses. Diagnosis is by Doppler ultrasound, CT,
CT angio and MRI. Treatment is by transarterial/transvenous embolization or combined operative
or ultrasound-guided transtorcular endovascular methods.

FURTHER READING
1. Alvarez H, Garcia Monaco R, Rodesh G, Sachet M, Krings T, Lasjaunias P. Vein of Galen aneurysmal
malformations. Neuroimaging Clin N Am. 2007;17(2):189-206.
2. Jones BV, Ball WS, Tomsick TA, et al. Vein of Galen aneurysmal malformation: Diagnosis and
treatment of 13 children with extended clinical follow up. Am J Neuroradiol. 2002;23:1717-24.
3. Raybaud CA, Strother CM, Hald JK. Aneurysms of the vein of Galen: Embryonic considerations
and anatomical features relating to the pathogenesis of the malformation. Neuroradiology.
1989;31(2):109-28.
Neurological System Imaging 331
CASE 7

A 22-year-old female with recurrent headaches.

Figure 1  Axial T1w


MRI showing a large
hypointense right
cerebellopontine angle
mass producing
extrinsic compression of
the adjacent brain

Figure 2  T1w right


parasagittal image
showing the mass
332 A Teaching Atlas of Case Studies in Diagnostic Imaging

Figure 3  Coronal T2w


image showing the lesion
to be uniformly hyperintense

Diagnosis: Cerebellopontine angle arachnoid cyst.

DISCUSSION
See under Case 9 (Page 336).
Neurological System Imaging 333
CASE 8

A 17-year-old female presented with recurrent seizures.

Figure 1  Axial T1w MRI


showing a large, uniformly
hypointense lesion in
the left temporal fossa

Figure 2  Axial T1w image


(higher level than Figure 1),
showing the lesion
indenting the adjacent brain
334 A Teaching Atlas of Case Studies in Diagnostic Imaging

Figure 3  Axial T2w image


showing a uniformly
hyperintense lesion,
consistent with a cyst

Figure 4  Coronal T2w


image showing
another view of the cyst
Neurological System Imaging 335

Figure 5  Left parasagittal


T1w image demonstrating
the cyst

Diagnosis: Temporal fossa arachnoid cyst.

DISCUSSION
See under Case 9 (Page 336).
336 A Teaching Atlas of Case Studies in Diagnostic Imaging
CASE 9

A 27-year-old female complained of persistent frontal headaches for a year. She had no history of
trauma, and no other associated symptom. Physical examination was normal. A CT of the brain was
done, followed by MRI.

Figures 1A and B  A
well-defined extra-axial
homogenous mass with
attenuation characteristics
of CSF is shown in the right
frontal region. There is
thinning and remodeling of
B the overlying skull vault
Neurological System Imaging 337

Figures 2A and B  T1w and


T2w MRI axial images
show the lesion to be
isointense to CSF.
The appear­ances are typical
of an arachnoid cyst B
338 A Teaching Atlas of Case Studies in Diagnostic Imaging

DISCUSSION
Arachnoid cysts arise within a split in the arachnoid membrane, and are largely develop­mental, but
a few may be secondary to trauma or previous meningitis. They constitute about 1% of intracranial
masses. Their most common sites are the anterior and middle cranial fossa, parasellar region and
the convexities. In the posterior fossa, they are commonly seen in the cerebellopontine angle
and the quadrigeminal cistern. They displace brain tissue and cause thinning and remodeling of the
overlying skull vault.
The vast majority of cases are asymptomatic, and are discovered incidentally. They may, however
present with headaches, seizures and symptoms related to raised intracranial pressure. Association
with psychiatric disorders is controversial.

FURTHER READING
1. Gelabert-Gonzalez M. Intracranial arachnoid cysts. Rev Neurol. 2004;39(12):1161-6.
2. Weber F, Knopf H. Incidental findings in magnetic resonance imaging of the brains of healthy
young men. J Neurol Sci. 2006;240(1-2):81-4.
3. Vakis AF, Koutentakis DI, Karabetsos DA, Kalostos GN. Psychosis-like syndrome associated with
intermittent intracranial hypertension caused by a large arachnoid cyst of the left temporal lobe.
Br J Neurosurg. 2006;20(3):156-9.
Neurological System Imaging 339
CASE 10

A 1-year-old boy presented with sudden onset of vomiting, diminished alertness and low-grade
fever.

Figure 1  Coronal fluid-


attenuated inversion
recovery (FLAIR)
MRI showing hypersignal
intensity of the entire right
cerebellum, adjoining
occipital lobe, and
periventricular white matter

Figure 2  Axial T2w image


of same patient
340 A Teaching Atlas of Case Studies in Diagnostic Imaging

Figure 3  Diffusion-
weighted image of
the same patient

Diagnosis: Massive right cerebellar infarct.

DISCUSSION
Cerebellar infarcts are rare in children, but have been reported as a consequence of perinatal
ischemic anoxic injury, sepsis, vasculopathies, fibromuscular dysplasia of the vertebral arteries,
or due to cardioembolic disease.

FURTHER READING
1. Barinagarrementeria F, Amaya LE, Cantu C. Causes and mechanisms of Cerebellar infarction in
young patients. Stroke. 1997;28:2400-4.
2. Lin JJ, Lin KL, Chou ML, et al. Cerebellar infarction in the territory of the Superior cerebellar artery
in Children. Pediatr Neurol. 2007;37(6):435-7.
3. Mercuri E, He J, Walter L, et al. Cerebellar infarction and atrophy in infants and children with
a history of premature birth. Pediatr Radiol. 1997;27(2):139-43.
Neurological System Imaging 341
CASE 11

A 10-year-old male presented with headache and seizures.

Figure 1  Axial T1w


MRI showing a right
frontoparietal mass
with foci of high signal
intensity, suggesting acute
hemorrhage

Figure 2  Coronal image of


same patient
342 A Teaching Atlas of Case Studies in Diagnostic Imaging

Figure 3  Axial T2w image


showing tubular signal
voids within the mass

Figure 4  MRI cerebral


angiogram confirming an
arteriovenous malformation
Neurological System Imaging 343

DISCUSSION
Arteriovenous malformations (AVM) may present with headache and seizures, but in more
than 50% of cases, especially in children, they may present with consequences of hemorrhage.
These include sudden severe headache, nausea and vomiting and loss of consciousness. AVMs
may be diagnosed by CT or MRI, but catheter angiography still remains the gold standard for
diagnosis, with the additional advantage that embolization can be attempted at the same seating in
suitable cases.

FURTHER READING
1. Di Rocco C, Tamburrini G, Rollo M. Cerebral arteriovenous malformations in children. Acta
Neurochir. 2000;142:145-58.
2. Kondziolka D, Humphreys RP, Hoffman HJ, et al. Arteriovenous malformations of the brain in
children: A forty year experience. Can J Neurol Sci. 1992;19(1):40-5.
344 A Teaching Atlas of Case Studies in Diagnostic Imaging
CASE 12

A 50-year-old male presented with recent onset tonic-clonic seizures.

Figure 1  Axial T1w MRI


shows a focus of low
signal intensity in the left
frontoparietal cortex

Figure 2  T1w parasagittal


image of the same lesion
Neurological System Imaging 345

Figure 3  T2w image showing


the lesion to be hyperintense,
with a broad dural base

Figure 4  Postcontrast study


shows profuse enhancement
of the mass, with a classical
“dural tail” and surrounding
edema

Diagnosis: Meningioma.

DISCUSSION
See under Case 15 (Page 350).
346 A Teaching Atlas of Case Studies in Diagnostic Imaging
CASE 13

A 30-year-female previously diagnosed with type 2 neurofibromatosis.

Figure 1  Sagittal T1w


contrast MRI showing
multiple enhancing
cerebral, cerebellar and
pontine meningiomas

Figure 2  Axial image of


multiple meningiomas.
Note the parasagittal
distribution of the lesions
Neurological System Imaging 347

Figure 3  Sagittal cervical


contrast MRI showing
enhancing lesions in
the pons, medulla
oblongata and cervical cord

DISCUSSION
See under Case 15 (Page 350).
348 A Teaching Atlas of Case Studies in Diagnostic Imaging
CASE 14

A 62-year-old female with recurrent headache.

Figure 1  Axial T1w


MRI showing a right
cerebellopontine angle
mass, slightly hypointense
relative to the adjacent
brain

Figure 2  Axial T2w MRI


showing the CP angle mass
to be of high signal intensity
Neurological System Imaging 349

Figure 3  Axial T1w


postcontrast image showing
profuse enhancement of the
mass. Note the characteristic
dural tail over the lateral
aspect of the petrous ridge
and extending into the internal
auditory canal

Figure 4  Coronal postcontrast


T1w MRI showing
the enhancing CP angle mass

Diagnosis: Cerebellopontine angle meningioma.

DISCUSSION
See under Case 15 (Page 350).
350 A Teaching Atlas of Case Studies in Diagnostic Imaging
CASE 15

A 32-year-old male presenting with severe head­ache and seizures.

Figure 1  T1w axial image


showing a large parasagittal
left frontal convexity
mass producing midline
shift to the right and
hydrocephalus. Note the
peripheral flow void from
hypervascularity

Figure 2  T1w parasagittal


image of the same patient
Neurological System Imaging 351

Figure 3  T2w axial image of


the mass showing internal
foci of high signal intensity,
suggesting areas of necrosis

Figure 4  Postcontrast
image showing profuse
contrast enhancement.
Central areas of low signal
intensity indicate necrosis
352 A Teaching Atlas of Case Studies in Diagnostic Imaging

Diagnosis: Malignant meningioma.

DISCUSSION
Meningiomas are the most common extra-axial tumors in adults, with a peak age incidence between
50 and 60 years. They are usually slow-growing, and frequently located in the parasagittal areas
and in the convexities of the brain, but also occur in the sphenoid wing, parasellar areas, olfactory
groove, and other locations, including intraspinal.
Plain skull features include focal hyperosto­sis and prominent vascular markings from enlarged
middle meningeal arteries.
Meningiomas are hyperdense on CT, often with calcific foci and surrounding edema, with
profuse contrast enhancement.
On MRI, they are hypointense to gray matter on T1w and isointense to hyperintense on T2w
sequences, with profuse enhancement. Meningiomas have a broad dural base with 60% showing a
characteristic “tail” from dural thickening.
Malignant meningiomas are more aggres­ sive, constituting some 10% of meningiomas.
Neurofibromatosis type 2 is associated with meningiomas.

FURTHER READING
1. Ferner RE: Neurofibromatosis 1 and neurofibromatosis 2—a twenty first century perspective.
Lancet Neurology. 2007;6(4):340-51.
2. Kizana E, Lee R, Young N, et al. A review of the radiological features of intracranial meningiomas.
Australas Radiol. 1996;40(4):454-62.
3. Ricci PE: Imaging of adult brain tumors. Neuroimaging. Clin North Am. 1999;9:651-69.
4. Smirniotopulous JG: Extra-axial masses of the central nervous system. In: Smirniotopulous JG (Ed).
Categorical course in Diagnostic Radiology: Neuroradiology. Oak Brook, Ill, Radiological Society of
North America; 2000. pp. 123-32.
Neurological System Imaging 353
CASE 16

A 34-year-old male, presented with left-sided hearing loss, tinnitus and vertigo.

Figure 1  T2w MRI


cerebellopontine angle
showing an oval mass
occupying the left internal
auditory canal

Figure 2  T1w postcontrast


MRI showing the mass
enhancing profusely
with contrast
354 A Teaching Atlas of Case Studies in Diagnostic Imaging

Figures 3A and B  Another


patient with right
intracanalicular vestibular
schwannoma; axial and
coronal MR images showing
B postcontrast enhancement
Neurological System Imaging 355

Diagnosis: Intracanalicular vestibular schwannomas.

DISCUSSION
Vestibular schwannoma (acoustic neuroma) is a benign tumor arising from the myelin-forming
cells of the vestibular portion of the 8th cranial nerve. Patients present with sensorineural hearing
loss, vertigo, tinnitus, nausea and vomiting. The tumor arises within the internal auditory canal,
but may extend to the cerebellopontine angle. MRI is the imaging modality of choice, with the
tumor enhancing profusely with gadolinium, although false positive cases have been reported.
Conservative treatment is usually adopted in the elderly, but surgery and radiation (“gamma knife”)
therapy are adopted in others.

FURTHER READING
1. Curtin HD, Hirsch WL. Imaging of acoustic neuromas. Neurosurg Clin N Am. 2008;19:175-205.
2. House JW, Bassim MK, Schwartz M. Falsepositive magnetic resonance imaging in the diagnosis of
vestibular schwannoma. Otol and Neurotol. 2008;29:1176-8.
3. Rosenberg SI. Natural history of acoustic neuromas. Laryngoscope. 2000;110(4):497-508.
356 A Teaching Atlas of Case Studies in Diagnostic Imaging
CASE 17

A 28-year-old male presented with recurrent headaches.

Figure 1  Axial CT showing


a large right frontal
hypodense mass producing
mid-line shift to the left,
with an attenuation
coefficient of –32 HU

Figure 2  Sagittal
reformatted image of
same patient
Neurological System Imaging 357

Figure 3  Coronal
reformatted image
of the same patient.
Note the fluid level

Diagnosis: Giant intracerebral lipoma.

DISCUSSION
Intracranial lipomas are uncommon, with disputed origin. Most are small, asymptomatic and
discovered incidentally, but may present with headache and seizures. Their lipid content is obvious
on CT or MRI, where they show characteristic high signal intensity, becoming hypointense on fat
saturated sequences.

FURTHER READING
1. Britt PM, Bindal AK, Balko MG, Yeh HS. Lipoma of the cerebral cortex: Case Report. Acta Neurochir.
1993;121(1-2):88-92.
2. Ramirez-Zamora A, Asconape J. Intracranial lipomas: Radiographic and clinical characteristics.
Int J Neuro. 2009;12(1).
3. Truwit CL, Barkovich AJ. Pathogenesis of intracranial lipoma: An MR study in 42 patients. AJR Am J
Roentgenol. 1990;155(4):855-64.
358 A Teaching Atlas of Case Studies in Diagnostic Imaging
CASE 18

A 30-year-old male with recent onset epilepsy.

Figure 1  Axial T1w MRI


showing an oval hypointense
nodule in the left temporal
lobe

Figure 2  Left parasagittal


T1w view of same patient
showing the temporal lobe
lesion
Neurological System Imaging 359

Figure 3  Axial T2w image


showing the nodule to be
hyperintense

Figure 4  Coronal FLAIR


sequence showing
the nodule to be of high
signal intensity
360 A Teaching Atlas of Case Studies in Diagnostic Imaging

Figure 5  Axial T1w


postcontrast image showing
tumor enhancement

Diagnosis: Ganglioglioma
[Dysembryoplastic neuroepithelial tumor (DNET) is a differential].

DISCUSSION
Gangliogliomas are slow-growing neuro­epithelial tumors, with a peak incidence in children and
young adults. They typically present as solid enhancing tumors in the temporal lobe in young
patients with com­plex partial seizures. The tumors are well circum­scribed, and surrounding edema
is unusual. Seizures from temporal lobe gangliogliomas are surgically curable.

FURTHER READING
1. Adachi Y, Yagishita A. Gangliogliomas: Characteristic imaging findings and role in the temporal
lobe epilepsy. Neuroradiology. 2008;50(10):829-34.
2. Park SH, Kim E, Son EI. Cerebellar ganglioglioma. J Korean Neurosurg Soc. 2008;43(3):165-8.
Neurological System Imaging 361
CASE 19A

A 9-year-old boy presented with headache, dizziness, vomiting and an unsteady gait. MRI of
the brain was done.

Figure 1  T1w parasagittal


image shows a large well-
defined hypointense left
cerebellar mass with cystic
areas and dilated lateral
ventricles

Figure 2  T2w axial images


show the mass in the left
cerebellar hemisphere.
It is hyperintense and shows
cystic areas. The fourth
ventricle is compressed, and
the third and lateral ventricles
are dilated
362 A Teaching Atlas of Case Studies in Diagnostic Imaging

Figure 3  Postcontrast
image shows moderate
enhancement with
contrast. The appearances
are consistent with
cerebellar astrocytoma,
but medulloblastoma is also
a possibility

DISCUSSION
Medulloblastoma, pilocystic astrocytoma and atypical teratoid/rhabdoid tumor are the most
common intra-axial cerebellar tumors in children. Hemangioblastoma is uncommon in children
except in von Hippel–Lindau disease. Other posterior fossa tumors include brainstem glioma,
dermoid and epidermoid, meningiomas and skull base lesions such as glomus tumor, histiocytosis
X and Ewing’s sarcoma.
Medulloblastomas are slightly more common than astrocytomas and are common in boys.
They typically present on CT as a hyperdense midline vermian mass. They are typically cystic with
an enhancing nodule. Cerebellar tumors present with headache, vomiting and ataxia.

FURTHER READING
1. Borkovich AJ. Brain tumors of childhood. In Paediatric Neuroimaging, 2nd edition. Raven Press,
New York; 1995. pp. 321-437.
2. Griffiths PD. A protocol for imaging paediat­ ric brain tumors. United Kingdom Children’s
Cancer Study Group (UKCCSG) and Societe Francaise D’onclogie Pediatrique (SFOP). Clin Oncol
(r. Coll Radiol). 1999;11:290-4.
3. Kestle J, Townsend JJ, Brockmeyer DL, Walker ML. Juvenile pilocytic astrocytoma of the brainstem
in children. J Neurosurg. 2004;101(1):1-6.
4. Wilne S, Coller J, Kennedy C, et al. Presentation of childhood CNS tumours: A systematic review
and meta-analysis. Lancet Oncol. 2007;8(8):865-95.
Neurological System Imaging 363
CASE 19B

A 7-month-old male child presented with persistent vomiting and bulging fontanels.

Figures 1A and B B
364 A Teaching Atlas of Case Studies in Diagnostic Imaging

Figure 1C
Figures 1A to C  Plain axial
CT images show a large
hyperdense, lobulated
cauliflower mass in the
atrium of the left lateral
ventricle. There is marked
associated hydrocephalus
with transependymal
interstitial edema.
There is no parenchymal
brain invasion
C (Courtesy: Dr Binu Jose)

DISCUSSION
Choroid plexus tumors arise from the neuroepithelial lining of the ventricular choroid plexus.
They are the most common brain tumors in children under 1 year, and represent 2–5% of brain
tumors in childhood. They are more frequently benign (papilloma) than malignant (carcinoma).
Most tumors originate in the atria of the lateral ventricle. Fourth ventricle tumors are unusual
in children. They present with hydrocephalus due to overproduction of CSF, or obstruction due
to hemorrhage, arachnoiditis or carcinomatosis in carcinomas. They are lobulated, frond-like,
isodense or slightly hyperdense on CT and show avid homogeneous enhancement after intravenous
contrast administration. On MRI, they are isointense or slightly hyperintense on T1w, hyperintense
on T2w, and enhance intensely after IV gadolinium. Calcifications are uncommon. Choroid plexus
carcinomas invade the ependyma and extend to the brain parenchyma. They frequently show
necrosis and surrounding vasogenic edema. Dissemination within the ventricular and CSF spaces
is not uncommon.

FURTHER READING
1. Coates TL, Hinshaw DB, Peckman N, et al. Paediatric choroid plexus neoplasms: MR, CT and
pathologic correlation. Radiology. 1989;173:81-8.
2. Packer RJ, Perilongo G, Johnson D, et al. Choroid plexus carcinoma of childhood. Cancer.
1992;69:580-5.
3. Wolff JE, Sajedi M, Brant R, et al. Choroid plexus tumours. Br J Cancer. 2002;87(10):1086-91.
Neurological System Imaging 365
CASE 20

A 22-year-old female presented with headaches, lethargy and vomiting for two weeks followed by
an ataxic gait. Plain and contrast-enhanced CT of the brain were done.

Figure 1  Plain axial CT


shows a large hyperdense
midline vermian mass
abutting and compressing
the roof of the fourth
ventricle, with mild
perilesional edema and
dilated third and lateral
ventricles. Multiple foci of
calcifications are seen
within the mass.
There was patchy
enhancement after
intravenous contrast
administration.
The appearances are
typical of medulloblastoma

DISCUSSION
Medulloblastomas are very malignant, small round cell tumors that may be associated with some
rare oncogenetic disorders such as Gorlin’s or basal cell nevus syndrome, Turcot or Cowden
syndrome. They are aggressive, high grade tumors that may show intracranial or intraspinal
leptomeningeal metastasis. They commonly present from 7 years to late childhood, but can occur
rarely in adults, as in this case. They are hyperdense on CT and hypointense on T2w, with restricted
diffusion on diffusion weighted imaging. Other posterior fossa tumors with imaging features that
may suggest medulloblastoma include primitive neuroectodermal tumor or atypical teratoid
tumor, choroid plexus carcinoma, chondrosarcoma, Ewing’s sarcoma, chordoma and lymphoma.

FURTHER READING
1. Atlas SW. Intra and extra-axial brain tumors. In Magnetic Resonance Imaging of the Brain and
Spine, Raven Press, New York; 1991. pp. 223-318.
2. Keller KK, Rushing EJ. From the Archives of the AFIP. Medulloblastoma: A comprehen­sive review
with Radiologic-Pathologic correlation. Radiographics. 2003;23:1613-37.
366 A Teaching Atlas of Case Studies in Diagnostic Imaging
CASE 21

A 33-year-old male was referred from the accident and emergency department with head and facial
injuries sustained in a car accident.

Figure 1  Axial CT brain shows


extensive intracerebral air
bubbles. Note soft-tissue
swelling over the left temporal
bone. An underlying fracture
was shown on bone window

Figure 2  Axial CT at the level


of the orbits. There are fractures
of the lateral orbital walls and
right wall of the sphenoid sinus.
The sphenoid and left ethmoids
are opaque from hemorrhage.
Note left orbital emphysema
and proptosis
Neurological System Imaging 367

DISCUSSION
Pneumocephalus results most frequently from craniofacial trauma, especially involving the sinuses
and orbits, but can be iatrogenic or from cranial tumors. The gas can be localized in the subdural,
arachnoid, or epidural spaces, or be intraventricular or intraparenchymal, as in this patient. Tension
pneumocephalus is a situation where gas collects increasingly in the subdural space through a
ball-valve phenomenon, creating pressure on the underlying brain, and can constitute a
neurosurgical emergency.
The Mount Fuji sign is the CT appearance of bilateral subdural pneumocephalus along
the frontal lobes, producing compression of the frontal lobe parenchyma, tapering anteriorly.
The appearance mimics Mount Fuji in profile, hence the sign.

FURTHER READING
1. Mauriello JA, Lee HJ, Nguyen L. CT of soft tissue injury and orbital fractures. Radiologic Clin North
Am. 1999;37(1):241-52.
2. Michel SJ, The Mount Fuji Sign. Radiology. 2004;232:449-50.
3. Ozturk E, Kantarci M, Karaman K, et al. Diffuse pneumocephalus associated with infratentorial
haemorrhages as a compli­cation of spinal surgery. Acta Radiologica. 2006;47(5):497-500.
4. Schirmer CM, Heilman CB, Bhardwaj A. Pneumocephalus: Case illustrations and Review.
Neurocritical care. 2010;13(1):152-8.
368 A Teaching Atlas of Case Studies in Diagnostic Imaging
CASE 22

A 23-year-old male was involved in a high velocity road traffic accident and was admitted with
altered consciousness. Emergency CT brain was carried out.

Figure 1  Plain axial CT


image shows a large right
frontal extradural hematoma,
compression of the anterior
horn of the lateral ventricle
with subfalcine herniation
and slight midline shift

Figure 2A
Neurological System Imaging 369

Figures 2B and C
Figures 2A to C  Axial,
coronal and parasagittal
reformatted CT images
of subacute subdural
hematoma in another
patient C
370 A Teaching Atlas of Case Studies in Diagnostic Imaging

DISCUSSION
Extradural hematomas are seen on CT sections as biconcave dense areas immediately beneath
the skull vault, and often lie beneath a fracture. Areas of low density within them may indicate
continuing bleeding. Frontal, vertical and posterior fossa hematomas may be difficult to diagnose
and coronal images may be required. Wide window settings and MRI may be very helpful. Subdural
hematomas usually result from tears to cortical veins, and appear as crescent-shaped extra-axial
hyperdense collections, which change in density with the age of the bleed.

FURTHER READING
1. Barr RM, Gean AD, Le TH. Craniofacial trauma. Fundamentals of Diagnostic Radiology. In: Brandt
WE, Helms CA (Eds), Lippincott Williams & Wilkins; 2007. pp. 57-62.
2. Making the best of a Department of clinical Radiology: Guidelines for doctors. RCR London.
Royal College of Radiologists; 2007.
3. Teasdale G, Teasdale E, Hadley D. Computed tomography and magnetic resonance imaging
classification of head injury. J Neurotrauma. 1992;9:249-57.
Neurological System Imaging 371
CASE 23

A 65-year-old male presented with partial complex seizures which were refractory to medical
treatment. Neurological examination was essentially normal. MRI examination was done.

Figure 1  The FLAIR coronal


image shows atrophy of
the left temporal lobe with
widening of the ipsilateral
Sylvian fissure, and a high-
signal intensity focus in the
involved temporal lobe.
The appearances are
consistent with mesial
temporal sclerosis (MTS)

DISCUSSION
Mesial temporal sclerosis is the most common cause of partial complex seizures arising from
the temporal lobe. The others are arteriovenous malformations, and benign and malignant tumors.
In this condition, neuronal loss and gliosis occur in the hippocampal formation. MRI shows
atrophy (decreased volume) of the lobe and increased signal on T2-weighted images. Associated
limbic system findings include ipsilateral atrophy of the fornix and mamillary body, and increased
signal intensity of the anterior temporal lobe cortex. Quantified T2 relaxation times are more
sensitive for MTS than visual relaxation inspection of the T2-weighted images.

FURTHER READING
1. Bronen RA. Epilepsy: the role of MR imaging. Am J Roentgenol. 1991;159:165-74.
2. Jack CR, Knecke KN, Letetner PH, et al. Diagnosis of mesial temporal sclerosis with conventional
versus fast spin-echo MR imaging. Radiology. 1994;192:123-7.
3. Jack CR, Rydberg CH, Krecke KN, et al. Mesial temporal sclerosis: Diagnosis with fluid-attenuated
inversion-recovery versus spin-echo MR imaging. Radiology. 1996;199(2):367-73.
4. Provenzale JM, Barboriak DD, VanLand­ingham K, et al. Hippocampal MRI signal hyperintensity
after febrile status epilepticus is predictive of subsequent mesial temporal sclerosis. AJR Am J
Roentgenol. 2008;190(4):976-83.
372 A Teaching Atlas of Case Studies in Diagnostic Imaging
CASE 24

A 34-year-old female presented with recurrent episodes of fatigue, unstable gait and blurred vision.
Clinical studies and CSF analysis suggested a diagnosis of multiple sclerosis. MRI of the brain and
spine were done.

Figures 1A and B  MRI brain


showing multiple hyperintense
white matter lesions in the
periventricular regions, corona
radiata, and centrum semiovale in
the FLAIR sequences. The lesions
are perpendicular to the corpus
callosum (Dawson’s fingers) on the
T2-weighted sagittal image.
A few of the lesions are rounded,
well-defined and tumefactive.
Lesions are seen in the cerebellar
B peduncles and brainstem

DISCUSSION
See under Case 25 (Page 373).
Neurological System Imaging 373
CASE 25

Figure 1  Cervical cord


involvement is shown
in another patient with
multiple sclerosis. Note a
hemangioma in body of C4

DISCUSSION
Multiple sclerosis (MS) is a CNS inflammatory disease characterized by demyelination and axonal
injury. The most common form relapses and remits, and patients have episodes of neurological
deficits and partial or complete recovery. Lesions are seen better on FLAIR and PD sequences,
but FLAIR is less sensitive for infratentorial lesions. Spinal cord involvement is common. Acute
lesions may show solid or ring enhancement. Large acute tumefactive lesions may mimic glioma.
The differential diagnosis of MS includes acute disseminated encephalomyelitis (ADEM), vasculitis,
sarcoidosis and small vessel ischemia.

FURTHER READING
1. Pretorius PM, Quaghebear G. The role of MRI in the diagnosis of MS. Clin Radiol. 2003;58:434-48.
2. Ye G. Multiple sclerosis: The role of MR imaging. Am J Neuroradiol. 2006;27:1165-76.
374 A Teaching Atlas of Case Studies in Diagnostic Imaging
CASE 26

A 6-year-old female with recurrent left otitis media presented with recent onset of headache and
neck stiffness.

Figure 1  Axial CT brain


showing a large ring shadow
in the left temporal lobe with
surrounding hypodensity

Figure 2  Postcontrast
image showing marked
enhancement of the ring
shadow with a central
homogenous hypodensity and
surrounding edema
Neurological System Imaging 375

Figure 3  Postcontrast
coronal image of
the same patient

Diagnosis: Brain abscess.

DISCUSSION
Brain abscesses may arise from the spread of infection from sinusitis, dental and scalp infections,
or otitis media. The latter is thought to be the source of infection in this patient. A brain abscess
can also result from hematogenous spread from bacterial endocarditis or generalized sepsis.
The typical brain abscess shows a central hypodense center with postcontrast rim enhancement.
It has to be differentiated from cystic or necrotic brain tumors or radiation necrosis.
A brain abscess may be complicated by daughter abscesses, or it may rupture into the ventricular
system, resulting in ventriculitis.

FURTHER READING
1. Glickstein JS, Chandra RK, Thompson JW. Intracranial complications of pediatric sinusitis.
Otolaryngol Head Neck Surg. 2006;134(5):733-6.
2. Haimes AB, Zimmerman RD, Morgello S, et al. MR Imaging of brain abscesses. AJR Am J Roentgenol.
1989;152(5):1073-85.
3. Lai PH, Ho JT, Chen WL, et al. Brain abscess and necrotic brain tumor: Discrimination with Proton
MR spectroscopy and Diffusion weighted imaging. AJNR Am J Neuroradiol. 2002;23:1369-77.
4. Muzumdar D, Jhawar S, Goel A. Brain abscess: An overview. Int J Surg. 2011;9(2):136-44.
376 A Teaching Atlas of Case Studies in Diagnostic Imaging
CASE 27A

A 35-year-old female was brought to emergency room with sudden, severe headache followed by
confusion and loss of consciousness. A plain CT brain showed fresh hemorrhage in the cerebral
sulci and basal cisterns. She was thought to have either an AVM or cerebral aneurysm. She was
prepared for MR angiography.

Figure 1  Coronal FLAIR brain


MR image shows hypersignal
intensities in the sulci and right
lateral ventricle, consistent with
hemorrhage. Periventricular
and deep white matter high
intensity lesions may be due to
small vessel ischemia

Figure 2A
Neurological System Imaging 377

Figure 2B
Figures 2A and B  Cerebral
MR angiogram with
axial collapsed and
right sagittal views
showing a tip aneurysm
of the basilar artery
378 A Teaching Atlas of Case Studies in Diagnostic Imaging
CASE 27B

A 45-year-old woman with headaches.

Figure 1  Lateral 3D cerebral


computed tomographic
angiogram, showing
anterior cerebral artery
saccular aneurysm at
the junction of A2 and A3
segments

Figure 2  Corresponding
lateral digital subtraction
angiogram (DSA) of Figure 1,
showing the anterior
cerebral artery saccular
aneurysm at the junction
of pericallosal and the
callosomarginal arteries
Neurological System Imaging 379

Figure 3  Frontal cerebral


computed tomographic
angiography demonstrating
anterior cerebral artery
and middle cerebral artery
saccular aneurysms

Figure 4  Corresponding
digital subtraction
angiogram (DSA) of Figure 3
above, showing saccular
aneurysm at the junction of
M1 and M2 segments of the
left middle cerebral artery
(Courtesy: Dr Musa Tabari)
380 A Teaching Atlas of Case Studies in Diagnostic Imaging

DISCUSSION
Cerebral aneurysms can arise from the internal carotid artery or any of its branches, or from the
vertebral artery, basilar artery, or any of their branches. The most common sites of occurrence
of intracranial aneurysms are around the anterior communicating artery (30–35%), the posterior
communicating artery (30–35%), the middle cerebral artery bifurcation (20%), and the basilar
artery (5%).
Intracranial aneurysms are classified into saccular (berry) and nonsaccular types on the basis
of their shape and etiology.
Saccular or berry aneurysms have several anatomic characteristics that distinguish them
from other types of intracranial aneurysms. Typically, saccular aneurysms arise at a bifurcation
(see Figs 1 to 4) or along a curve of the parent vessel, or they point in the direction in which flow
would proceed if the curve were not present.
Aneurysms may manifest with subarachnoid hemorrhage (SAH), thromboembolic events, or
mass effect. SAH is associated with significantly higher mortality.
CT angiography (CTA) performed on multidetector row scanners is convenient for evaluating
patients with suspected aneurysm. Most commonly the images are acquired in the axial plane
to generate reconstructed multiplanar reformatted (MPR) or 3D rendered images. CTA may also
show ongoing extravasation of contrast material, indicating hemorrhage or rehemorrhage in cases
of ruptured aneurysm. The limitations of CTA include poor sensitivity for lesions involving the
carotid artery at the skull base or within the contrast-filled cavernous sinuses, where MRI has an
advantage.
Treatment options consist of surgical clip­ping or endovascular coiling.

FURTHER READING
1. Brisman JL, Song JK, Newell DW. Cerebral aneurysms. N Engl J Med. 2006;355:928-39.
2. Hacein-Bey L, Provenzale JM. Current imaging assessment and treatment of intracranial aneurysms.
AJR Am J Roent­genol. 2011;196(1):32-44.
Neurological System Imaging 381
CASE 28

A 34-year-old old female presented with recurrent discomfort, discharge and severe hearing
impairment affecting the left ear since childhood.

Figures 1A and B  T1w


and T2w axial MR images
of the nasopharynx show
a mass with low signal
intensity on T1w and high
on T2w, at the lateral wall
of the left nasopharynx.
Its location and
characteristics are indicative
of Eustachian tube dermoid B
382 A Teaching Atlas of Case Studies in Diagnostic Imaging

DISCUSSION
The Eustachian tube (pharyngotympanic tube) links the middle ear to the nasopharynx and drains
mucus from it. Obstruction of the tube produces stasis and can predispose to otitis media. Dermoid
cyst of the Eustachian tube is a congenital abnormality, and its location at the parapharyngeal
end of the tube produces obstruction and recurrent ear infection from childhood. It is one of the
recognized causes of deafness in children.
Eustachian tube dermoid is rare. Most of the reported cases are in females, and on the left side,
as in this patient.

FURTHER READING
1. Gourin CG, Sofferman RA. Dermoid of Eustachian tube. Otolaryngol Head Neck Surg.
1999;120(5):772-5.
2. Kollias SS, Ball WS, Prenger EC, Meyers CM. Dermoids of the Eustachian tube: CT and MR findings
with histologc correlation. AJNR Am J Neuroradiol. 1995;16(4):663-8.
3. Sichel JY, Dano I, Halperin D, Chisin R. Dermoid cyst of the Eustachian tube. Int J Pediatr
Otolaryngol. 1999;48(1):77-81.
Neurological System Imaging 383
CASE 29

A 28-year-old male presented with fever, headaches, and focal neurological deficits. He was negative
for HIV.
CT brain showed ill-defined low attenuation areas in the right temporal and both parieto-
occipital areas. The lesions were better demonstrated on MRI (see Figs 1 to 2) which show multiple,
rounded ring enhancing lesions in the corticomedullary junction of the parieto-occipital lobes
on both sides and in the right temporal lobe. Basilar meningeal enhancement with multiple ring
enhancing lesions were seen in the sellar region.

Figures 1A and B  Axial contrast


MRI at different levels B
384 A Teaching Atlas of Case Studies in Diagnostic Imaging

Figure 2  Right parasagittal


contrast MRI Diagnosis:
Tuberculous meningitis with
parenchymal tubercu­loma

DISCUSSION
Tuberculosis of the central nervous system occurs in 5% of cases of tuberculosis. It is common
in patients younger than 20 years of age. The chest radiograph is abnormal in 45–60% of cases.
Meningitis is the most frequent manifestation of CNS tuberculosis, with involvement of the basal
meninges. CT shows obliteration of the cisterns by isodense or slightly hyperdense exudates,
and diffuse meningeal enhancement after IV contrast medium injection. Communicating
hydrocephalus and infarction of the basal ganglia and internal capsule may result due to arteritis
of the penetrating arteries. MRI is more sensitive in depicting these changes. Tuberculomas
(parenchymal granulomas) are seen most often in the gray/white matter interphase. They are
small, rounded isodense or hypodense lesions with a variable amount of surrounding edema, and
with homogenous or ring enhancement. Tuberculomas may calcify, but this is uncommon. Ring
enhancing lesions are not specific to tuberculosis, but may be seen in cysticercosis, toxoplasmosis,
metastasis, glioblastoma, resolving hematoma, and lymphoma.

FURTHER READING
1. Bernaerts A, Vanhoenacker F, Debois V, Parizel PM, et al. Tuberculosis of the central nervous
system: Overview of neurological findings. Eur Radiol. 2003;12:1876-90.
2. Fitz CR. Inflammatory diseases of the brain in childhood. Am J Neuroradiol. 1992;13:551-67.
3. Whiteman M, Espinoza L, Post JD, et al. Central nervous system tuberculosis in HIV infected
patients: Clinical and radiographic findings. Am J Neuroradiol. 1995;16:1319-27.
Neurological System Imaging 385
CASE 30

A 36-year-old alcoholic with chronic liver disease was admitted with dehydration and hematemesis.
He became confused and comatose after receiving several liters of intravenous fluids to correct his
biochemical imbalance. Plain CT of the brain was normal.

Figures 1A and B B
386 A Teaching Atlas of Case Studies in Diagnostic Imaging

Figure 1C
Figures 1A to C  MR images
show a symmetric central
round pontine lesion,
hypointense on T1w
(Fig. A), hyperintense on
both T2w (Fig. B) and FLAIR.
The periphery of the pons
is spared, and there is
no mass effect

Diagnosis: Central pontine myelinolysis.

DISCUSSION
In central pontine myelinolysis (osmotic myelinolysis), rapid correction of hyponatremia causes
acute demyelination in the central pons and also in extrapontine sites such as the cerebellum, basal
ganglia, thalami, subcortical cerebral white matter, corona radiata, and hippocampi. Predisposing
factors include chronic alcoholism, malnutrition, liver failure, orthotopic liver transplant patients
on cyclosporine therapy and extensive burns. It is more common in males and rarely affects the
pediatric age group. Central pontine myelinolysis causes progressive and fatal quadriparesis,
mutism, dysarthria and bulbar palsy. The prognosis is usually poor, but recovery can occur,
demonstrable by serial MRI scans.

FURTHER READING
1. Chua GC, Stoh YY, Lim CC, et al. MRI findings in osmotic myelinolysis. Clin Radiol. 2002;57:800-6.
2. Lampl C, Yazdi K. Central pontine my­elinolysis. Eur Neurol. 2002;47:3-10.
3. Martin RJ. Central pontine myelinolysis: The osmotic demyelination syndrome. J Neurol Neurosurg
Psychiatry. 2004;75:22-8.
Neurological System Imaging 387
CASE 31

A 35-year-old woman presented with severe headaches, dizziness, flushing and palpitations,
and was found to be severely hypertensive. Elevated catecholamines were present in her urine.
Her chest radiograph was normal. Abdominal sonography showed a large hyperechoic solid mass
in the hypogastrium. Contrast-enhanced CT of the abdomen and pelvis was performed.

Figures 1A and B B
388 A Teaching Atlas of Case Studies in Diagnostic Imaging

Figure 1C
Figures 1A to C  Contrast-
enhanced sagittal, coronal
and axial images show a
large rounded solid mass
at the level of the aortic
bifurcation with peripheral
calcification and minimal
contrast enhancement.
The other solid abdominal
organs are normal.
A diagnosis of extra-adrenal
pheochromocytoma was
made

DISCUSSION
Pheochromocytomas are rare tumors that cause hypertension, tremors, etc. because of their
production of catecholamines. They arise from the paraganglion cells anywhere in the autonomic
nervous system. Ninety percent originate in the adrenal medulla. Extra-adrenal sites include the
organ of Zuckerkandl near the aortic bifurcation, as in this case, the para-aortic sympathetic chain,
and the urinary bladder. They are said to follow the rule of ‘tens’: 10% are bilateral, 10% are extra-
adrenal, 10% are malignant and 10% are familial. They are associated with such neuroendocrine
disorders as von Hippel–Lindau Syndrome, multiple endocrine neoplasia (MEN 2A and 2B),
neurofibromatosis, and tuberous sclerosis. Most are larger than 2 cm and may be solid or show
cystic degeneration. Calcification is rare but is typically eggshell or speckled. They usually enhance
intensely after IV contrast medium and show slow washout. MRI is the modality of choice to search
for extra-adrenal pheochromocytomas which show very high signal intensity on T2w sequences.
MIBG scintigraphy is also very useful in detecting ectopic lesions and for the identification of
metastases or locally recurrent disease.

FURTHER READING
1. Sahder, Sohaid A, Mousa JP, et al. CT and MR imaging of unusual locations of extra-adrenal
paragangliomas (pheochromocytomas). Eur Radiol. 2005;15:85-92.
2. Sohaib SA, Reznek RH. Adrenal imaging. BJU Int. 2000;86(Suppl 1):95-110.
Neurological System Imaging 389
CASE 32

A 53-year-old male complained of pain and stiffness in the neck for 4 months, dysphagia for
2 months and recent motor and sensory disturbances in his upper limbs. Plain radiographs of
the cervical spine followed by MRI were done.

Figure 1  Lateral plain


radiograph of the cervical
spine shows typical changes
of diffuse idiopathic skeletal
hyperostosis (DISH) with
massive ossification, along the
anterior cervical spine
(C4 to C7) with preservation
of the disc spaces. Ossification
of the posterior longitudinal
ligament (OPLL) is also shown

Figure 2  Sagittal T1w MRI


Image of the cervical spine
showing extensive irregular
thickening of the posterior
longitudinal ligament with
narrowing of the canal in OPLL
390 A Teaching Atlas of Case Studies in Diagnostic Imaging

Figure 3  Axial T2w MRI image


of the cervical spine showing
the severe OPLL

DISCUSSION
Diffuse idiopathic skeletal hyperostosis (DISH) is a common skeletal disorder characterized
by increased bone formation at multiple sites in the spine and the peripheral skeleton. It affects
the lower cervical, middle and lower thoracic and entire lumbar spine. Its cause is uncertain but
associated risk factors are said to include diabetes, obesity, hyperinsulinemia and other endocrine
abnormalities. The diagnostic criteria described by Resnick are: (1) Flowing ossification along
at least 4 contiguous vertebrae, (2) relative preservation of disc height, and (3) absence of joint
ankylosis or erosion.
Peripheral changes include ossification in the pubic symphysis, pelvic ligaments, tuberosities
and trochanters, whiskering of iliac crests and formation of large calcaneal, olecranon and patellar
spurs. Patients with DISH may develop pathological fractures with subluxations.
OPLL is characterized by ossification of the posterior longitudinal ligament and growth of mature
cortical bone along the midline of the posterior vertebral bodies. Involvement of the cervical spine
is most common. Eventual encroachment of the ossified ligament on the cervical cord leads to cord
compression, infarction and/or demyelination.
OPLL both symptomatic and asymptomatic occurs in association with DISH, as in this case, or
independently.

FURTHER READING
1. Diederichs G, Engelken F, Marshall LM, et al. Diffuse idiopathic skeletal hyperostosis. (DISH):
Relation to vertebral fractures and bone density. Osteoporosis Int. 2011;22(6):1789-97.
2. Resnick D, Niwayama G. Radiographic and pathologic features of spinal involvement in diffuse
idiopathic skeletal hyperostosis (DISH). Radiology. 1976;119:559-68.
3. Resnick D, Guerre J, Robinson CA, Vint VC. Association of diffuse idiopathic skeletal hyperostosis
(DISH) and calcification and ossification of the posterior longitudinal ligament. AJR Am J
Roentgenol. 1978;131:1049-53.
Neurological System Imaging 391
CASE 33

A 45-year-old man presented with recurrent low back pain. He was investigated with MRI of the
lumbosacral spine.

Figures 1A and B 
(A) Sagittal T1w image A shows
a hyperintense, oval mass
in the conus medullaris,
extending into the proximal
cauda equina. (B) It has a
high signal intensity on
T2w image B
392 A Teaching Atlas of Case Studies in Diagnostic Imaging

Figure 2  Low on fat-


saturated sequences.
Features are those of spinal
cord lipoma

Figure 3  Axial T2w image


shows septa in the mass
with the cord displaced
anteriorly
Neurological System Imaging 393

DISCUSSION
Spinal cord lipomas are rare and may occur in any part of the cord. They may produce neurological
manifestations due to compression. Symptoms usually develop slowly, with indolent but progressive
deterioration. MRI is invaluable in the diagnosis, since the masses behave like adipose tissue on all
sequences.

FURTHER READING
1. Kamat A, Findlay G. Intramedullary migration of spinal cord lipoma. J Neurol Neurosurg Psychiatry.
2003;74:1593-4.
2. Kim CH, Wang KC, Kim SK, et al. Spinal intramedullary lipoma: Report of three cases. Spinal cord.
2003;41:310-5.
3. Kujas M, Sichez JP, Lalam TF, Poirier J. Intradural spinal lipoma of the conus medullaris without
spinal dysraphism. Clin Neuropathol. 2000;19:30-3.
4. Patwardhan V, Patanakan T, Armao D, Murkherji SK. MR Imaging findings of intramedullary
lipomas. AJR Am J Roentgenol. 2000;174:1792-3.
394 A Teaching Atlas of Case Studies in Diagnostic Imaging
CASE 34

A 46-year-old man presented with severe lower back pain radiating to the right lower limb. He had
MRI examination of the lumbosacral spine.

Figures 1A and B  T1w right


parasagittal and T2w sagittal
images showing L3/4 disc
prolapse with cephalad
migration. A disc bulge is
B also noted at L5/S1
Neurological System Imaging 395

Figures 2A and B  (A) T2w


axial images at the level of
L3/4 neural foramina, and
(B) over the body of L3 show
a right paracentral disc
compressing the thecal sac
and right nerve root and
extending upwards in the
epidural space behind L3 B
396 A Teaching Atlas of Case Studies in Diagnostic Imaging

DISCUSSION
Herniated disc fragments are known to migrate within the spinal canal in cephalad, caudal or lateral
directions. Unrecognized free fragments are important as they can be a cause of postoperative failed
back syndrome.

FURTHER READING
1. Chen CY, Chuang YL, Yao MS, et al. Posterior epidural migration of a sequestrated lumbar disc
fragment: MR Imaging findings. Am J Neurol. 2006;27:1592-4.
2. Kim IS, Lee SW, Son BC, Sung JH. Posterior epidural migration of thoracic disc fragment. J Korean
Neurosurg Soc. 2008;43(5):239-41.
3. Mobbs RJ, Steel TR. Migration of lumbar disc herniation: An unusual case. J Clin Neurosci.
2007;14(6):581-4.
Neurological System Imaging 397
CASE 35

A 38-year-old male presented with radicular pain, weakness of both legs and sphincter dysfunction
for three months. Plain X-rays of the spine were normal. MRI of the lumbosacral spine was
performed.

Figures 1A and B  T1w


and T2w sagittal images
reveal a well-defined, ovoid
expansile intramedullary
lesion in the conus
medullaris, isointense on
T1 and hyperintense on
T2 and with a hypointense
hemosiderin rim on T2
(Cap sign). A tumor
associated homogeneous
tubular cyst (of low signal
on T1 and high signal on T2)
is shown above the tumor
which also enhances
inhomogeneously after
intravenous gadolinium B
398 A Teaching Atlas of Case Studies in Diagnostic Imaging

The findings are those of ependymoma of the conus—the myxopapillary type.

DISCUSSION
Spinal tumors occur in the extradural extramedullary and in the intramedullary compartments.
Intramedullary tumors are mainly ependymomas and astrocytomas. The myxopapillary type of
ependymomas predominates in the conus and filum terminale. Other rare intramedullary tumors
are hemangioblastomas and metastases. Intramedullary tumors are associated with three types
of cysts: Intratumoral, peritumoral or syringomyelia. Rarely ependymomas of the filum terminale
may present as a destructive lesion of the sacrum or a presacral mass which is prone to metastasis,
especially to the lungs.

FURTHER READING
1. Bazan C. Imaging of lumbosacral spine neoplasms, Neuroimaging Clin North Am. 1993;3:591-608.
2. Miyazawa N, Hida K, Iwasaki Miyazawa N, Hida K, Iwasaki, et al. MRI at 1.5T of intramedullary
ependymoma and classification of pattern of contrast enhancement. Neuroradiology. 2000;42:
828-32.
Neurological System Imaging 399
CASE 36

A 46-year-old male with a painful, firm nodule in the right popliteal fossa of one year duration.
Ultrasonography demonstrated a well-delineated solid mass of low echogenicity, inseparable from
the tibial nerve. A knee MRI was done without and with gadolinium.

Figure 1  Sagittal T1w image of


the knee showing
an oval mass with low signal
intensity, inseparable from
the tibial nerve

Figure 2  T2w image shows


the mass of intermediate signal
intensity contained within and
splaying the tibial nerve
400 A Teaching Atlas of Case Studies in Diagnostic Imaging

Figure 3  Postgadolinium
injection, the mass
shows intense contrast
enhancement

Figure 4  Proton
density image showing
intermediate signal intensity
Neurological System Imaging 401

Figure 5  Axial view after


contrast injection, showing
intense enhancement

Diagnosis: Tibial nerve neuroma-in-continuity.

DISCUSSION
A neuroma-in-continuity is one occurring in a nerve with an intact perineural sheath. A tibial nerve
neuroma is rare in the popliteal fossa, and is readily recognized by its location between the lateral
and medial heads of the gastrocnemius muscle. It is also a rare cause of posterior knee pain, as in
this patient.

FURTHER READING
1. Deluca PF, Bartolozzi AR. Tibial neuroma presenting as a Baker cyst: A Case Report. J Bone Joint
Surg. 1999;81(6):856.
2. Donovan A, Rosenberg ZS, Cavalcanti CF. MR imaging of entrapment neuropathies of the lower
extremity. Part 2. The Knee, Leg, Ankle and Foot. Radiographics. 2010;30:1001-19.
402 A Teaching Atlas of Case Studies in Diagnostic Imaging
CASE 37

A 32-year-old man presented with a swelling in the left palm. He was referred for MRI to characterize
the nature of the mass and its relationship to surrounding structures.

Figure 1  Coronal inversion


recovery MR image shows a
high signal oval mass medial to
the thenar eminence

Figure 2  T1w coronal


image shows that the mass
is isointense with thenar
muscles. Note the tapering
from its ends, indicating the
presence of entering and
exiting nerves
Neurological System Imaging 403

Figures 3A and B  (A) T2w


axial images with fat
suppression and without.
(B) The mass shows a central
reduction in signal intensity,
giving a positive “target sign”.
Features are those of a
neurofibroma, probably
from a digital branch of the
median nerve B
404 A Teaching Atlas of Case Studies in Diagnostic Imaging

DISCUSSION
The differential diagnosis of masses in the palm includes giant cell tumor, lipoma, fibroma and
neuroma, and neurofibroma. Neurofibromas are characterized by the identification of entering
and exiting nerves. On T2w images, neurofibromas tend to show a central reduced signal intensity
surrounded by high intensity (see Fig. 3B), referred to as the “target sign”. This appearance is thought
to be due to the composition of the mass, with low-intensity fibrocollagenous tissue surrounded
by high intensity myxomatous tissue.

FURTHER READING
1. Andreisek G, Crook DW, Burg D, et al. Peripheral neuropathies of the median, radial, and ulnar
nerves: MR imaging features. Radiographics. 2006;26:1267-87.
2. Banks KP. The Target sign: Extremity. Radio­logy. 2005;234:899-900.
3. Murphey MD, Smith WS, Smith SE, et al. From the archives of the AFIP. Imaging of musculoskeletal
neurogenic Tumors: Radiologic-Pathologic correlation. Radio­graphics. 1999;19:1253-80.
4. Reynolds DL, Jacobson JA, Inampudi P, et al. Sonographic characteristic of peripheral nerve sheath
tumors. AJR Am J Roentgenol. 2004;182(3):741-4.
6 Miscellaneous Images

CASE 1

A 35-year-old woman with painful right breast and fever.

Figures 1A and B  Breast ultrasound


showed fine, mobile echoes from
a breast abscess. Note the abscess
pointing towards the skin (arrow) B
406 A Teaching Atlas of Case Studies in Diagnostic Imaging
CASE 2

A 40-year-old woman with previously diagnosed bilateral breast cysts came with pain in both
breasts.

B Figures 1A and B
Miscellaneous Images 407

Figure 1C
Figures 1A to C 
(A and B) MRI left and right
breasts showing multiple
cysts of varying sizes.
(C) Note ring enhancement
of some of the cysts
after contrast injection
suggesting secondary
inflammation
408 A Teaching Atlas of Case Studies in Diagnostic Imaging
CASE 3

A 47-year-old woman presented with right breast mass.

B Figures 1A and B
Miscellaneous Images 409

Figure 1C
Figures 1A to C  Ultrasound
shows a hypoechoic mass
with a branching pattern (A),
and irregular margins in
parts (C). Excision biopsy
confirmed carcinoma
410 A Teaching Atlas of Case Studies in Diagnostic Imaging
CASE 4

A 50-year-old woman with a left breast mass diagnosed on ultrasound as cystic.

Figure 1  Mammogram
shows a retroareolar mass
with smooth outlines,
consistent with a simple cyst
Miscellaneous Images 411
CASE 5

A 48-year-old woman presented with a left breast mass.

Figure 1  Mammogram
shows a mass with stellate
(spiculated) margins,
consistent with malignancy.
A smaller adjacent nodule
is noted, probably an
intramammary lymph node
412 A Teaching Atlas of Case Studies in Diagnostic Imaging
CASE 6

A 55-year-old woman presented with a left breast mass.

Figures 1A and B  Bilateral


mammograms show
a normal right breast
parenchyma. Left breast
[magnified in (B)] shows a
large mass with irregular
outlines, spiculated margins,
retracted nipple, and focal
B skin thickening
Miscellaneous Images 413
CASE 7

A 56-year-old woman with a left breast mass.

Figure 1  Bilateral
mammograms show normal
right breast parenchyma.
There is a left breast mass
with marked irregularity of
its outlines and extensive
spiculation. She also has
microcalcifications (poorly
reproduced). Features are
those of malignancy
414 A Teaching Atlas of Case Studies in Diagnostic Imaging
CASE 8

A 34-year-old woman had a left breast mass discovered during the second trimester of her
pregnancy. Shortly after, she presented with severe back pain and paraparesis.

Figure 1  MRI both breasts.


There is a deep-seated mass
in the lateral aspect of
the left breast

Figure 2A
Miscellaneous Images 415

Figure 2B
Figures 2A and B  T2w and
T1w MR images of the left
breast show the large mass

Figure 3  Sagittal MR image


of the thoracolumbar spine.
Note collapse of D10
vertebral body with
extension posteriorly,
obliterating the anterior
thecal sac and indenting
the spinal cord
416 A Teaching Atlas of Case Studies in Diagnostic Imaging

Figure 4  Sagittal MR
image with contrast shows
intense enhancement of
the vertebral body and
focal enhancement of
the cord. Features are those
of carcinoma of the breast
with metastasis to the spine

FURTHER READING
1. American College of Radiology. ACR Breast Imaging Reporting and Data System (BIRADS) Website:
www.acr.org.
2. Hong AS, Rosen EL, Soo MS, Baker JA. BIRADS for sonography: Positive and negative predictive
values of sonographic features. AJR Am J Roentgenol. 2005;184(4):1260-5.
3. Raza S, Goldkamp AL, Chikarmane SA, Birdwell RL. US of breast masses categorized as BI-
RADS 3, 4, and 5: Pictorial review of factors influencing clinical management. Radiographics.
2010;30(5):1199-213.
4. Uematsu T, Kasami M, Yuen S, et al. Comparison of 3- and 1.5-T dynamic breast MRI for visual­
ization of spiculated masses previously identified using mammography. AJR Am J Roentgenol.
2012;198(6):W611-7.
Miscellaneous Images 417
CASE 9

A 48-year-old male diabetic presented with claudication, impotence and absent femoral pulses.
CT angiogram of the abdominal aorta was performed.

Figure 1  The image shows


severe atheromatous
changes in the aorta and
virtually complete aortic
occlusion with thrombus in
the aortic lumen

Diagnosis: Leriche Syndrome.

DISCUSSION
See under Case 10 (Page 418).
418 A Teaching Atlas of Case Studies in Diagnostic Imaging
CASE 10

Figure 1  CT angiogram of
another patient, 50-year-old
diabetic man. There is
atheromatous occlusion
of the distal abdominal
aorta and the common iliac
arteries. The lower limb
arteries are reconstituted
by collaterals

DISCUSSION
Aortoiliac occlusive disease is most commonly caused by atherosclerosis. Patients with this disease
usually present with claudication. Bilateral buttock claudication, impotence and absent femoral
pulses constitute Leriche syndrome, although this term is often used to signify aortic occlusion
by imaging as in this case, rather than based on clinical symptoms. Aortoiliac occlusive disease
can also be caused by inflammatory disease, in particular Takayasu arteritis. Other causes include
neurofibromatosis and fibromuscular dysplasia.

FURTHER READING
1. Brewster DC. Current controversies in the management of aortoiliac occlusive disease. J Vasc Surg.
1997;25(2):365-79.
2. Chang IS, Park KB, Do YS, et al. Heavily calcified occlusive lesions of the iliac atrtery: Long-term
patency and CT findings after stent placement. J Vasc Interv Radiol. 2011;22(8):1131-7.
3. Weitz JI, Byrne, J, Clagett GP. Diagnosis and treatment of chronic arterial insufficiency of the lower
extremeties: a critical review. Circulation. 1999;94(11):3026-49.
Miscellaneous Images 419
CASE 11

A 7-year-old girl presented with differential tightness of her swim suit over the left thigh, with
associated left leg pain after each swimming lesson, which was relieved by rest. A Doppler
ultrasound study showed reduced flow in the left common femoral vein, compared with the right.
Vascular MRI studies were done to exclude venous obstruction.

Figure 1  Axial MR images at


the middle of both thighs. The left
thigh is larger and more vascular

Figure 2  MR arteriogram and


venogram from the distal aorta
and proximal inferior vena cava
to the thighs. A low-signal filling
defect is seen in the left external
iliac vein. Note multiple collateral
veins draining the left thigh
420 A Teaching Atlas of Case Studies in Diagnostic Imaging

Figures 3A and B  (A) Left


thigh venogram shows the
filling defect in the external
iliac vein and the numerous
collaterals; (B) A coned-
down image for better
demonstration of the filling
defect (arrow). Features
are those of a congenital
B venous web
Miscellaneous Images 421

DISCUSSION
Congenital venous webs are rare. One case was reported in a girl of similar age, involving the
subclavian vein and presenting as exercise-induced upper limb swelling.

FURTHER READING
1. Fisher JB, Grason MA. Congenital venous web causing subclavian vein obstruction:
A case report. J Vasc Surg. 1989;10(4):460-2.
2. Thakur S, Comerota AJ. Bilateral non­thrombotic subclavian vein obstruction causing upper extremity
venous claudication. J Vasc Surg. 2010;52(1):208-11.
422 A Teaching Atlas of Case Studies in Diagnostic Imaging
CASE 12

A 78-year-old man presented with urinary frequency, urgency and nocturia. He denied any history
of hematuria. A CT urogram with contrast was requested.

Figure 1  Contrast-enhanced axial


CT image of the pelvis shows a large,
right-sided irregular filling defect
occupying almost the entire bladder
cavity, with right perivesical fat
stranding

Figure 2  Postcontrast CT
abdominal radiograph shows
irregular masses arising from
the right bladder wall, causing
nonexcretion from the right kidney
and gross left hydronephrosis.
Features are those of carcinoma of
the bladder which was confirmed
at cystoscopy. Histology showed
well-differentiated squamous cell
carcinoma

DISCUSSION
See Chapter 3, Case 24 (Page 208).
Miscellaneous Images 423
CASE 13

A 26-year-old woman had abdominal ultrasound done for vague abdominal pain. A right
ureterocele was discovered incidentally. Both kidneys and the other abdominal organs were
normal.

Figure 1  Transverse and


sagittal ultrasound of
the urinary bladder
shows a cyst within a cyst
appearance typical for
a ureterocele

DISCUSSION
A ureterocele is a cystic dilatation of the terminal end of the ureter. Eighty percent occur in
association with ureteric duplication, but some occur without duplication. Ultrasonography is
the most sensitive modality for the diagnosis of ureterocele, but errors can occur if the bladder
is empty or if it is too full. On intravenous urography, the contrast-filled ureterocele is separated
from the bladder lumen by a lucent halo representing the ureterocele wall—an appearance termed
the “spring onion” or “cobra head” deformity.

FURTHER READING
1. Ayers E. Incidental sonographic finding of bilateral ureteroceles. J Diag Med Sonography.
2006;22:123-6.
2. Merlini E, Lelli CP. Obstructive ureterocelean ongoing challenge. World J Urol. 2004; 22(2):104-14.
3. Zerin JM, Baker DR, Casale JA. Single-system ureterocele in infants and children: Imaging features.
Pediatr Radiol. 2000;30(3):139-46.
424 A Teaching Atlas of Case Studies in Diagnostic Imaging
CASE 14

A 33-year-old man presented with recurrent left flank pain. An abdominal ultrasound showed a left
upper pole calyceal diverticulum containing fine internal echoes. CT abdomen was advised.

Figure 1  Plain axial CT in


supine position. There is a
density in a left renal calyceal
diverticulum, with a fluid
level and the density layering
posteriorly

Figure 2  Sagittal reformatted


CT image shows the opacity
layering posteriorly. Note its
upper pole location
Miscellaneous Images 425

Figure 3  Axial CT with


patient in prone position.
The opacity has shifted
anteriorly. Features are
those of a milk of calcium
calyceal diverticulum

DISCUSSION
Milk of calcium cyst is a collection of small calcific granules, usually composed of calcium carbonate.
The granules layer out in the most dependent position, changing with posture. It most frequently
occurs in an upper pole calyceal diverticulum. The predisposing factors for the development of
a milk of calcium cyst include urinary stasis, urinary tract infection, and prolonged immobility,
as in debilitated patients and those with spinal cord injury.
At ultrasonography, the milk of calcium shows as an echogenic material which layers in the
most dependent part, shifting with posture. Acoustic shadowing is often absent except in large
collections. As shown in this patient, CT demonstrates features which are diagnostic.

FURTHER READING
1. Bude RO, Korobkin MT. Milk of calcium renal cyst: CT findings. Urology. 1992;40(2):149-51.
2. Rathaus V, Konen O, Werner M, et al. Pyelocalyceal diverticulum: The imaging spectrum with
emphasis on the ultrasound features. Br J Radiol. 2001;74:595-601.
3. Uluson S, Koc Z. Milk of calcium collection in the differential diagnosis of giant renal calculus. Br J
Radiol. 2008;81(962):e35-6.
4. Vaidyanathan S, Hughes PL, Soni BM. Bilateral renal milk of calcium masquerading as nephrolithi­
asis in patients wih spinal cord injury. Adv Ther. 2007;24(3):533-44.
426 A Teaching Atlas of Case Studies in Diagnostic Imaging
CASE 15

A 24-year-old woman had abdominal ultrasound for vague abdominal pain. It was observed that
the right kidney was enlarged, but the left kidney could not be located. CT was done in an attempt
to localize the left kidney.

Figure 1 Coronal
reformatted contrast-
enhanced CT abdomen
shows a large solitary right
kidney. The image mimics
a crossed fused ectopia but
differs in having only
a single ureter

DISCUSSION
Unilateral renal agenesis occurs in 1 out of 1,000–2,000 live births. It may occur without any other
anomaly, but has been associated with the VACTERL anomalies. (V: Vertebral; A: Anorectal;
C: Cardiovascular; T: Trachea; E: Esophageal; R: Renal and Radial; L: Limb), as well as genitourinary
anomalies, including cryptorchidism. Many cases are diagnosed in utero by ultrasonography.
Reports of long-term follow-up of cases show an increased risk of proteinuria, hypertension, and
renal insufficiency.

FURTHER READING
1. Argueso LR, Ritchey ML, Boyle ET, et al. Prognosis of patients with unilateral renal agenesis. Pediatr
Nephrol. 1992;6(5):412-6.
2. Onwuchekwa RC, Sapira MK, Onwuchekwa AC. Unilateral renal agenesis coexisting with bilateral
cryptorchidism in an adult Nigerian: Case report. Niger Med J. 2009:50:71-3.
3. Vu KH, Van Dyck M, Daniels H, Proesmans W. Renal outcome of children with one functioning
kidney from birth. A study of 99 patients and a review of the literature. Eur J Pediatr. 2008;167(8):
885-90.
Miscellaneous Images 427
CASE 16

A 4-year-old emaciated boy with growth retardation was sent for abdominal ultrasound with
clinical history of failure to thrive and repeated episodes of urinary tract infection.

Figure 1  Longitudinal
ultrasound images of both
kidneys show normal size
with smooth outline.
There is increased
echogenicity of medullary
pyramids and extensive
calcification with posterior
acoustic shadowing in the
region of the medulla

Figure 2  Erect frontal


radiograph of abdomen
shows large coarse granular
calcifications in the renal
pyramids bilaterally.
Features are those of
medullary nephrocalcinosis.
428 A Teaching Atlas of Case Studies in Diagnostic Imaging

DISCUSSION
Medullary nephrocalcinosis refers to deposition of calcium salts in the medulla of the kidney.
Due to the concentrating effects of the loops of Henle, and the biochemical millieu of the medulla,
compared to the cortex, it is 20 times more common than cortical nephrocalcinosis.
Medullary nephrocalcinosis is characteristic of medullary sponge kidney (MSK), but it can occur
in several systemic metabolic disorders such as hypercalcemia, hypercalciuria and hyperoxaluria.
It is also seen in hyperparathyroidism, renal tubular acidosis, sarcoidosis and metastatic disease.
MSK is a developmental anomaly character­ized by cystic dilatation of the collecting tubules in
one or more renal medullary pyramids. It is of uncertain etiology and almost always diagnosed
radiologically. It may be asymptomatic or may be characterized by repeated renal colics, calculous
disease, hematuria or repeated urinary tract infections.
As in this patient, ultrasonography demon­strates echogenic medullary pyramids, but can also
demonstrate complications such as calculi, hydronephrosis and pyelonephritis.
Plain film and CT demonstrate characteristic clusters of pyramidal medullary calcification.
Delayed postcontrast CT may demonstrate a “paint brush” appearance of the renal med­
ullary regions, and pyelography may give a characteristic “bouquet of flowers” appearance.
(Courtesy: Dr Sujatha Rajkumar)

FURTHER READING
1. Cheidde L, Ajzen SA, Tamer Langen CH, et al. A critical appraisal of the radiological evaluation of
nephrocalcinosis. Nephron Clin Pract. 2007;106(3):119-24.
2. Ginalski JM, Schnyder P, Portmann L, Jaeger P. Medullary sponge kidney on axial computed
tomography: Comparison with excretory urography. Eur J Radiol. 1991;12(2):104-7.
3. Schell-Feith EA, Holscher HC, Zonderland HM, et al. Ultrasonographic features of nephrocalcinosis
in preterm neonates. Br J Radiol. 2000;73(875):1185-91.
Miscellaneous Images 429
CASE 17

A 39-year-old woman presented with thyroid swelling.

Figures 1A and B 
Longitudinal and transverse
thyroid ultrasound images
show numerous cysts with
tiny echogenic foci, some of
the foci show typical
“comet-tail” artefacts
characteristic of colloid
cysts. Those echogenic
foci are considered to be
probably microcrystals B
430 A Teaching Atlas of Case Studies in Diagnostic Imaging
CASE 18

An 11-year-old girl presented with thyroid swelling.

Figure 1  Transverse
ultrasound of the
thyroid shows numerous
hypoechoic lesions
involving both lateral lobes
and the isthmus. Diagnostic
possibilities would include
lymphoma and Hashimoto’s
thyroiditis

DISCUSSION
Although thyroid imaging techniques include sonography, scintigraphy, radiography, CT and MRI,
ultrasound is still the imaging modality of choice. Because of the superficial location of the gland,
the thyroid lends itself to high definition images using high frequency transducers (7.5–15 MHz).
Sonography assists in demonstrating the nature of a nodule or nodules, whether cystic or solid,
the outlines, presence and type of calcification, and degree of vascularity, in the attempt to define
malignancy, benignity, or inflammation.
The ultrasound examination of the thyroid is incomplete without assessing the supraclavicular,
submandibular and jugular chain of lymph nodes.
Ultrasonography is also invaluable in guiding fine needle aspirations.

FURTHER READING
1. Bonavita JA, Mayo J, Babb J, et al. Pattern recognition of Benign Nodules at Ultrasound of the
Thyroid: Which nodules can be left alone? AJR Am J Roentgenol. 2009;193(1):207-13.
2. Kim DW, Park JS, In HS, et al. Ultrasound-based diagnostic classification for solid and partially
cystic thyroid nodules. AJNR Am J Neuroradiol. 2012;33(6):1144-9.
3. Wu S, Liu G. Recognition of specific morphologic patterns for identifying benign thyroid nodules.
AJR Am J Roentgenol. 2010;194(4):w353.
Miscellaneous Images 431
CASE 19

A 3-year-old girl was referred for ultrasound due to bilateral angle of mandible swelling and
discomfort.

Figure 1  Longitudinal
ultrasound of the left
parotid show multiple
hypoechoic lesions of
varying sizes diffusely
distributed in the gland.
The large ones are
intraglandular lymph nodes.
Note few hyperechoic
foci, probably from specks
of calcification. The right
parotid also shows
a similar appearance

Diagnostic possibilities include: HIV, lymphoma and parotitis

DISCUSSION
Most cases of HIV infection in young children represent prenatal infection. Parotid involvement
appears on ultrasonography as widespread hypoechoic areas suggesting lym­phoid infiltration.

FURTHER READING
1. Howlett DC. High resolution ultrasound assessment of the parotid gland. Br J Radiol. 2003;76:271-7.
2. Lowe LH, Stokes LS, Johnson JE, et al. Swelling of the angle of the mandible: Imaging of the pediatric
parotid gland and periparotid region. Radiographics. 2001;21(5):1211-27.
3. Sodhi KS, Bartlett M, Prabhu NK. Role of high resolution ultrasound in Parotid lesions in Children.
Int J Pediatr Otorhinolaryngol. 2011;75(11):1353-8.
432 A Teaching Atlas of Case Studies in Diagnostic Imaging
CASE 20

A 31-year-old woman was referred for obstetric ultrasound because of slight vaginal bleeding at
34 weeks of pregnancy.

Figure 1  Ultrasound
images show an anterior
placenta in the upper
uterine segment. There was
no placental separation.
The fetus is presenting by
the feet

DISCUSSION
Breech delivery occurs in about 4% of vaginal deliveries. It has distinct disadvantages over
cephalic-presenting deliveries, as the fetal buttocks and feet do not provide effective dilatation
of the cervix during labor, and there is also a higher risk of umbilical cord prolapse. This risk is
highest in foot presentation (17%). Overall, there is increased morbidity to the fetus and the mother
in breech deliveries.

FURTHER READING
1. Stitely ML, Gherman RB. Labor with abnormal presentation and position. Obstet Gynecol Clin
North Am. 2005;32(2):162-79.
Miscellaneous Images 433
CASE 21

A 20-year-old man presented with chronic headache. An MRI brain was requested, which showed
no intracranial abnormality. An incidental lesion was seen in the posterior nasopharynx.

Figure 1  Sagittal T1w MR


image shows a nodule in
the posterior wall of the
nasopharynx. Its high
signal intensity suggests
hemorrhage or a high
proteinaceous content

Figure 2A
434 A Teaching Atlas of Case Studies in Diagnostic Imaging

Figure 2B
Figures 2A and B  Axial
and coronal T2w image
shows an oval, hyperintense
midline nodule, with
smooth outlines in the roof
of the nasophrynx. Features
are those of a Tornwaldt’s
(or Thornwaldt’s) cyst

DISCUSSION
Thornwaldt’s cyst represents a persistent communication between the roof of the nasopharynx
and the notochord, and lies in the midline. Its peak age of onset is the 2nd and 3rd decades.
The content of Thornwaldt’s cyst may be highly proteinaceous or hemorrhagic, and therefore show
high signal intensity on T1w MR sequences (see Fig. 1). The cyst is most commonly asymptomatic
and discovered incidentally during imaging for other reasons. When symptomatic, patients may
present with headache, halitosis, postnasal drip, earache and otitis media. An adenoid retention
cyst is a differential.

FURTHER READING
1. Ben Salem D, Dulliard C, Assous D, et al. Imaging of nasopharyngeal cysts and bursae. Eur Radiol.
2006;16(10):2249-58.
2. Lohman BD, Sarikaya B, McKinney AM, Hadi M. Not the typical Tornwaldt’s cyst this time?
A nasopharyngeal cyst associated with canalis baslaris medianus. Br J Radiol. 2011;84(1005):
e169-e71.
3. Magliulo G, Fusconi M, D’Amico R, de Vincentiis M. Tornwaldt’s cyst and magnetic resonance
imaging. Ann Otol Rhinol Laryngol. 2001;110(9):895-6.
4. Palacios E, Valvassori G. Tornwaldt’s cyst. Ear Nose Throat J. 2000;79(5):348.
Miscellaneous Images 435
CASE 22

A 66-year-old woman presented with a painful swelling on the dorsum of the right foot.

Figure 1  Sagittal inversion


recovery MR image shows
a high-signal intensity
mass on the dorsum of
the foot opposite to
the metatarsophalangeal
joint

Figure 2  Sagittal MR
image after gadolinium
injection shows peripheral
enhancement. Features are
those of a ganglion
436 A Teaching Atlas of Case Studies in Diagnostic Imaging

DISCUSSION
A ganglion is thought to arise from repetitive trauma of the tendon sheath, leading to mucoid cystic
degeneration. On ultrasonography, ganglia show acoustic enhancement. On MRI, they show low
or intermediate intensity on T1w images and high on T2w and inversion recovery images, and
show peripheral ring enhancement after contrast injection. In the foot, they are usually located on
the dorsum at the level of the metatarsophalangeal joints.

FURTHER READING
1. Beaman FD, Peterson JJ. MR imaging of cysts, ganglia, and bursae about the knee. Radiol Clin North
Am. 2007;45(6):969-82.
2. Pham H, Fessel DP, Femimo JE, et al. Sonography and MR imaging of selected Benign masses in
the Ankle and Foot. AJR Am J Roentgenol. 2003;180(1):99-107.
Miscellaneous Images 437
CASE 23

A 19-year-old worker in the packaging section of a factory had his right hand accidentally caught in
the stitching machine.

Figures 1A and B  AP and oblique


views of the right hand show
the numerous metallic foreign
bodies from the packaging clips B

FURTHER READING
1. Donaldson JS. Radiographic imaging of foreign bodies in the hand. Hand Clin. 1991;7(1):125-34.
438 A Teaching Atlas of Case Studies in Diagnostic Imaging
CASE 24

A 26-year-old man presented with a painless right testicular swelling.

Figure 1  Ultrasound
right testis shows a
large hypoechoic area
with irregular margins
and scattered foci of
calcification. The left testis
was normal

Figure 2  The testicular


lesion shows high
vascularity on Doppler and
an associated hydrocele.
Possibilities include
teratoma or seminoma.
CT of the chest and
abdomen were normal.
Beta-hCG and alpha-
fetoprotein assays were also
normal. Patient was referred
to a cancer center where
a right orchidectomy was
performed
Miscellaneous Images 439

DISCUSSION
Testicular germ cell tumors account for 1% of cancers in men, but are the most common
malignancy in the 15–35 years age group. Seminomas comprise 56% of germ cell tumors, while
the nonseminomatous tumors (embryonal cell carcinoma, choriocarcinoma, teratoma and yolk sac
tumors) form the rest.
Testicular microlithiasis is generally a benign condition, but has also been described in
association with testicular tumors.

FURTHER READING
1. Coffey J, Huddart RA, Elliott F, et al. Testicular microlithiasis as a familial risk factor for testicular
germ cell tumor. Br J Cancer. 2007;97:1701-6.
2. Krohmer SJ, McNulty NJ, Schned AR. Testicular seminoma with lymph node metastases.
Radiographics. 2009;2117-83.
3. Sohaib SA, Koh DM, Husband JE. The role of imaging in the diagnosis, staging, and management
of testicular cancer. AJR Am J Roentgenol. 2008;191(2):387-95.
4. Tsili A, Argyropoulou MI, Giannakis D, et al. MRI in the characterization and local staging of
testicular neoplasms. AJR Am J Roentgenol. 2010;194(3):682-9.
440 A Teaching Atlas of Case Studies in Diagnostic Imaging
CASE 25

A 43-year-old female had pelvic MRI done for evaluation of possible uterine fibroids. A fundal
uterine fibroid was present, but a right introitus cystic swelling was incidentally discovered.

Figure 1  Axial T1w MR image


of the pelvis shows an oval
mass with intermediate signal
intensity n the right outer
vaginal wall, displacing
the vaginal canal medially

Figure 2  Axial image after


contrast injection shows no
enhancement of the mass
Miscellaneous Images 441

Figures 3A and B  Axial and


coronal T2w images
show the mass to be
hyperintense with
smooth outlines B
442 A Teaching Atlas of Case Studies in Diagnostic Imaging

Figure 4  Sagittal T2w


image again showing the
mass. The features are
those of a Bartholin cyst.
Note a fundal uterine
fibroid, a Nabothian cyst
and a right ovarian follicular
cyst

DISCUSSION
The Bartholin glands are located, one on each side of the introitus. Their physiological function
is to secrete the fluid that provides lubrication to the vagina in the sexually aroused woman.
A Bartholin’s cyst arises when the duct draining the gland is occluded. The cyst is painless and may
be discovered incidentally, as in this patient. However, if it becomes infected, an abscess can arise,
with considerable pain.

FURTHER READING
1. Grant LA, Sala E, Griffin N. Congenital and acquired conditions of the vulva and vagina on magnetic
resonance imaging: A pictorial review. Semin Ultrasound. CT MR. 2010;31(5):347-62.
2. Hahn WY, Israel GM, Lee VR. MRI of female urethral and peri-urethral disorders. AJR Am J
Roentgenol. 2004;182(3):677-82.
3. Omole F, Simmons BJ, Hacker Y. Management of Bartholin’s duct cyst and gland abscess. Am Fam
Physic. 2003;68(1):135-40.
Index

Page numbers followed by f refer to figure


A B Carcinoma
bladder 207
Abdominal swelling, Baker’s cyst 88, 154 bronchogenic 45f, 46, 52, 52f
progressive 278 Bamboo spine 77f hepatocellular 281
Abscess Bartholin’s cyst 442, 442f Cardioembolic disease 340
prostatic 183 Bartholin’s glands 442 Carpus, scapholunate dislocation
pyogenic 45 Basal cell nevus syndrome 365 of 106f
retropharyngeal 296 Basal ganglia calcifications Cellulitis, retropharyngeal 297
Achilles tendon, complete 320f Central nervous system,
rupture of 104f Basal pneumatoceles 19f tuberculosis of 384
Acoustic neuroma 355 Basilar artery 377f, 380 Cerebellopontine angle
Actinomycetoma 145 Bladder arachnoid cyst 332
Acute pancreatitis, recurrence carcinoma of 422f mass 348f
of 275f extrophy 83 meningioma 349
Adenocarcinoma 209 Blooming artefacts 123f Cerebral artery 378f, 379f
Air-bronchogram 20 Bochdalek hernia 33 Charcot joint 144, 144f
Alveolar proteinosis, Bone Chest syndrome 10f
pulmonary 65f cyst, aneurysmal 95, 95f Cholecystitis 267f
Amenorrhea 195 scalloping of 152f Choledochal cyst 250, 283
Amyloidosis 144 tumor 148 Chondrosarcoma 150, 365
Aneurysms, cerebral 380 Bowel infarction 290f Chordoma 365
Angiomyolipoma, bleeding 196f Bowel ischemia 290 Choriocarcinoma 439
Anorectal malformation 241 Brain abscess 375 Cirrhotic liver 281f
Aorta Breast abscess 405f Claw sign 214f
abdominal 156f Breast cancer 148 Clonorchis sinensis
coarctation of 29, 29f Breast cysts, bilateral 406 infestation 283
distal abdominal 418f Breast mass 408, 411-413 Cobra head deformity 423
Aortic lumen 417f Breast parenchyma 412f, 413f Collar sign 32
Aortoiliac occlusive disease 418 Breech delivery 432 Colles fracture 106f
Appendicitis 300 Brodie’s abscess 113f, 114 Colloid cysts 429f
Appendix 265 Brown tumor 148 Comet-tail artefacts 429f
Arterial hypertension, Buchem disease 143 Computed tomographic
pulmonary 25 Bulbar palsy 386 angiogram, cerebral 378f
Arteries Constipation 305
choroid 329 C
Conus medullaris 391f, 397f
pulmonary 26f Calcifications, multiple foci Cord, cervical 347f
Arteriovenous malformations of 365f Corona radiate 372f
67f, 85f, 322, 342f, 343 Calcium calyceal diverticulum, Coronary by-pass surgery 30
Arthropathy, degenerative 144 milk of 425f Corpus callosum 372f
Aspergillosis 45 Calcium cyst, milk of 425 Cortex, internal scalloping
Atresia, bronchial 37 Caliber jejunum 312f of 130f
Atrophy 371f Camurati-Engelmann Cowden syndrome 365
cerebral 326f disease 143 Crohn’s disease 291
444 A Teaching Atlas of Case Studies in Diagnostic Imaging

Cryptorchidism 83, 213 Emphysema, pulmonary 8f Gardner’s syndrome 82


Cushing’s disease 40 Endotracheal tube 7f Gastric herniation 32
Cyst 334f Epidermoid inclusion cyst 79f Gastroesophageal junction 256f
arachnoid 337f Epigastric pain, recurrent 278 Gastroesophageal reflux 256f
bronchogenic 36f Epiphysiolysis 76 Gastrointestinal system
hepatic 166f Esophageal compression 35f imaging 240
hydatid 87f Esophageal hiatus 256f Genu valgum and varum 118
intrapulmonary 37 Eumycetoma 145 Gerota’s fascia 219
Cystic adenomatoid Eustachian tube 382 Giant cell tumor 152f
malformation 70 Ewing’s sarcoma 149, 150, Giant intracerebral lipoma 357
Cystic bronchiectasis, 362, 365 Gorlin-Goltz syndrome 74
congenital 43 Extensive intracerebral air Ground-glass bats-wing
Cystic duct 250f, 277f bubbles 366f opacities, bilateral 15f
Cystic fibrosis 43 Extensive osteolytic
Cystoscopy 422f destruction 148f H
Cytomegalovirus 322 Extensive soft-tissue emphysema
110f Harrison’s sulcus 118
D Extrahepatic bile ducts 284f Hashimoto’s thyroiditis 430f
Extrahepatic Headache 378, 387
Dawson’s fingers 372f chronic 433
Diaphragmatic hernia, cholangiocarcinoma 283f
Extra­thoracic heart 5f recurrent 331, 356
congenital 3 Hemangioma 251f, 272
Diffuse idiopathic skeletal Extrophic bladder 833
cavernous 272f
hyperostosis 389f, 390 F Hemangiosarcoma 252
Diffuse reticulogranular Hemarthrosis, subacute 102f
opacities 7f Fatal quadriparesis 386 Hematocele, subacute 239f
Digital subtraction angiogram Feeding pericallosal arteries 327f Hematocolpometra 227
378f, 379f Fever 181 Hematoma
Disseminated encephalomyelitis Fibromuscular dysplasia 184, extradural 368f, 370
373 184f, 340, 418 subcapsular 192
Distal fibular metaphysis 112f Fibrosarcoma 150 Hemorrhage 104f, 277, 366f, 376f
Distal interphalangeal joints 119f Fibrous dysplasia 129f, 131, 148 subarachnoid 380
Distal ureteric obstruction 205f Fissural nodules 65f Hepatic ducts, dilatation of 283f
Double posterior cruciate Flank pain, chronic 188 Hepatic hemangioma,
ligament sign 109 Flowers appearance, Bouquet hepatic 270f
Draining infrafalcine vein 328f of 428 Hepatolithiasis 283
Duodenal ulcer 310f Flowing candle wax appearance Hereditary multiple diaphyseal
Duodenojejunal junction 292f 140f sclerosis 143
Dysarthria 386 Football injury 97 Hiatus hernia 256f, 257
Dysembryoplastic Fournier’s gangrene 234-236 Hilar arteries, central 25f
neuroepithelial tumor Frank hematuria 215 Hip joint 75
360 Frontoparietal cortex 344f idiopathic transient
Dysmorphic facies 318 Fungal infections 14 osteoporosis of 136
Dysplasia, bronchopulmonary 8f Fungal osteomyelitis 145 transient osteoporosis of 136
Dyspnea 49 Fungi 145 Hip metastasis 204
Dysuria 181 Hirschsprung’s disease 253, 253f
G
Hirschsprung’s tends 253
E
Gallbladder 277f HIV 431
Ectopia cordis 5f calculi 268 Hoffa’s fat pad 126
Edema, pulmonary 61f Gangliogliomas 360 Horseshoe kidneys 156f, 157f,
Eisenmenger’s syndrome 25 Ganglioneuroblastomas 56 158f, 159f, 160
Embryonal cell carcinoma 439 Ganglioneuromas 56 fusion of 156f
Index 445

Hydatid disease 45, 88 Ischial spines 241 McCune-albright syndrome 131


Hydrocephalus 321 Ischiopubic rami 148f Meckel’s diverticulum 265
progressive 325 Mediastinal lymphadenopathy
Hydronephrosis 188f, 422f J 44f
bilateral 169f Jaundice 284 Medulla oblongata 347f
Hydroureter 188f Jejunal polyp 259 Medullary nephrocalcinosis
Hypercalcemia, familial 116 Jejunojejunal intussusception 427f, 428
Hyperechoic hilum 195f 258f Medullary sponge kidney 428
Hyperparathyroidism 116f Joubert syndrome 319 Medulloblastomas 362, 365, 365f
Hypertension 167, 192, 426 Juvenile dermatomyositis 121 Meningioma 345, 352
Hypodense foci 215f, 275f Meningitis 338
central 221f K Meniscal/parameniscal cyst 125f
Hypodense mass 230f, 269f Mesenteric ischemia 291
Hypoglycemia 322 Kaposi sarcoma 14 Mesial temporal sclerosis
Hypoplasia, cerebellar 318f Kidney, ureter and bladder 187f 371, 371f
Hypotonia 318 Klatskin tumor 283 Metastatic melanoma 265
Hypoxia 322 Klein’s line 76 Metastatic renal cell
Knee, swelling of 99 carcinoma 201
I Metatarsophalangeal joint
L
Idiopathic pulmonary fibrosis 63f 119, 435f
Ileocecal junction 292f Laceration, pulmonary 34f Micturating cystourethrogram
Iliac arteries 418f Langerhans cell histiocytosis 150 169f
Iliac bones, bilateral uniform Laparotomy 259f Middle cerebral artery
sclerosis of 140f Leprosy 144 bifurcation 380
Ilio­lumbar ectopic kidney 161f Leriche syndrome 417, 418 saccular aneurysms 379f
Infarction, pulmonary 46 Lipoblastoma 40 Midureteric calcular obstruction
Infarcts, cerebellar 340 Lipoma 40, 154f 218f
Inferior vena cava 281f Liposarcoma 40 Mild perilesional edema 365f
Inflammatory bowel disease Littoral cell angioma 252 Milkman’s fractures 117f
78, 292 Liver disease, chronic 118 Molar tooth sign 318, 319
Inguinal hernia 83 Liver function tests 284 Multidetector computerized
Injuries, abdominal 287, 288 Lymph node enlargement 54 tomography 25
Intense bone edema 104f Lymphadenopathy, cervical 53 Multiple cranial vault
Internal auditory canal 353f Lymphoblastic leukemia 54 swellings 82f
Internal cerebral vein 329 Lymphoma 365, 431 Multiple cystic bronchiectasis 42f
Internal echogenic debris 227f Multiple endocrine neoplasia
M 116, 388
Internal hernia 299
Interstitial edema, pulmonary 30 Madelung’s disease 40 Multiple hypoechoic lesions 431f
Intra-abdominal abscess 295 Madura foot 145 Multiple meningiomas 346f
Intracanalicular vestibular Magnetic resonance Multiple pulmonary
schwannoma 354f, 355 cholangiopancreato­ metastases 200f
Intracranial aneurysms 380 graphy 2677f Multiple renal and hepatic
Intracranial tumors 322 Malignant meningioma 352 cysts 165f
Intraglandular lymph nodes 431f Mammogram 410f Multiple sclerosis 373, 373f
Intrahepatic biliary bilateral 412f Multisystem granulomatous
dilatation 283 Mandible swelling and disorder 65
Intrahepatic biliary radicles 283f discomfort, bilateral angle Musculoskeletal system
Intramammary lymph node 411f of 431 imaging 71
Intravenous urogram 161f, 185f, Mass, cerebellar 361f Mycetoma 145
186f, 243f Massive right cerebellar Mycobacterium avium
Ipsilateral Sylvian fissure 371f infarct 340 complex 14
446 A Teaching Atlas of Case Studies in Diagnostic Imaging

Myeloma 148 Parotitis 431 Psoriatic arthropathy 78


Myelomeningocele 144 Patchy hypodensities 175f, 179f Pubic symphysis 83
Myositis ossificans 133 Pelvic appendix 304f Pulmonary arteries, lobar and
Pelvicalyceal distension 205f segmental branches
N Pelviureteric junction obstruction of 26f
Nabothian cyst 442f 168, 168f Pulmonary cavitation, causes
Nasal congestion 81 Penumbra sign 113f of 45
Necrotizing enterocolitis Pericardial calcification 24 Pulmonary metastases,
246f, 247 Pericardial defects 37 bilateral 215f
Nephroblastoma 214 Pericolic fat stranding 314f Pyelonephritis 175, 177
Neurofibromas 56 Perihepatic hemoperitoneum bilateral acute 180
Neurofibromatosis 346, 418 287f emphysematous 178
Neuropathic arthropathy 144 Perinatal ischemic anoxic injury,
consequence of 340 R
Nevoid basal cell carcinoma
syndrome 74 Peripheral pulmonary
Rachitic rosary 118
Nonenhancing cysts 248f arteries 61f
Rasmussen’s aneurysm 67
Non-Hodgkin’s lymphoma 14 Peripheral vascular
Rectourinary fistula 240f
Nonuniform pulmonary malformations 85
Rectum, persistent gross
aeration 8f Perirenal hematoma 196f
distension of 253f
Nonvascular cystic mass 86 Perirenal urinoma 190f
Reducible inguinal hernias,
Normal liver parenchyma 273f Perivesical fat planes 205f
bilateral 169
Normal renal arteries 172f Persistent obstruction 192
Reiter’s syndrome 78
Pharyngotympanic tube 382
Renal cell carcinoma 204
O Pheochromocytoma 223,
Renal cysts, demonstration
388, 388f
Obstructive uropathy 207 of 163f
Pilocystic astrocytoma 362
Orchidectomy 213, 438 Renal disease 118
Pleural fluid collection,
Orchidopexy 213 Renal failure 167
bilateral 23f
Osmotic myelinolysis 386 Renal parenchymal injury 192
Plexus carcinoma, choroid 365
Osteomalacia 117f Renal tubular dysfunction 192
Pneumocystis jiroveci
Osteopetrosis 143 Retroareolar mass 410f
pneumonia 14, 17, 63
Osteopoikilosis 139 Rhabdomyosarcoma 147
Polycystic kidney 166f
Ovarian follicular cyst 442f Ribbing disease 143
Polyostotic fibrous dysplasia 143
Ribs, surface-shaded display
Pontine meningiomas 346f
P of 286f
Pontine myelinolysis, central 386
Rosenthal, basal vein of 329
Pain abdominal 158, 275, 284, Popliteal fossa, subcutaneous
Rubella 322
305, 311, 355 layer of 154f
Painful micturition 170 Popliteal synovial cysts 88 S
Paint brush appearance 428 Posterior communicating
Pampiniform plexus 237f artery 380 Saccular aneurysm 379f
Pancolitis 308f Posterior longitudinal ligament, Sacroiliac joint 148f
Pancreatic duct 267f, 283f ossification of 389f complete fusion of 77f
Pancreatic necrosis 277 Posterior urethral valve 171 Sail sign 1f, 2
Pancreatic pseudocysts 279f Postgadolinium injection 400f Sapho syndrome 78
Pancreatitis 276f, 277 Postrenal transplant Sarcoidosis 46, 65, 65f, 373
Paraduodenal hernias 299 lymphocele 194 Sarcoma 209
Paragangliomas 223 Pott’s disease 13 synovial 88
Paraparesis 127 Presinusoidal periportal Scaphoid bone, fracture of 105f
Parenchymal granulomas 384 fibrosis 274 Scaphoid fractures 105
Parenchymal hypodensities 281f Prostate hypertrophy 220 Schistosoma mansoni
Parenchymal nodular opacities, Proteinuria 426 infestation, hepatic 274
bilateral 64 Pseudocyst formation 277 Schistosomiasis 209
Parenchymal tuberculoma 384 Pseudofractures 117f Sclerosing cholangitis 283, 284f
Index 447

Scrotal skin avulsion 239 Telephone receiver Urethral stricture 220


Scrotal varicocele 238f appearance 72 Urethrocystogram, retrograde
Sebaceous cysts 88 Temporal fossa arachnoid 220
Seizures 341 cyst 335 Urinary bladder 195f, 304f, 306
recurrent 333 Tension pneumoperitoneum carcinoma 209
Seminoma 438 310f distension of 235f
Sepsis 340 Teratoid tumor 365 multiple sacculations of 170f
Shoulder joint 144f Teratoma 231, 438f ultrasound of 423f
Sickle cell disease 10 Terry Thomas or David Letterman Urinary tract infection 159, 427
Sigmoid carcinoma 314f sign 106f recurrent 156, 158, 172
Slipped capital femoral Testicular germ cell tumors 439 Urinoma 192
epiphysis 76 Testicular microlithiasis 439 Urogenital system imaging 156
Small cell carcinoma 209 Thalassemia 294 Usual interstitial pneumonia 63f
Small vessel ischemia 373 Thanatophoric dysplasia 71 Uterus, chambers of 232f
Soft tissue sarcomas 147 Thenar muscles 402f
Solitary osteochondroma 89f
V
Thoracolumbar spine 415f
Spaghetti sign 216f, 217 Thornwaldt’s cyst 434 Vague abdominal pain 251, 426
Sphenoid sinus 366f Thyroid gland 110f Valsalva maneuver 237f, 238
Spinal cord 415f Thyroid swelling 429, 430 Valvular obstruction, congenital
injury 144 Tibial nerve 399f 171
lipoma 392f, 393 Tinnitus 353 Vasculitis 373
Spinal tumors 398 TORCH infections 322 Vastus intermedius muscle 97f
Spine, cervical 390f Toxoplasmosis 322 Vastus lateralis muscle 96f
Spinning-top appearance 169f Trachea, cervical 110f Vein of Galen 325f, 326f, 327,
Splenic hemangiomas 252 Tracheal injury 111 327f-329f
Spondylitis, ankylosing 77 Transcranial Doppler Venous thrombosis 277
Spring onion deformity 423 sonogram 328f, 329 Vertebrae, osteolytic destruction
Squamous cell carcinoma 422f Transependymal interstitial of 13f
Staphylococcus aureus 183 edema 364f Vertebral arteries 340
Stomach 312f Transitional cell carcinoma 160 Vertigo 353
Strongyloidiasis 45 Traumatic pulmonary Vesicoureteral reflux 170f,
Sturge-Weber syndrome 322 laceration 46 173, 174
Subdural hematoma, Tuberculosis 14, 291 Vestibular schwannoma 355
subacute 369f pulmonary 20, 23 Vomiting 305, 311
Subfalcine herniation 368f Tuberculous meningitis 384 von Hippel-Lindau disease 362
Subhepatic mass 262f Tuberous sclerosis 322 von Hippel-Lindau syndrome
Superior mediastinal lipoma 58 Tumor cells, extension of 282 388
Superior mesenteric artery 293 Turcot syndrome 365 W
Superior mesenteric vein
289f, 293 U Wegener’s granulomatosis 14
Surfactant deficiency Whirl sign 293
syndrome 7, 8 Ulna, distal metaphysis of 93f Widespread metastatic disease
Symphysis pubis, typical Undescended testis 213 148
Undisplaced fractures 286 Wilm’s tumor 160, 214
diastasis of 83f
Unicameral bone cyst 92f, 95 Winslow, foramen of 299
T Unilateral renal agenesis 426
Upper abdominal pain 275 Z
Takayasu arteritis 418 Ureterovesical junction Zenker’s diverticulum 254f, 255
Target sign 290, 403f, 404 obstruction, acquired 83 Zuckerkandl, organ of 388

You might also like